Você está na página 1de 224

CRIMINAL PROCEDURE

Prepared by Andrew Carvajal Instructor: Prof. Alana Klein Fall 2008

Notes: 1. Most of the material summarized corresponds to the casebook Learning Canadian Criminal Procedure 9th Edition by Stuart et al. 2. Feminine pronouns are generally used as neutral pronouns throughout the text, unless the facts of the case require otherwise. 3. Many thanks to the law clerks with the tedious task of putting case headnotes together. Their work doesnt go unappreciated.

Index
Jurisdiction............................................................................................................................................5 Legislative division of powers.........................................................................................................5 Classification of offences.................................................................................................................6 Indictable offences............................................................................................................................6 A note on jurisdiction.......................................................................................................................7 Summary conviction offences..........................................................................................................9 Crown elections offences (dual, hybrid)........................................................................................10 Time limitation in Canadian law....................................................................................................11 Territorial limitations......................................................................................................................11 Police Stopping Powers......................................................................................................................15 Doctrine of ancillary police powers...............................................................................................17 Stop Powers and Racial Profiling...................................................................................................17 1. Vehicle Stops..............................................................................................................................17 2. Investigative Detention...............................................................................................................18 3. Roadblock Stops.........................................................................................................................21 Search and Seizures............................................................................................................................24 Before the Charter...........................................................................................................................24 Search and seizure under the Criminal Code.................................................................................24 Two special warrant powers to search the person..........................................................................28 Three main requirements of search warrants.................................................................................28 Search and seizure under the common law....................................................................................29 Three types of search powers recognized at common law............................................................29 A) Search incident to Arrest...........................................................................................................29 B) Consent to search.......................................................................................................................37 C) Consent incident to duties of police officers if reasonably necessary (ancillary powers doctrine)..........................................................................................................................................39 Doctrine of ancillary police powers...............................................................................................41 Provincial statutes...........................................................................................................................43 Constitutional minimum standards.................................................................................................43 Lecture by Sarah Henningsson - Provincial prosecutor.................................................................57 Arrest...................................................................................................................................................58 Citizens Power of Arrest...............................................................................................................58 Constitutional Minimum Standards (Charter ss. 9 and 10(a)).......................................................63 2

Reasons for Arrest..........................................................................................................................65 Entry into Premises.........................................................................................................................67 Meaning of Arrest...........................................................................................................................73 Appearance Notice..........................................................................................................................75 Interrogation Right to Counsel and Arrest Informational duties....................................................77 Right to counsel (Charter s. 10(b)).................................................................................................78 1. Triggering mechanisms..............................................................................................................79 2. Implementation duties................................................................................................................89 3. Waiver and duty to be reasonably diligent in exercise of right.................................................95 Right to Silence...............................................................................................................................99 Confession rule...............................................................................................................................99 What is the Crown requested to show under the voluntary confession rule?..............................100 Bail Hearings....................................................................................................................................107 The rational for the Bail Reform Act of 1961..............................................................................107 Show cause hearings.....................................................................................................................125 Grounds for Detention..................................................................................................................126 3. The Judge Must Decide............................................................................................................133 4. Reasonable Conditions on Release..........................................................................................134 5. Charter Standards.....................................................................................................................134 Disclosure and Discovery.................................................................................................................136 Distinction between disclosure and production...........................................................................136 Production of Third Party Records...............................................................................................139 Lost evidence cases......................................................................................................................141 Bill C-46.......................................................................................................................................141 Preliminary Inquiry...........................................................................................................................144 Preferring the Indictment..............................................................................................................144 Preliminary inquiry as discovery vehicle.....................................................................................146 Test for committal: sufficiency of evidence.....................................................................................147 Constitutional Remedies...................................................................................................................151 Exclusion of evidence...................................................................................................................152 Judicial stay as abuse of process..................................................................................................161 Pleas and Plea Bargaining................................................................................................................167 Plea Bargaining.............................................................................................................................173 The Trial Process: Preliminary Considerations................................................................................178 3

Right to counsel at trial (Charter s. 7)..........................................................................................178 Adjournments...............................................................................................................................183 Trial within a reasonable time (Charter s. 11(b)).........................................................................185 Formal attacks on the information or indictment.........................................................................191 Three reasons to challenge the indictment...................................................................................192 A) Grounds...................................................................................................................................192 1. Insufficiency.............................................................................................................................192 2. Duplicity...................................................................................................................................197 3. Improper joinders of Counts.....................................................................................................199 B) Saving devices.........................................................................................................................201 3. On appeal..................................................................................................................................206 The Trial Process: Jury Trials...........................................................................................................206 Challenge to array.........................................................................................................................208 Peremptory challenges..................................................................................................................211 Challenges for cause.....................................................................................................................213 The process of challenges.............................................................................................................213 The Trial Process Further Considerations.....................................................................................217 Res Judicata..................................................................................................................................218 1. Double jeopardy.......................................................................................................................219 When are matters considered identical?.......................................................................................221 2. Multiple punishment.................................................................................................................223

Jurisdiction
Sources of criminal procedure Constitution Statutes o Not limited to the Criminal Code; Competition Act, Tax Act Common law/judge-made law o Procedural defences created by the courts Rules of Court o S.486 of the CC gives allows the courts to create their own rules o They cannot be inconsistent with statutes but they are there

Legislative division of powers


s. 91 - Constitution Act, 1867 It shall be lawful for the Queen, by and with the Advice and Consent of the Senate and House of Commons, to make Laws for the Peace, Order, and good Government of Canada, in relation to all Matters not coming within the Classes of Subjects by this Act assigned exclusively to the Legislatures of the Provinces; and for greater Certainty, but not so as to restrict the Generality of the foregoing Terms of this Section, it is hereby declared that (notwithstanding anything in this Act) the exclusive Legislative Authority of the Parliament of Canada extends to all Matters coming within the Classes of Subjects next hereinafter enumerated; that is to say, ... 27. The Criminal Law, except the Constitution of Courts of Criminal Jurisdiction, but including the Procedure in Criminal Matters. 28. The Establishment, Maintenance, and Management of Penitentiaries. s. 92 - Constitution Act, 1867 In each Province the Legislature may exclusively make Laws in relation to Matters coming within the Classes of Subjects next hereinafter enumerated; that is to say, ... 6. The Establishment, Maintenance, and Management of Public and Reformatory Prisons in and for the Province. ... 13. Property and Civil Rights in the Province. 14. The Administration of Justice in the Province, including the Constitution, Maintenance, and Organization of Provincial Courts, both of Civil and of Criminal Jurisdiction, and including Procedure in Civil Matters in those Courts. 15. The Imposition of Punishment by Fine, Penalty, or Imprisonment for enforcing any Law of the Province made in relation to any Matter coming within any of the Classes of Subjects enumerated in this Section.

Legislative jurisdiction over criminal law and procedure is given to the Federal Parliament under s.91 of the Constitution Act But provinces can enact penal legislation for enforcement for matters that would otherwise fall under provincial jurisdiction, provided that they do not infringe on Parliaments exclusive jurisdiction The federal Parliament has enacted the procedure to be followed for the prosecution of all crimes enacted by the Criminal Code and federal legislation Provincial provisions vary, yet they mostly mirror federal ones The interpretation section of the Criminal Code (s.2) set out for each of the provinces the meaning of the court or appeal, court of criminal jurisdiction, magistrate, territorial division, and superior court of criminal jurisdiction The prosecution of federal matters will depend on whether the matter concerns the Criminal Code or other federal statutes Look at the interpretation section of the Criminal Code (s.2) in relation to the definitions of prosecutor and Attorney General The enforcement of the Code will normally be given to the Attorney General of the provinces, while the enforcement of other general legislation is given to the Attorney General of Canada R. v. Trimarchi (1987) determined that a provincial judge is entitled to try offences created by the Code and other federal legislation In A.G. Canada v. C.N. Transportation (1983), the court held that whether it was criminal legislation or not, the federal government could authorize who would prosecute it

Classification of offences
At common law a distinction was drawn between indictable offences triable only by judge and jury and offences triable only summarily by justices of the peace sitting alone In Canada there are, broadly speaking, four types of offences 1. Indictable conviction 2. Indictable summary conviction 3. Dual (hybrid) offences 4. Contraventions (regulatory) offences under the Contraventions Act The wording of the criminal offence states the type of offence Indictable offences There are the more serious offences Usually the offences, or its penalty section, set out the punishment applicable to the offence o Otherwise, s.743 of the Code provides a maximum of 5 years

Where the accused is charged with an indictable offence, she must in general be present at all stages of proceedings (except for the exceptions provided in 650 650.02 of the Code) There are mainly three types of indictable offences, and their classification affects their forum of trial and jurisdiction o The most serious are given to the exclusive jurisdiction of the superior court of criminal jurisdiction (see ss. 468 and 469) o The least serious are absolutely within the jurisdiction of a provincial court (s. 553) o For the great bulk of offences, the accused is entitled to choose her model of trial (s. 536(2)) If the accused does not elect a mode of trial, she would have been deemed to have elected trial by judge and jury, s. 565(1)(c)

A note on jurisdiction Every superior court of criminal jurisdiction has jurisdiction to try any indictable offence as per s. 468 of the Code o This does not imply that every superior court will do so, however, see s. 469 Other courts of criminal jurisdiction are allowed to try anything except for the offences described there o This refers to the exclusive jurisdiction of the superior court Provincial court judges are said to have absolute jurisdiction because they are absolutely entitled to try s. 553 offences without being dependent on the accuseds choice As for the rest of offences, the jurisdiction lies on the superior court except for Quebec o See s. 552 and the definition of judge o Here the accused will have the choice of jurisdiction Trial by jury, trial by judge alone or trial in provincial jurisdiction s. 468 Criminal Code Superior court of criminal jurisdiction Every superior court of criminal jurisdiction has jurisdiction to try any indictable offence. s. 469 Criminal Code Court of criminal jurisdiction Every court of criminal jurisdiction has jurisdiction to try an indictable offence other than (a) an offence under any of the following sections: (i) section 47 (treason), (ii) section 49 (alarming Her Majesty), (iii) section 51 (intimidating Parliament or a legislature), (iv) section 53 (inciting to mutiny), (v) section 61 (seditious offences), (vi) section 74 (piracy), (vii) section 75 (piratical acts), or (viii) section 235 (murder); 7

Accessories (b) the offence of being an accessory after the fact to high treason or treason or murder; (c) an offence under section 119 (bribery) by the holder of a judicial office; Crimes against humanity (c.1) an offence under any of sections 4 to 7 of the Crimes Against Humanity and War Crimes Act; Attempts (d) the offence of attempting to commit any offence mentioned in subparagraphs (a)(i) to (vii); or Conspiracy (e) the offence of conspiring to commit any offence mentioned in paragraph (a). PART XIX: INDICTABLE OFFENCES TRIAL WITHOUT JURY s. 552 Criminal Code In this Part, "judge" juge "judge" means, (a) in the Province of Ontario, a judge of the superior court of criminal jurisdiction of the Province, (b) in the Province of Quebec, a judge of the Court of Quebec, (c) in the Province of Nova Scotia, a judge of the superior court of criminal jurisdiction of the Province, (d) in the Province of New Brunswick, a judge of the Court of Queens Bench, (e) in the Province of British Columbia, the Chief Justice or a puisne judge of the Supreme Court, (f) in the Provinces of Prince Edward Island and Newfoundland, a judge of the Supreme Court, (g) in the Province of Manitoba, the Chief Justice or a puisne judge of the Court of Queens Bench, (h) in the Provinces of Saskatchewan and Alberta, a judge of the superior court of criminal jurisdiction of the province, (i) in Yukon and the Northwest Territories, a judge of the Supreme Court, and (j) in Nunavut, a judge of the Nunavut Court of Justice. Quebec appears to be the exception, since all other courts mentioned are federal 8

s. 553 Criminal Code Absolute jurisdiction The jurisdiction of a provincial court judge, or in Nunavut, of a judge of the Nunavut Court of Justice, to try an accused is absolute and does not depend on the consent of the accused where the accused is charged in an information (a) with (i) theft, other than theft of cattle, (ii) obtaining money or property by false pretences, (iii) unlawfully having in his possession any property or thing or any proceeds of any property or thing knowing that all or a part of the property or thing or of the proceeds was obtained by or derived directly or indirectly from the commission in Canada of an offence punishable by indictment or an act or omission anywhere that, if it had occurred in Canada, would have constituted an offence punishable by indictment, (iv) having, by deceit, falsehood or other fraudulent means, defrauded the public or any person, whether ascertained or not, of any property, money or valuable security, or (v) mischief under subsection 430(4), where the subject-matter of the offence is not a testamentary instrument and the alleged value of the subject-matter of the offence does not exceed five thousand dollars; (b) with counselling or with a conspiracy or attempt to commit or with being an accessory after the fact to the commission of (i) any offence referred to in paragraph (a) in respect of the subject-matter and value thereof referred to in that paragraph, or (ii) any offence referred to in paragraph (c); or (c) with an offence under (i) section 201 (keeping gaming or betting house), (ii) section 202 (betting, pool-selling, book-making, etc.), (iii) section 203 (placing bets), (iv) section 206 (lotteries and games of chance), (v) section 209 (cheating at play), (vi) section 210 (keeping common bawdy-house), (vii) [Repealed, 2000, c. 25, s. 4] (viii) section 393 (fraud in relation to fares), (viii.1) section 811 (breach of recognizance), (ix) subsection 733.1(1) (failure to comply with probation order), (x) paragraph 4(4)(a) of the Controlled Drugs and Substances Act, or (xi) subsection 5(4) of the Controlled Drugs and Substances Act. Summary conviction offences These convictions are set in Part XXVII of the Criminal Code 9

Summary offences are time barred by 6 months The maximum penalty is 2,000 dollars or six month imprisonment or both (s.787) o In theory, Parliament could establish a penalty of five years less one day without violating the right to a jury trial under s. 11(f) of the Charter Some summary conviction can have convictions to imprisonment longer than 6-months o See s.271.1 Parliament may want to give a higher punishment, yet allow more flexibility to the prosecution in avoiding a preliminary inquiry These can be called super-summaries o There are some examples of offences involving summary conviction which have higher fines Summary convictions are always tried in Provincial Court or by justice of the peace (where applicable) o While superior court judges have the jurisdiction to try these, they rarely exercise it Provincial offences are all summary offences What is known as the indictment for indictable offences is called an information for summary convictions Accused individuals charged with a summary offence do not normally need to appear in court, but may instead chose to have a lawyer or some other agent present, unless the judge requires otherwise (s. 800(2)) Summary convictions are appealed to a summary conviction appeal court with a judge of the superior court seating alone o Indictable offences are appealed at the court of appeal of the province

Crown elections offences (dual, hybrid) The prosecutor can decide whether to prosecute as a summary conviction or indictable offence o This should be indicated prior to trial An example of this types of offences is Fraud (s. 380) If the choice is to proceed by indictment, the forum of the trial will depend on the type of indictable offence involved Various considerations are taken by the Crown in the election o Higher penalty of indictable offences o Prior convictions of the accused o The desire to require the accuseds personal presence o Wishing to get the matter over more quickly In 1994, Parliament increased the maximum penalty for several hybrid offences proceeded against by way of summary conviction to 18 months If the Crown chooses summary proceedings, the accused no longer has the choice of a preliminary inquiry under s. 536 S. 34(1) of the Interpretation Act (see below) has been applied to equate hybrid offences with indictable offences in contexts such as arrest and fingerprinting s. 34(1) - Interpretation Act 10

Where an enactment creates an offence, (a) the offence is deemed to be an indictable offence if the enactment provides that the offender may be prosecuted for the offence by indictment; ... Even in looking at the powers of arrest without warrant we see the importance of the classification of offence S.495 o Officers can arrest without warrant people who have committed indictable offences, or any person fleeing from any type of offence The classification also applies to judicial interim release

Time limitation in Canadian law


Summary convictions are barred by 6-months of the commission of the offence (s. 786(2)) s. 786(2) Criminal Code Limitation No proceedings shall be instituted more than six months after the time when the subject-matter of the proceedings arose, unless the prosecutor and the defendant so agree. For the case of indictable offences there is only time limitation, which is a 3-year bar to the crime of treason (see s. 48(1)) Provincial offences usually specify their own time limitation

Territorial limitations
In the time of jury trial it was natural that criminal trial should be conducted at the place where the criminal offence occurred While the jury system has changed, there is still a right for a community to see firsthand that justice is made with respect to matters that concern it The general common principle can be seen today in s. 6(2) of the Code s. 6(2) Criminal Code Offences outside Canada Subject to this Act or any other Act of Parliament, no person shall be convicted or discharged under section 730 of an offence committed outside Canada. There are certain statutory exceptions to this principle Certain offences on aircrafts (s. 7(1)) 11

Certain offences against internationally protected protected person pr property (s. 7(3)) Hostage taking (s. 7(3.1) Offences involving nuclear material (s. 7(3.2)) War crimes (s. 7(3.71)) Terrorist offences (ss. 7(3.73) 7(3.75)) Child sex tourism (s. 7(4.1)) Treason (s. 46(3)) Others include forging a passport, using a citizenship certificate fraudulently, priacy, bigamy, and conspiracy

R. v. Greco (2001), 159 CCC (3d) 146 (Ont. CA) Facts Appeal by the accused from his conviction for breach of probation Greco and a female companion named Smith travelled to Cuba for a holiday,during which he viciously assaulted her The Cuban authorities refused to become involved The police in Toronto would not charge Greco for the assault It was subsequently discovered that, at the time of the holiday, Greco was subject to a probation order after he was convicted of assault and threatening death o The order required him to keep the peace and be of good behaviour. Greco tried to have the charge stayed . He relied upon section 6(2) of the Criminal CodeAt issue was whether Greco was. The second question was whether Issues: 1. Was the accused obligated to comply with the probation order outside of Canada? 2. Does the Ontario court have jurisdiction to try him for this offence. Holding: 1. Yes, 2. Yes; appeal dismissed Moldaver JA 1. Territorial limitations applied to the ability of the court to enforce its orders However, such limitations did not affect the jurisdiction of the court to make orders that applied outside of Canada. Various provisions existed in the Code that asserted such jurisdiction Probation orders had to apply outside of Canada for policy reasons The probation order did not require an express provision for it to apply outside of Canada 2. The court had jurisdiction to try and convict Greco There was a real and substantial connection between the offence and this country The connection was that Canada had an interest to ensure that its court orders were 12

complied with Section 6(2) did not apply to this situation.

Re Bigelow and R (1982), 37 OR (2d) 304 Facts The appellant was charged with unlawful detention of his infant son The appellant and his wife were separated, with custody of their infant son given to the wife with access to the appellant every second weekend o The wife and son resided in Ontario On a weekend allotted to him, the appellant removed the son to Alberta, whereupon the police were required to intervene to return the son to the wife The appellant was committed for trial in Ontario, where he brought a motion for certiorari to quash for lack of jurisdiction The trial judge dismissed the application, whereupon the appellant appealed. Issue: Could the accused be charged in Ontario for having removed his son to another province? Holding: Yes; appeal dismissed The Court In considering the question of jurisdiction over inter-provincial offences, Ontario courts would have no jurisdiction if an offence were WHOLLY committed in another province However, Ontario courts would have jurisdiction over an offence if any element could be said to have been committed in Ontario This required an analysis of the acts of an accused on a flexible basis and included three categories: first, a continuity of operation extending from Ontario to the other province; second, the commission of an overt act in Ontario; or, third, the registration of effects in Ontario from acts committed in other provinces Ramifications of the offence occurred in Ontario, and the execution of the offence, including the obtaining of possession of the child and boarding of the plane, took place in Ontario, together with a refusal to redeliver the child to Ontario The offence of detention is a "withholding", which was beyond the mere confinement in Alberta

Jurisdiction under the Charter


s. 24 Canadian Charter of Rights and Freedoms (1) Anyone whose rights or freedoms, as guaranteed by this Charter, have been infringed or denied may apply to a court of competent jurisdiction to obtain such remedy as the court considers appropriate and just in the circumstances. (2) Where, in proceedings under subsection (1), a court concludes that evidence was obtained in a manner that infringed or denied any rights or freedoms guaranteed by this Charter, the evidence shall be excluded if it is established that, having regard to all the circumstances, the admission of 13

it in the proceedings would bring the administration of justice into disrepute. Charter remedies under s.24 can be given by a court of competent jurisdiction In R. v. Mills (1986) the SCC held that all criminal trial courts as defined by s. 2 of the Code were courts of competent jurisdiction (this includes provincial and federal courts) o However, a justice at a preliminary inquiry would not be a court of competent jurisdiction under s.24 of the Charter o The SCC also determined that s.24 of the Charter does not confer any jurisdiction in the form of a any new rights of appeal This position is no longer clear The SCC has not directly pronounced on the issue of standing of complainants at trial to raise Charter rights, but it has conferred such jurisdiction in some of its rulings As for s. 52 of the Charter In R. v. Big M Drug Mart (1985), the SCC held that s. 52 has a remedial role in criminal law independent of s. 24 An accused may defend a charge arguing that the law under which the charge has been brought was constitutionally invalid The Charter normally has no remedy when the conduct of officials in question occurred outside of Canada s. 52(1) Constitutional Act, 1982 The Constitution of Canada is the supreme law of Canada, and any law that is inconsistent with the provisions of the Constitution is, to the extent of the inconsistency, of no force or effect.

Timeline of Criminal Proceedings


Charge information containing name and offence. If judge is satisfied then complaint can be laid. Its the decision of the judge, certainly not of complainant. o Summons goes out. Youve been charged, show up at this court at this time, if you dont youre guilty of another offence o Arrest not everyone arrested will be charged for the thing they were arrested, must be brought before judge within 24 hours. o Once someone is charged, there must be a judicial decision formalizing the charge of the accused. Its the judge who decides, not crown Disclosure Judicial interim release (bail) Preliminary Inquiry used to be to test whether there was sufficient evidence to go to trial and also to give defence chance to test crown witnesses and get testimony on record o The accused used to be entitled to it, but now you have to ask for it, specify which witnesses you want to hear and what you want to hear from them (practically this is never refused but technically it could be) Pre-trial motions (e.g. based on the Charter) 14

Preferred indictment (can supersede preliminary inquiry) o I.e. gives crown right to take away preliminary inquiry o E.g. Paul Bernardo case, allows crown to spare family from testifying twice. Its entirely at the discretion of the crown. Trial o Information is a sworn declaration o If s.469 offence, or other offence in the middle, then document at trial is called an indictment Verdict Sentence Appeal o Part 21 of the Code for indictable offences

Police Stopping Powers


Some of the issues which are salient when looking at police stopping powers The rule of law Limits if state authority vis-s-vis the Charter Treatment of aboriginals by the criminal justice system The state of affairs before the Charter in relation to stopping powers Police were entitled to detain people for the purposes of investigation There are some limits, however, on police power o For long they have not be allowed to randomly stand and ask people for identification o Deadman is a good example of the state of the law before the Charter o Dickson J (dissenting) states what the main state of the law was then While the police was assumed to have stopping powers to question an individual, she would not be obliged to answer question If the police were to stop someone because of failure to cooperate they would need reasonable belief that the person had committed an offence or was about to commit an offence Absent of statutory or clear common law provision, the powers of the police s. 9 Canadian Charter of Rights and Freedoms Detention or imprisonment Everyone has the right not to be arbitrarily detained or imprisoned. s. 10 Canadian Charter of Rights and Freedoms Arrest or detention

15

Everyone has the right on arrest or detention a) to be informed promptly of the reasons therefor; b) to retain and instruct counsel without delay and to be informed of that right; and c) to have the validity of the detention determined by way of habeas corpus and to be released if the detention is not lawful. R. v. Deadman (1985), 46 CR (3d) 193 Facts The accused was randomly stopped as part of a spot check program, whose principal aim is to detect, deter and reduce impaired driving He was accused with failing, without reasonable excuse to provide a breath sample, but was acquitted On appeal, the acquittal was confirmed as it was held that there was neither statutory nor common law authority to require him to stop his motor vehicle, so he had a reasonable excuse The Ontario CA reversed the acquittal Issue: Did the police officer possess authority, either statutory or at common law, to require the appellant to stop his motor vehicle? Holding: Yes; appeal dismissed Le Dain J The test laid down in R. v. Waterfield, while generally invoked in cases where it is at issues whether a police officer had acted in the execution of her duties, has been recognized as a test for whether the officer had common law authority to do what she did Applying the Waterfield test, the random vehicle stop was a prima facie unlawful interference with liberty since it was not authorized by statute The right to circulate in a motor vehicle on the public highway may be described as a liberty; however, it cannot be regarded as a fundamental liberty like an individual's right of movement, since it is a licensed activity subject to regulation and control for the protection of life and property The random stop falls within the general scope of police duties to prevent crime and to protect life and property by the control of traffic as these are the very objects of the program, a measure intended to improve the deterrence and detection of impaired driving The stop was not an unjustifiable use of police power because it was both necessary to the execution of police duty and reasonable, having regard to the nature of the liberty interfered with and the importance of the public purpose served by the interference Dickson CJ (dissenting) It has always been a fundamental tenet of the rule of law that police, in carrying out their general duties as law enforcement officers have limited powers and are only entitled to interfere with the liberty or property of the citizen to the extent authorized by law The fact that a police officer has a general duty to prevent crime and protect life and property does 16

not mean that he or she can use any or all means for achieving these ends. In the criminal law, the rules and principles relating to arrest establish justifiable limits upon a citizen's liberty Short of arrest, the police have never possessed legal authority at common law to detain any one against his or her will for questioning or to pursue an investigation These random stops are indistinguishable from detention for questioning or investigation whether or not they might be committing a criminal offence and, without validly enacted legislation to support them, are unlawful It would be contrary to the long standing protection accorded individual liberty by the common law and detrimental to the individual's fundamental right to be free from arbitrary interference to conclude that this action of the police was authorized and lawful The fact that driving a motor vehicle is a licensed activity subject to regulation and control in the interests of safety is irrelevant to police power if the conditions for licensing have been met and are adhered to Lecture Notes This case establishes the new police power of investigative detention The Waterfield test is the source of the doctrine of ANCILLARY POLICE POWERS o It came about, however, in trying to determine whether a police officer was acting within her legally prescribed authority The general duties of police officers, were a common law authority that would justify this type of stopping If the powers of the police officer are prima facie unlawful, you can check whether the actions fall within the general powers of the police, and whether they are justifiable o The stops were justifiable given the importance of highway traffic and the fact that the activity was a licensed one A crucial point about this case, was whether the creation of these powers was a matter of the legislature or courts could do it as well Since then, many provinces have enacted statutory authorizations for random road stops

Stop Powers and Racial Profiling 1. Vehicle Stops


In R. v. Hufsky (1988) and R. v. Ladouceur (1990) the SCC held that detention in the unfettered discretion of a police officer is necessarily arbitrary and contrary to s.9 of the Charter At issue in both cases, was the constitutionality of the spot check procedure of s.216(1) of the Ontario Highway Safety Act However, Le Dain J states that the limit on this Charter rights was justified under the s.1 test by the importance of highway safety, the gravity of motor vehicle accidents, and the role played by random stopping in helping detect dangerous drivers which may otherwise not be detected by mere observation of driving 17

Hufsky dealt with a situation where the random sport check was in a fixed location, while in Ladouceur the random stop was roving

According to the Report of the Commission on Systemic Racism in the Ontario Criminal Justice System (1995), black men in the Toronto metropolitan are particularly vulnerable to being stopped by the police In the SCC case of R. v. Mellenthin (1992), Cory J decided that while random stops were justifiable in order to reduce mort vehicle accidents, and as such could include licence, sobriety and ownership tests, the scope of the check could not be extended beyond these aims A man who was found to be in possession of hash oil through a random vehicle check was acquitted as the evidence related to this offence was excluded Random blocks could not be used to go on fishing expeditions unrelated to driving In R. v. Orbanski (2005), the SCC held that implied in the operation requirements of drunk driving legislation were police powers to ask motorists whether they had anything to drink and administer sobriety tests

2. Investigative Detention
R. v. Simpson (1993), 20 CR (4th) 1 Facts Acting on information that a particular house was suspected to be a "crack house", a police officer patrolled the area and observed a woman leaving the place with the accused The police officer, who had no information pertaining to either person, followed them and stopped the vehicle The police officer noticed a bulge in the accused's front pant pocket, he touched it and felt a hard lump o At that point the officer did not have reasonable grounds to arrest the accused o He asked the accused to remove the object, which turned out to be a baggie containing cocaine The accused was charged with possession of cocaine for the purpose of trafficking The trial judge rejected the accused's argument that his rights under ss. 9 and 8 of the Canadian Charter had been infringed, and he was convicted Issue: Was the accused arbitrarily detained in contravention of his s.9 Charter right? Holding: Yes; appeal allowed Doherty JA The accused was arbitrarily detained contrary to s. 9 of the Charter The police officer admitted that his decision to stop the motor vehicle had nothing to do with the enforcement of laws relating to the operation of motor vehicles o Hence he was not statutorily authorized Once road safety concerns are removed as a basis for the stop, the powers associated upon those particular concerns cannot be relied on to legitimize the stop 18

The detention was also not authorized by the common law Where an individual is detained by the police in the course of efforts to determine whether that individual is involved in criminal activity being investigated by the police, that detention can only be justified if the detaining officer has some ARTICULABLE CAUSE for the detention o There must exist a constellation of objectively discernable facts which give the detaining officer reasonable cause to suspect that the detainee is criminally implicated in the activity under investigation o A hunch based entirely on intuition cannot suffice. However, something less than the grounds required to support an arrest will suffice. The presence of an articulable cause does not render any detention for investigative purposes a justifiable exercise of a police officer's common law powers The inquiry into the existence of an articulable cause is only the first step in the determination of whether the detention is justified There was no articulable cause in this case justifying the detention The police officer had information of unknown origin that another police officer had been told that the residence was believed to be a crack house He had no reason to suspect that the accused or the driver of the car was involved in criminal activity Lecture notes Using the Waterfield test, while there is a general police duty which is being carried out here by the detaining Constable, the detention is not justified The articulable cause introduced by Doherty JA is the justification for detention o The standard is lower than reasonable and probable grounds needed for detention The cases of Terrence and Thompson influenced this lower standard of detention The case does not talk about the cause needed for search, nor s.10 Charter rights and the right to counsel o It also fails to mention whether this lower standard of detention allows for the use of force R. v. Brown (2003), 9 CR (6th) 240 This case is the leading pronouncement on racial profiling Morden JA Racial profiling involves the targeting of individual members of a particular racial group, based on the supposed criminal propensity of the group The attitude underlying racial profiling may be consciously or unconsciously held The test to be applied under s. 9 of the Charter, is whether the police officer who stopped the motorist had articulable cause for the stop Articulable cause exists where the grounds are reasonable and can be clearly expressed 19

If a police officer stops a person based on her colour or other discriminatoiry ground, there is no articulable cause

To succeed, the accused had to prove that it was more probable than not that there was no articulable cause and that the real reason for the stop was race I.e. there is a reverse burden Comments The reverse burden to prove that the cause for arrest was race has been dismissed by the Ontario CA Courts tend to reject racial profiling claims where race was only part of the reason for police intervention Some lower courts favoured a reverse onus for racial profiling, but Doherty JA for the Ontario CA in Peart v. Peel Police Services Board (2006), held that while the reality of racial profiling cannot be denied, it could not accept it as the rule rather than the exception when police detain a black person R. v. Mann, [2004] 3 SCR 59 Facts As two police officers approached the scene of a reported break and enter, they observed Mann, who matched the description of the suspect, walking casually along the sidewalk They stopped him, he identified himself and complied with a pat-down search of his person for concealed weapons During the search, one officer felt a soft object in his pocket, and as he reached into the pocket he found a small plastic bag containing marijuana The accused was arrested and charged with possession of marijuana for the purpose of trafficking The trial judge found that the search of his pocket contravened s. 8 of the Charter and evidence was excluded under s. 24(2) The Manitoba CA set aside the acquittal and ordered a new trial, finding that the detention and the pat-down search were authorized by law and were reasonable in the circumstances Issue: Was the police officer justified in looking into the accuseds pockets? Holding: No; appeal allowed Iacobucci J The police were entitled to detain the accused for investigative purposes and to conduct a pat-down search to ensure their safety, but the search of his pockets was unjustified and the evidence discovered therein must be excluded Although there is no general power of detention for investigative purposes, police may detain an individual if there are reasonable grounds to suspect that the individual is connected to a particular crime and that the detention is reasonably necessary on an objective view of the circumstances

20

Investigative detentions carried out in accordance with the common law power recognized in this case will not infringe the detainee's rights under s. 9 of the Charter Investigative detentions should be brief in duration, so compliance with s. 10(b) will not excuse prolonging, unduly and artificially, any such detention They do not impose an obligation on the detained individual to answer questions posed by the police At a minimum, individuals who are detained for investigative purposes must be advised, in clear and simple language, of the reasons for the detention Where a police officer has reasonable grounds to believe that his safety or the safety of others is at risk, the officer may engage in a protective pat-down search of the detained individual The investigative detention and protective search power must be distinguished from an arrest and the incidental power to search on arrest In this case, the seizure of the marijuana contravened s. 8 of the Charter The officers had reasonable grounds to detain the accused and to conduct a protective search, but no reasonable basis for reaching into his pocket Deschamps J (dissenting) concurred with the majoritys analysis on the issues of a common law power to detain, but disagreed that the evidence obtained would bring justice into disrepute, so as to exclude it pursuant to s.24(2) of the Charter There is a common law power to detain and search those who the police have an articulable cause to believe have been or will be involved in the commission of a criminal offence In formulating the standard which must be met in order to give rise to the common law power to detain, the term "articulable cause" is preferable to the term "reasonable grounds to detain" o Using the term "reasonable grounds" could lead to the erroneous conclusion that the same degree of justification is required to detain as to arrest, which would undermine the very purpose of the common law power to detain A search incidental to detention has to be rationally connected to the purpose of the initial detention and reasonably necessary to either to ensure the security of police officers or the public, to preserve evidence or to prevent the escape of an offender o (I.e. she expands the scope of what a lawful search incidental to detention can encompass, and does not limit it to the security of the officer like the majority) Comment Iacobucci believes that courts can create powers of detention to adjust the common law to police practices While this case does not expand on the scope of investigative detention, it does raise some constitutional limitations on the power of search

3. Roadblock Stops
21

R. v. Clayton and Farmer (2007), 47 CR (6th) 219 (SCC) Facts A 911 caller indicated that a number of persons were openly displaying handguns in a strip club's parking lot, and identified four of the cars in the lot Within minutes, police stopped the first car leaving from the lot, yet this car was not one of the cars described by the caller When asked to step out of the car, Farmer protested twice, giving rise to concerns for the officers' safety. Meanwhile, Clayton gave strange and evasive answers o Clayton then ran away from police, was chased and subdued o Clayton was searched by police and a loaded prohibited handgun was found in his pocket o Farmer was also searched and a loaded prohibited handgun was found under his jacket The Court of Appeal quashed their convictions concluding that the accused's Charter rights under sections 8 and 9 were violated o According to the CA, the roadblock was unlawful as there was no individualized suspicion of criminal activity, and there were no specific statutory powers to establish roadblocks Issue: Did the roadblock violate the accuseds Charter rights? Holding: No; appeal allowed Abella J The police constables lawfully exercised their common law powers when they detained and searched the accused In determining the boundaries of police powers, caution was required to ensure the proper balance between preventing excessive intrusions on an individual's liberty and privacy, and enabling the police to do what was reasonably necessary to perform their duties in protecting the public Searches incident to an investigative detention could be justified if the officer believed on reasonable grounds that his or her own safety, or the safety of others, was at risk This determination is made based on the totality of the circumstances: the seriousness of the offence; the information known to the police about the suspect or the crime; and, the extent to which the detention was reasonably responsive or tailored to these circumstances, including its geographic and temporal scope The initial and continuing detentions of Clayton and Farmer's car were justified based on the information the police had, the nature of the offence, and the timing and location of the detention Requiring police to stop only those vehicles described in the 911 call imposed an unrealistic burden on police in this case, which was inconsistent with their duty to respond in a timely manner, at least initially, to the seriousness of the circumstances Based on their observations, there were reasonable grounds for the police to conclude that the two 22

occupants of the car they had stopped were implicated in the crime being investigated The officers' safety concerns also justified the searches incidental to the lawful investigative detention Binnie J (concurring) On occasion the Crown will argue that a common law which authorizes police conduct that infringes individual Charter freedoms may nevertheless be justified in the larger interest of society The majority applies the pre-Charter test set out in Waterfield, however, continued use of the Waterfield approach without modification not only adds to the problematic elasticity of common law police powers, but sidesteps the real policy debate in which competing individual and societal interests should be clearly articulated in the established framework of Charter analysis The common law Waterfield test should be modified where police claim a common law power that is challenged on Charter grounds by requiring a court first to determine whether the power claimed by the police exists at common law, and if so whether the common law would, if valid, authorize police interference with Charter rights, and finally whether the interference thus authorized can be justified under s. 1 of the Charter If necessary, the court may also have to determine in a particular case whether the power was exercised reasonably by the police in the totality of the circumstances. The common law authority of detention by a carefully tailored blockade in response to a 911 gun call constitutes a reasonable limit under s. 1 of the Charter on the rights of the accused guaranteed by s. 9 of the Charter to be free of arbitrary detention Section 1 is the proper place to weigh the accuseds individual rights against societys collective rights The protection of society from the flaunting of illegal handguns in a crowded public place is clearly a pressing and substantial public purpose An investigation prompted by a gun call engages fundamental issues of public peace and public order. Handguns pose a serious and growing societal danger The roadblock was a rational response to the 911 gun call The blockade in this case minimally impaired the accuseds right to be free from arbitrary detention. The roadblock was tailored to the information given to the police and anything less than a full blockade would not have served the purpose of the roadblock The salutary effects of the roadblock exceeded its deleterious effects Lecture notes The scope of an investigative detention is a balancing exercise Compared to Simpson, there was more specific information with regards to the suspects, and the presence of guns and the seriousness of the offence made the detention more justifiable than if it had been a lesser offence The reasonably necessary standard in looking at the totality of circumstances comes from the second branch of the Waterfield test Binnie J in the dissent objects to an absence of Charter analysis and that the second branch of the Waterfield case seems to not require a Charter analysis 23

o Better to use Charter analysis than a British case decided 20 years before the advent of the Charter o The Oakes test provides a better assessment of the balancing act to be made between the objectives pursued by the State and individual freedom

Search and Seizures


In Canada, all powers to search and seize are now subject to s.8 of the Charter, which has forced courts to grapple with the balance between the rights of a citizen to be free from state intrusion and the need of state intervention for a legitimate purpose s. 8 Canadian Charter of Rights and Freedoms Search or Seizure Everyone has the right to be secure against unreasonable search or seizure. Before the Charter, the purpose of a search warrant was to avoid an action in trespass The action in trespass or other writs would be used to determine the illegality of police conduct There was no exclusionary requirement Because of the lack of remedies and a clear procedure to avoid illegal searches, this area of the law was largely underdeveloped A good example of this is Colet v. R.

Search and seizure under the Criminal Code


The search powers conferred by the Criminal Code are often restricted to peace officers as defined in s.2, which include police, correctional guards custom officers, mayors and certain members of the Canadian Forces The powers to search and seize without a search warrant are restricted to certain stipulated offences Possession of weapons, ss.117.02(1) and 117.04(2) Common gaming houses, s.199(2) Impaired driving, s.254(2) to (4) Suspected stolen timber, s.339(3) Cockpits, s.447(2) Counterfeit money, s.462 These shows that there are very few statutory grounds for warrantless searches Under ss.487-489 a justice is empowered to issue a search warrant to search in relation to an offence under the Criminal Code or other federal statute

24

s. 487 Criminal Code Information for search warrant (1) A justice who is satisfied by information on oath in Form 1 that there are reasonable grounds to believe that there is in a building, receptacle or place (a) anything on or in respect of which any offence against this Act or any other Act of Parliament has been or is suspected to have been committed, (b) anything that there are reasonable grounds to believe will afford evidence with respect to the commission of an offence, or will reveal the whereabouts of a person who is believed to have committed an offence, against this Act or any other Act of Parliament, (c) anything that there are reasonable grounds to believe is intended to be used for the purpose of committing any offence against the person for which a person may be arrested without warrant, or (c.1) any offence-related property, may at any time issue a warrant authorizing a peace officer or a public officer who has been appointed or designated to administer or enforce a federal or provincial law and whose duties include the enforcement of this Act or any other Act of Parliament and who is named in the warrant (d) to search the building, receptacle or place for any such thing and to seize it, and (e) subject to any other Act of Parliament, to, as soon as practicable, bring the thing seized before, or make a report in respect thereof to, the justice or some other justice for the same territorial division in accordance with section 489.1 ... s. 487.1(1) Criminal Code Telewarrants Where a peace officer believes that an indictable offence has been committed and that it would be impracticable to appear personally before a justice to make application for a warrant in accordance with section 256 or 487, the peace officer may submit an information on oath by telephone or other means of telecommunication to a justice designated for the purpose by the chief judge of the provincial court having jurisdiction in the matter. ... s. 487.11 Criminal Code Where warrant not necessary A peace officer, or a public officer who has been appointed or designated to administer or enforce any federal or provincial law and whose duties include the enforcement of this or any other Act of Parliament, may, in the course of his or her duties, exercise any of the powers described in subsection 487(1) or 492.1(1) without a warrant if the conditions for obtaining a warrant exist but by reason of exigent circumstances it would be impracticable to obtain a warrant. Exigent circumstances provision 25

s. 488 Criminal Code Execution of search warrant A warrant issued under section 487 or 487.1 shall be executed by day, unless (a) the justice is satisfied that there are reasonable grounds for it to be executed by night; (b) the reasonable grounds are included in the information; and (c) the warrant authorizes that it be executed by night. s. 489 Criminal Code Seizure of things not specified (1) Every person who executes a warrant may seize, in addition to the things mentioned in the warrant, any thing that the person believes on reasonable grounds (a) has been obtained by the commission of an offence against this or any other Act of Parliament; (b) has been used in the commission of an offence against this or any other Act of Parliament; or (c) will afford evidence in respect of an offence against this or any other Act of Parliament. Seizure without warrant (2) Every peace officer, and every public officer who has been appointed or designated to administer or enforce any federal or provincial law and whose duties include the enforcement of this or any other Act of Parliament, who is lawfully present in a place pursuant to a warrant or otherwise in the execution of duties may, without a warrant, seize any thing that the officer believes on reasonable grounds (a) has been obtained by the commission of an offence against this or any other Act of Parliament; (b) has been used in the commission of an offence against this or any other Act of Parliament; or (c) will afford evidence in respect of an offence against this or any other Act of Parliament. s. 29 Criminal Code Duty of person arresting (1) It is the duty of every one who executes a process or warrant to have it with him, where it is feasible to do so, and to produce it when requested to do so. The Code speaks of searching a building, receptacle or place, can one then obtain a warrant to search person? Laporte v. Laganire (1972), 18 CRNS 357 26

Facts The police had reason to suspect that the petitioner was involved in a hold-up, which had taken place about a year and a half before There are scars on the petitioner's neck and shoulder which resemble bullet wounds. X-rays reveal the presence in the petitioner's shoulder of a foreign body, a metallic object corresponding in size and shape to a 38-calibre slug To remove the slug it would require more than minor or superficial surgery o Doctors were however in agreement that the operation, if performed, would require a general anaesthetic and ipso facto would involve a certain element of risk to the petitioner o They also agreed that they would not normally perform such an operation without the consent of the patient, which they do not have The case then deals with is a writ of certiorari for the purpose of quashing and setting aside a search warrant issued by justice Laganire Issue: Did justice Laganire have jurisdiction to order a search warrant of the petitioners body? Holding: No; appeal allowed Hugessen J All parties conceded that there is no precedent in point that they have been able to find either in Canadian or British case law The issues arose in the US, however, in Re Crowder, where Curran C.J. authorized the issuance of a search warrant for the surgical removal of what was thought to be a bullet "lying superficially beneath the skin" of a prisoner's forearm o This case relied on another US case Schmerber v. California, which concerned the result of a blood test, and whereby the majority of the US Supreme Court repeatedly emphasized the minor nature of the "operation", which involved an almost total lack of risk o It is to be noted that in Crowder the search warrant was authorized under provisions of the Federal Criminal Rules of Practice The question is then, does the right to search the person exist either under the provisions of the Criminal Code or at common law? Is it possible to say that a living human body is a "building, receptacle or place" (see s.487) into which a surgical intrusion may be justified by means of a search warrant? Clearly it is not a "building", nor a "receptacle" o Receptacle cannot be held to include the interior of a living human body With regards to place, the use of this word throughout the Code is indicative of the meaning intended by Parliament, which refers to a geographic and not an anatomical location o It is clear that the word "place" was never intended to extend to or to include the interior of a human body The criminal law has always had to strike the precarious balance between the protection of society on the one hand and the protection of the rights of individuals 27

Both rights are equally important, but any conflict between them must wherever possible be resolved in a manner most compatible with individual human dignity The constant preoccupation of our courts with the protection of the citizen against the state results in the Crown having always to bear the burden in any criminal prosecution The possibility that some guilty persons may escape the net of justice is not too high a price to pay for the right to live in freedom

It is my duty to interfere and prevent what I can only describe as a grotesque perversion of the machinery of justice and an unwarranted invasion upon the basic inviolability of the human person Lectures Notes Parliament responded quickly to this with powers to search the person Under s.487(1) police officers dont even have the power to search individuals Parliament has recently established two special warrant powers to search the person A blood sample where the person is reasonably suspected to have caused bodily harm due to impaired driving and is unable to consent to taking the sample, s.256 Obtaining bodily substances for forensic DNA analysis in the investigation of certain offences, such as murder, sexual assault, kidnapping, robbery, see ss.487.04-487.09 S.11 of the Controlled Substances Act also allows police powers to search a person Parliament had also authorized post-conviction DNA samples for a national DNA bank (see ss. 487.051-487.091) The constitutionality of this was upheld in R. v. Rodgers (2006) by the SCC The scheme of s. 487(1) has been judicially interpreted to include three main requirements of search warrants 1. The informant must present the justice with information upon oath that there are sufficient factual details to confer jurisdiction 2. The judge must act judicially in the independent assessment of the facts and in exercising discretion as to whether to issues the search warrant 3. The search warrant must obtain sufficient description of the objects of search in relation to category and offence The 1985 Criminal Code amendment added the possibility of a new telewarrant procedure for search warrants (s. 487.1(1)). and a procedure for the seizure of documents in the possession of a lawyer with whom solicitor-client privilege is maintained Nonetheless, in R. v. Lavellee, Rackel & Heintz (2002), the SCC struck down the provision dealing with solicitor-client privilege warrants as contrary to s.8 of the Charter o The court declared detailed common law rules that would guide this procedure instead Despite the procedure laid out in the Code many illegal warrants are issued in practice, as ultimately the efficacy of the procedure rest on the quality of those who are justices 28

S.11(d) of the Charter holds that a person is to be presumed innocent until found guilty by an independent and impartial tribunal The Ontario CA has held that both provincial judges and justices of the peace are independent and impartial o See R. v. Valente (2004), which was further confirmed by the SCC o For Quebec see Universal Spa Lte v. Valois (1986) and Charest v. Lipp (1990)

Even Form 1 (see below) at the end of the Criminal Code does not include the reasons why the warrant is being issued in relation to whether there is evidence on a particular crime, there may be evidence on a crime, or there will be evidence on a crime
FORM 1 (Section 487) INFORMATION TO OBTAIN A SEARCH WARRANT Canada, Province of ................, (territorial division). This is the information of A.B., of ................ in the said (territorial division), (occupation), hereinafter called the informant, taken before me. The informant says that (describe things to be searched for and offence in respect of which search is to be made), and that he believes on reasonable grounds that the said things, or some part of them, are in the (dwelling-house, etc.) of C.D., of ................, in the said (territorial division). (Here add the grounds of belief, whatever they may be.) Wherefore the informant prays that a search warrant may be granted to search the said (dwelling-house, etc.) for the said things

Search and seizure under the common law


The common law is an important source of search powers Common law searches are searches without a search warrant, since search warrants can only be based on valid legislation Three types of search powers recognized at common law 1. Search incident to arrest, or investigative detention 2. Consent searches 3. Searched pursuant to the ancillary powers doctrine

A) Search incident to Arrest

29

Unlike the search powers awarded in s.487, search incident to arrest does not require reasonable and probable grounds The arrest itself must be lawful, but once the standard for arrest is met, the standard for the search is lower and must satisfy mere reasonableness R. v. Brezack, [1949] OR 888 Facts The accused was charged with the assault of a police constable The police, having made some investigations, had reason to believe that the accused had in possession some narcotics which were concealed in his mouth Acting on the information they had, they approached the accused and seized him by his arms while holding his throat to prevent him from swallowing o The three of them fell to the ground and a struggle ensued there o The constable persistently tried to insert his finger in the accuseds mouth and was bit by him o When he finally succeeded in doing so, he discovered no drugs in his mouth o He did not have any narcotics on his clothes either They then took him to his car where they found two other individuals as well as some capsules of containing narcotics He was then taken into custody and later charged for assaulting the police officer Issue: Was the search of the accuseds mouth lawful? Holding: Yes; appeal dismissed Robertson CJO The evidence in this case shows that the constable was engaged in the lawful execution of his duty as a peace officer in making the arrest (given the information he had), and that the attempt to search the inside of appellant's mouth was a justifiable incident of that arrest That the appellant was liable to arrest without a warrant is beyond question, and the evidence afforded by the capsules containing a narcotic, found in appellant's motor car strongly supports the reasonableness of the constable's belief in the information he had It is important to observe that the search that was made is justifiable as an incident of the arrest The constable who makes an arrest has important duties, such as to see that the prisoner does not escape by reason of being armed, and to see if any evidence of the offence for which he was arrested is to be found upon him A constable may not always find his suspicions to be justified by the result of a search, it is sufficient if the circumstances are such as to justify the search as a reasonable precaution There was an arrest here when the constables seized the person of the appellant The evidence would indicate that they did not inform him immediately that he was arrested, and of the cause of the arrest. Constables have a task of great difficulty in their efforts to check the illegal traffic of drugs While it is important that constables should be instructed that there are limits upon their 30

right of search, including search of the person, they are not to be encumbered by technicalities in handling the situations with which they often have to deal in narcotic cases Lecture Notes This is a pre-Charter case Laporte v. Laganire supra (1972), 18 CRNS 357 Facts For full facts see above Issue: The court refused to extend the power to search incident to arrest to include a surgical operation Hugessen J There is no doubt that there is a common law right to search a prisoner at the time of her arrest This common-law right of search is limited to that search which is incidental to the making of an arrest or the continued detention of the prisoner in safe custody The reasons for such right are is to make the arrest effective, to ensure that evidence does not disappear and to prevent the commission of a further offence Examples would be to ascertain whether a suspect has weapons that could harm a police officer or others, or to recover evidence of the crime which the accused allegedly committed Nothing in the jurisprudence justifies a surgical intrusion into the body of a prisoner months after his arrest, for the purpose of obtaining evidence against him on a different charge than that for which he is being held R. v. Tomaso (1989), 70 CR (3d) 152 Facts The accused was involved in a car crash which resulted in a fatality The police obtained blood from a free-flowing wound while the accused was unconscious in hospital to determine whether he was impaired while driving Issue: Was the seizure of the accused's blood while he was unconscious unreasonable? Holding: Yes; appeal allowed Howland CJO At common law a police officer has the power to search a person as incident to a lawful arrest, and to take from his person any property which the officer reasonably believes is connected with the offence charged, or which may be used as evidence against the person arrested, or any weapon or instrument found upon the person arrested In R. v. Debot the Ontario CA went further and held that some searches incident to arrest can PRECEDE THE ACTUAL ARREST, provided that the police officer had reasonable grounds, prior to the search, for arresting the respondent under

31

In this case the seizure of the blood sample took place on July 14, however, the police officer decided to await the analysis of the blood sample and arrest the accused on July 29 It has not been established that the police officer had reasonable grounds on July 14 for the arrest, nor could it be said that the analysis of the blood sample was not part of the justification for such arrest In any case, it would be going too far to attempt to justify the seizure on July 14 on the basis that it was incident to an arrest more than two weeks later, for the offences with which the appellant was charged It should be noted that since the offence the Criminal Code has been amended (s. 254(3)) allowing a peace officer, who believes on reasonable and probable grounds that an offence is or has been committed as a result of the consumption of alcohol, the right to require that a person have blood samples taken by or under the direction of a qualified medical practitioner This right may be exercised only where there are reasonable and probable grounds to believe that the taking of breath samples is not possible, or is impracticable, by reason of any physical condition of the person Section 256 similarly makes provision for the issue of a warrant for the taking of blood samples where there is an accident resulting in the death of or bodily injury to any person, and the medical practitioner is of the opinion that the person is unable to consent, but that the taking of the samples would not endanger his life or health o These amendments did not come into force until after the offence in question Lecture Notes While a search may precede the arrest, the grounds for the arrest had to be present at the time of the search R. v. Stillman (1989), 70 CR (3d) 152 Facts The accused was arrested in 1991 for the brutal murder of a teenage girl He was the last person seen with the victim on the night of the crime The victim died from wounds to the head. Semen was found in her vagina and a human bite mark had been left on her abdomen. At the police station, the accuseds lawyers informed the police by letter that the accused was not consenting to provide any bodily samples, including hair and teeth imprints, or to give any statements o Once the lawyers left, police officers took, under threat of force, scalp hair samples from the accused and he was made to pull some of his own pubic hair; Plasticine teeth impressions were also taken o A police officer then interviewed the accused for an hour in an attempt to obtain a statement o The accused sobbed throughout the interview and, after being permitted to call his lawyer, he went to the washroom escorted by an officer where he used a tissue to blow his nose, which was retrieved from the wastebasket for DNA testing The accused was subsequently released but was arrested again several months later. At that time, a dentist took new impressions of the accuseds teeth without his consent in a procedure lasting two hours. More hair was taken from the accused, as well as a saliva 32

sample and buccal swabs Following a voir dire held to determine the admissibility of certain evidence, the trial judge found that the hair samples, buccal swabs and teeth impressions had been obtained in violation of s. 8 of the Charter but concluded that the evidence was nevertheless admissible o With respect to the tissue containing mucous, he found that it had not been obtained in violation of s. 8 and should thus be admitted The accused was later convicted by a jury of first degree murder The majority of the CA upheld the trial judges ruling and affirmed the verdict Issue: Was the evidence obtained in contravention to the accuseds s. 8 rights? Holding: Yes; the appeal was allowed and a new trial ordered Cory J There are three requirements which must be met if a search is to be found reasonable: (a) it must be authorized by law; (b) the law itself must be reasonable; and (c) the manner in which the search was carried out must be reasonable At the time the seizure occurred, the Criminal Code only provided a procedure for obtaining a warrant to search a "building, receptacle or place"; it did not authorize the search of a person, nor the seizure of parts of the body, and as such the seizure was not statutorily authorized Three conditions must be satisfied in order for a search to be validly undertaken pursuant to the common law power of search incident to a lawful arrest 1. The arrest must be lawful 2. The search must have been conducted as an "incident" to the lawful arrest 3. The manner in which the search is carried out must be reasonable 1. The accuseds arrest was lawful since the police officers, subjectively, were under the impression that they had reasonable and probable grounds to believe that the accused had committed the murder 2. The common law power of search incidental to arrest does not extend beyond the purpose of protecting the arresting officer from armed or dangerous suspects or of preserving evidence that may go out of existence or be otherwise lost The power of search incidental to arrest cannot be so broad as to encompass the seizure without valid statutory authority of bodily samples in the face of a refusal to provide them These samples are usually in no danger of disappearing Parliament has recently amended the Code so as to create a warrant procedure for the seizure of certain bodily substances for the purposes of DNA testing o If this type of invasive search and seizure came within the common law power of search incident to arrest, it would not have been necessary for the government to create a parallel procedure for the police to follow LHeureux-Dub (dissenting) While the search and seizure at issue was clearly not necessary in order to protect the accused, the police or the public, they were authorized by law under the common law search power incidental to 33

arrest The following guidelines strike a proper balance between society's interest in detecting and punishing crime and the individual's interests in personal privacy and autonomy over his own body where the taking of bodily samples or impressions occurs as an incident to arrest 1. The police must exercise their discretion to conduct the search given all of the circumstances 2. The search must be for a valid objective in pursuit of the ends of criminal justice without running counter to the general objectives of the proper administration of justice 3. Consideration must be given to the intrusiveness involved in the search: the more intrusive, the higher the threshold for finding that the taking of bodily samples or impressions is both justified and conducted in a reasonable manner in given circumstances 4. The police must have reasonable and probable grounds to conduct the search 5. The search must be predicated on sufficiently important circumstances; those circumstances will generally be established where: (a) it is impracticable to obtain a warrant to secure the desired evidence; (b) such evidence cannot be obtained by a less intrusive means; (c) there is no alternative evidence available; (d) the offence for which the arrest was made is a serious one; and (e) public policy is served by the type of search at issue 6. The manner in which the search is cinducted myst not be abusive or unreasonable given the totality of circusmtances [As for the discarded tissue, the court divided more narrowly (5:4) held that the seizure violate s. 8 of the Charter] Cory J Where an accused who is not in custody discards an item offering potentially valuable DNA evidence, the police may ordinarily collect and test the item, since the accused abandoned the item and ceased to have a reasonable expectation of privacy in it The situation is different, however, when an accused is in custody Here, the accused had announced through his lawyers that he would not consent to the taking of any samples of his bodily fluids In these circumstances the seizure was unreasonable The tissue containing the mucous, however, should not be excluded The police did not force, or even request, a mucous sample from the accused Where an accused who is not in custody discards an item offering potentially valuable DNA evidence, the police may ordinarily collect and test the item without any concern about consent since, in the circumstances, the accused abandoned the item and ceased to have a reasonable expectation of privacy in it. The violation of the accuseds Charter rights with respect to the tissue was not serious. The seizure did not interfere with the accuseds bodily integrity, nor cause him any loss of dignity In any event, the police could and would have obtained the discarded tissue. It was discoverable and the administration of justice would not be brought into disrepute if the evidence obtained from the mucous sample were to be admitted 34

McLachlin J (dissenting) The tissue was not obtained as a result of a search of the appellant nor was it seized from him; he had discarded it To put it another way, the appellant had abandoned any privacy interest in the tissue that he may have had Lecture Notes The court disagrees with the wide powers of search incidental to arrest awarded in Cloutier and does not seem to allow a search incidental to arrest that would lead to the discovery of evidence, beyond its preservation R. v. Caslake, [1998] 1 SCR 51 Facts An RCMP officer, several hours after arresting the accused for possession of narcotics, conducted an inventory search of the accused's impounded car pursuant to police policy and found $14,000 in cash and two individual packages of cocaine He did not have the accuseds permission or a search warrant The accused unsuccessfully appealed his conviction of possession of marijuana for the purposes of trafficking and of possession of cocaine to the Manitoba CA At issue here was whether of Rights and Freedoms which guarantees the right to be secure against unreasonable search or seizure, and if not consistent, whether the evidence should have been admitted Issue: Was the search of the car inconsistent with s. 8 of the Charter? Holding: Yes (but the evidence should not be excluded); appeal dismissed Lamer CJ Since search incident to arrest is a common-law power, there are no readily ascertainable limits on its scope It is therefore the courts' responsibility to set boundaries which allow the state to pursue its legitimate interests, while vigorously protecting individuals' right to privacy There are three main purposes of search incident to arrest 1. Ensuring the safety of the police and public 2. The protection of evidence from destruction at the hands of the arrestee or others 3. The discovery of evidence which can be used at the arrestee's trial The restriction that the search must be "truly incidental" to the arrest means that the police must be attempting to achieve some valid purpose connected to the arrest Whether such an objective exists will depend on what the police were looking for and why; there are both SUBJECTIVE and OBJECTIVE aspects to this issue o The police must have ONE of the purposes for a valid search incident to arrest in mind when the search is conducted (objective purpose) o Further, the officer's belief that this purpose will be served by the search must 35

be a reasonable one (subjective purpose) It would be contrary to s.8 of the Charter to allow searches incident to arrest which do not meet these subjective and objective criteria There is no need reasonable and probable grounds, the only requirement is that there be some reasonable basis for doing what the police officer did

Delay and distance do not automatically preclude a search from being incidental to arrest, but they may cause the court to draw a negative inference That inference may be rebutted by a proper explanation A police search of the car for the purpose of finding evidence which could be used at the trial on the charge of possessing marijuana for purposes of trafficking would have been well within the scope of the search incident to arrest power, as there was clearly sufficient circumstantial evidence to justify a search However, the police cannot rely on the fact that, objectively, a legitimate purpose for the search existed when that is not the purpose for which they searched Agents of the state must act in accordance with the rule of law; hence, they must not only objectively search within the permissible scope but also turn their mind to this scope before searching, and satisfy themselves that there is a valid purpose for the search Here, the purpose of the search was to inventory the contents of the vehicle which falls outside the bounds of the legitimate purposes of search incident to arrest Bastarache J The search, given that the arrest was lawful, was incidental to the arrest because it was related to the arrest, rather than the arrest's being incidental to the search Regardless of the police officer's subjective belief in the purpose and justification for his inventory search, the officer had the right to search the vehicle (as part of the accuseds immediate surroundings) The question of delay was immaterial for the search to qualify as "incidental". This power draws its authority from the arrest itself It is not necessary to establish reasonable and probable grounds independently to conduct a search incidental to an arrest The common law power to search incident to an arrest is not unreasonable and does not violate s. 8 of the Charter if it is consistent, in the circumstances, with the proper administration of justice The issue must be whether the inventory search was truly incidental to the arrest and reasonably performed, and not whether the Charter was infringed because the police officer could have obtained a warrant Lecture Notes Caslake seems to narrow the scope of search incidental to arrest by adding the subjective standard Here it is clear (unlike Stillman) that the search power incidental arrest can include the 36

discovery of new evidence o The standard is still reasonableness Caslake was applied to authorize the seizure of a briefcase as incident to an arrest of a person found in possession of a stolen vehicle (see R. v. Mohamad (2004))

B) Consent to search
Law Reform Commission of Canada, Working Paper 30: Police Powers Search and Seizure in Criminal Law Enforcement The common law tolerance of search with consent is founded on the principles that one who has invited to an act being done towards her cannot, when she suffers from it, complain of it as a wrong Few cases of consent search appear to have been litigated in the Canadian context o The discretionary basis upon which police forces have adopted practices to carry out consent searches points to wide gaps in Canadian law R. v. Deadman supra (1985), 46 CR (3d) 193 Facts The accused was randomly stopped as part of a spot check program, whose principal aim is to detect, deter and reduce impaired driving He was accused with failing, without reasonable excuse to provide a breath sample, but was acquitted On appeal, the acquittal was confirmed as it was held that there was neither statutory nor common law authority to require him to stop his motor vehicle, so he had a reasonable excuse The Ontario CA reversed the acquittal Issue: Was the seizure lawful because of the accuseds consent? Le Dain J The accused's compliance with the signal to stop did not alter the legal basis on which it must be justified Police officers, when acting or purporting to act in their official capacity as agents of the state, only act lawfully if they act in the exercise of authority which is either conferred by statute or derived as a matter of common law from their duties The reason for this is the authoritative and coercive character of police action A person should not be prevented from invoking a lack of statutory or common law authority for a police demand or direction by reason of compliance with it in the ABSENCE OF A CLEAR INDICATION from the police officer that the person is free to refuse to comply Because of the intimidating nature of police action and uncertainty as to the extent of police powers, compliance in such circumstances cannot be regarded as voluntary in any meaningful sense

37

A person should not be penalized for compliance with a signal to stop by having it treated as a waiver or renunciation of rights, or as supplying a want of authority for the stop Dickson CJ (concurring) Police officers only act lawfully when they exercise authority conferred upon them by statute or at common law The apparent voluntary compliance by a citizen with a police request to stop a motor vehicle cannot alter the legal basis which must justify such police action when it is challenged in later proceedings Having regard to the authoritative and coercive character of police requests, submission to a police officer's exercise of apparent authority, such as a demand to stop at a roadblock, cannot be characterized as voluntary or consensual unless it was clear to the person at the time that he was free to refuse to comply Deadman has now been relied upon to hold that the consent to a search must have been real and voluntary, and that the person searched must have sufficient awareness to have waived the constitutional right conferred by s.8 of the Charter, and the individual must be aware of the consequences of giving consent See R. v. Nielson (1988), R. v. Wills (1992) and R. v. Borden (1994( Note on R. v. Wills (1992), 70 CCC (3d) 529 Doherty JA The application of the waiver doctrine to situations where it is said that a person has consented to what would otherwise be an unauthorized search or seizure requires that the Crown establish on the balance of probabilities that: 1. There was a consent, express or implied 2. The giver of the consent had the authority to give the consent in question 3. The consent was voluntary in the sense that it was not the product of police oppression, coercion or other external conduct which negated the freedom to choose 4. The giver of the consent was aware of the nature of the police conduct to which she was being asked to consent 5. The giver of the consent was aware of her right to refuse to permit the police to engage in the conduct requested 6. The giver of the consent was aware of the potential consequences of giving the consent Note on R. v. Lewis (1998), 122 CCC (3d) 481 Doherty JA The police are not under a "duty" to advise a person of the right to refuse to consent to a search in the sense that the failure to do so will amount to a violation of s. 8 Unlike s. 10(b) of the Charter, s. 8 does not contain an informational component The failure to advise a person of the right to refuse to consent to a search may, 38

however, lead to a violation of s. 8 where the police conduct can be justified only on the basis of an informed consent It is well established that a person cannot give an effective consent to a search unless the person is aware of their right to refuse to consent to that search Where the police do not inform a person of the right to refuse to consent to a search, it is certainly open to a trial judge to conclude that the person was unaware of the right to refuse and could not, therefore, give an informed consent

C) Consent incident to duties of police officers if reasonably necessary (ancillary powers doctrine)
Colet v. R., [1981] 1 SCR 2 Facts The accused was charged with five counts including two counts of attempted murder and two counts of intending to cause bodily harm, all of which arose out of his conduct in defence of his property, which the City of Prince Rupert had instructed to be demolished, against what he deemed to be a wrongful intrusion of police officers acting under the purported authority of a warrant to seize firearms A verdict of acquittal was entered at a trial held before a judge and a jury in view of the trial judge's ruling that a warrant to seize did not give police officers a right to search The British Columbia CA allowed the appeal and directed a new trial Issue: Did the power to seize confer ancillary powers to the police to search around the accuseds property? Holding: No; appeal allowed Ritchie J What is involved here is the longstanding right of a citizen of this country to the control and enjoyment of his own property, including the right to determine who shall and who shall not be permitted to invade it There are occasions when the interest of a private individual in the security of his house must yield to the public interest, when the public at large has an interest in the process to be executed Whatever the occasion may be the police are not justified in making an entry unless they have first announced their presence and demonstrated their authority by stating a lawful reason for their entry In this case, although the police officers waved the warrant at the appellant from a distance, it was not a warrant "to search the premises" nor in my view did the authority to seize specified in that warrant carry with it the right to enter and search. Any statutory provision authorizing police officers to invade the property of others without invitation or permission would be an encroachment on the common law rights of the property owner and in case of any ambiguity would be subject to a strict construction in favour of the common law rights of the owner 39

. Any provision authorizing police officers to search and enter private property must be phrased in express terms and the provisions of the Interpretation Act are not to be considered as clothing police officers Extensive number of sections of the Criminal Code expressly include the dual authority "to search" and "to seize" If Parliament intended to include the power "to search" in the provisions of s. 105(1), the failure to do so was a clear case of legislative oversight Lecture Notes Ritchie J required strict statutory construction of the power to search within the power to seize o The principle of legality requires Parliament to speak clearly when justifying the intrusion of individuals liberty and property Parliament reacted to this by adding the words and search to the statutory provisions that dealt with seizure R. v. Deadman supra (1985), 46 CR (3d) 193 Facts The accused was randomly stopped as part of a spot check program, whose principal aim is to detect, deter and reduce impaired driving He was accused with failing, without reasonable excuse to provide a breath sample, but was acquitted On appeal, the acquittal was confirmed as it was held that there was neither statutory nor common law authority to require him to stop his motor vehicle, so he had a reasonable excuse The Ontario CA reversed the acquittal Issue: Did the police officer possess authority, either statutory or at common law, to require the appellant to stop his motor vehicle? Holding: Yes; appeal dismissed Le Dain J The test laid down in R. v. Waterfield, while generally invoked in cases where it is at issues whether a police officer had acted in the execution of her duties, has been recognized as a test for whether the officer had common law authority to do what she did Applying the Waterfield test, the random vehicle stop was a prima facie unlawful interference with liberty since it was not authorized by statute The right to circulate in a motor vehicle on the public highway may be described as a liberty; however, it cannot be regarded as a fundamental liberty like an individual's right of movement, since it is a licensed activity subject to regulation and control for the protection of life and property The random stop falls within the general scope of police duties to prevent crime and to protect life and property by the control of traffic as these are the very objects of the program, 40

a measure intended to improve the deterrence and detection of impaired driving The stop was not an unjustifiable use of police power because it was both necessary to the execution of police duty and reasonable, having regard to the nature of the liberty interfered with and the importance of the public purpose served by the interference Dickson CJ (dissenting) It has always been a fundamental tenet of the rule of law that police, in carrying out their general duties as law enforcement officers have limited powers and are only entitled to interfere with the liberty or property of the citizen to the extent authorized by law The fact that a police officer has a general duty to prevent crime and protect life and property does not mean that he or she can use any or all means for achieving these ends. In the criminal law, the rules and principles relating to arrest establish justifiable limits upon a citizen's liberty Short of arrest, the police have never possessed legal authority at common law to detain any one against his or her will for questioning or to pursue an investigation These random stops are indistinguishable from detention for questioning or investigation whether or not they might be committing a criminal offence and, without validly enacted legislation to support them, are unlawful It would be contrary to the long standing protection accorded individual liberty by the common law and detrimental to the individual's fundamental right to be free from arbitrary interference to conclude that this action of the police was authorized and lawful The fact that driving a motor vehicle is a licensed activity subject to regulation and control in the interests of safety is irrelevant to police power if the conditions for licensing have been met and are adhered to Lecture Notes This case establishes the new police power of investigative detention The Waterfield test is the source of the doctrine of ANCILLARY POLICE POWERS o It came about, however, in trying to determine whether a police officer was acting within her legally prescribed authority The general duties of police officers, were a common law authority that would justify this type of stopping If the powers of the police officer are prima facie unlawful, you can check whether the actions fall within the general powers of the police, and whether they are justifiable o The stops were justifiable given the importance of highway traffic and the fact that the activity was a licensed one A crucial point about this case, was whether the creation of these powers was a matter of the legislature or courts could do it as well The SCC accepted the ancillary powers doctrine as given in Godoy Here, the court recognized a common law power to enter the premises to investigate a disconnected 911 call

41

Other federal statutes


Many other federal statutes contain powers of search without warrant An example is the issuance of writs of assistance, conferring the holder a walking search warrant and not subject to judicial control In Noble writs of assistance were held to be contrary to s.8 of the Charter o The Law Reform Commission of Canada had reached this conclusions earlier and recommended their replacement with the telewarrant procedure o The Code now provides for this under s. 487.1 R. v. Noble (1984), 48 OR (2d) 643 Martin JA An understanding of the nature of writs of assistance is essential to a determination of their constitutional validity There are at the present time in Canada four statutes which provide for the issuance of writs of assistance: the Customs Act, the Excise Act, the Narcotic Control Act, the Food and Drugs Act There are some variations in the four statutes with respect to the powers conferred on the holder of a writ of assistance The writ is not a grant of authority to conduct a particular search of particular premises, but empowers the writ-holder without limitation as to time or place to exercise the search and seizure powers conferred by the statute under which the writ is issued At common law, general warrants, that is, warrants which did not specify the person to be arrested, the premises to be searched and the things to be seized, are illegal and void It is not fit, that the receiving or judging of the information should be left to the discretion of the officer The magistrate ought to judge; and should give certain directions to the officer Writs of assistance, because of the generality of the power conferred by them, are sometimes classified as a statutory species of general warrant The Law Reform Commission of Canada, however, expresses the view that a writ of assistance is more properly characterized as a certificate of the legal competence of its bearer to exercise a statutory power of search and seizure without a The commission also takes the view that since the judge of the Federal Court has no discretion to refuse to grant an application made in proper form for the issuance of the writ, in essence the writ is a ministerial or executive authority or commission to the holder to exercise without warrant statutory powers of search The fact that the writ is granted by the Federal Court inevitably tends to give the writ an aura of judicial authorization which is misleading, since the court has no discretion with respect to its granting. 42

Care must be taken to insure that the writs do not say anything other than that which Parliament has directed and does not contain anything that is calculated to mislead the reader into thinking that the writ is anything other than that which the terms of the legislation require The writ of assistance is a document issued out of the Federal Court which identifies the holder as a person entitled to exercise without a warrant the statutory powers of search and seizure under the relevant statute It is like an identification card signifying that the holder is entitled to conduct warrantless searches and seizures pursuant to the search and seizure powers conferred by the relevant statute Consequently, searches under a writ of assistance are warrantless searches by designated persons pursuant to statutory powers.

Provincial statutes
There are hundreds of search powers under various provincial statutes Wide powers to stop, inspect and seize are found under provincial Highway Traffic and Liquor acts

Constitutional minimum standards


s. 8 Canadian Charter of Rights and Freedoms Search or Seizure Everyone has the right to be secure against unreasonable search or seizure. The Canadian protection against unreasonable search and seizure is different from the more elaborate version of the Fourth Amendment to the US Constitution, which is discussed in Rao R. v. Rao (1984), 46 OR (2d) 80 Facts At the trial of the accused on a charge of possession of cannabis resin for the purpose of trafficking, the Crown sought to introduce certain narcotics which had been seized from the accused's office The search of the office had been conducted without warrant, relying on the warrantless search provisions of s. 10(1)(a) of the Narcotic Control Act The accused protested against the warrantless search and asked to call a lawyer The trial judge held that the warrantless search was unreasonable and contrary to s.8 of the Charter and excluded the evidence having regard to all the circumstances The accused was acquitted Issue: Did warrantless searches violate s.8 of the Charter? 43

Holding: Yes; appeal dismissed Martin JA In looking at the Fourth Amendment of the US Constitution, the first clause merely provides that a search or seizure must meet the standard of reasonableness, while the second clause merely provides that a warrant must meet the requirements specified in that clause An examination of the decisions of the US Supreme Court reveals two distinct lines of approach to the interpretation of the Fourth Amendment Some consider that the first clause should be read separately from the warrant clause, and that the existence of a warrant is only one factor in determining the reasonableness of a search Others have insisted that the two clauses must be read together so that warrantless searches are per se unreasonable, subject only to a few well-delineated exceptions where it would be impracticable to obtain a warrant There is some indication that the first interpretation is becoming more predominant In Texas v. Brown, the court listed a number of exceptions to the warrant requirement, two of which are of particular interest in the present case The first of those exceptions is the "automobile" exception, where it is not practicable to obtain a warrant because the vehicle can be quickly put out of reach of the jurisdiction of a police officer The second exception is the "exigent circumstances" exception o The Supreme Court of the United States has authorized bodily intrusions to seize evidence without a warrant where the police have reasonable cause to believe that the evidence exists and the delay in obtaining a warrant would almost certainly result in the loss or destruction of the evidence o A number of appellate courts have held that an entry and search of premises to prevent the removal or destruction of illicit drugs falls within the "exigent circumstances" exception Although the language of s. 8 of the Charter unmistakably shows the influence on the draftsman of the first clause of the Fourth Amendment, the second clause of the Fourth Amendment has no counterpart in the Charter o Consequently, the central issue is whether the particular search or seizure meets the constitutional requirement of reasonableness unfettered by any constitutional requirement of a warrant Whether a search was authorized by a warrant may be an important or even critical factor in assessing the reasonableness of a search in a given case, but the omission from s. 8 of a warrant provision signals caution in the extent of the use of the American jurisprudence Although there is no express constitutional warrant requirement under s. 8 of the Charter, it is manifest that the legal systems derived from the common law generally require a warrant to enter and search private premises, as distinct from vehicles and vessels which may rapidly move away 44

Section 10(1)(a) of the Narcotic Control Act authorizes a warrantless search of a "place" other than a dwelling-house by a peace officer who has reasonable grounds for believing that the "place" contains a narcotic o The word "place" includes places of fixed location as well as vehicles The search of an office without a warrant where the obtaining of a warrant is not impracticable, is unreasonable and, to that extent, s. 10(1)(a) is of no force or effect o The search of an office without a warrant in circumstances where it is not practicable to obtain a warrant may be entirely reasonable o A warrantless search of vehicles, vessels or aircraft, which may move quickly away, may be reasonable where there are reasonable grounds for believing that such contains a narcotic Hunter et al. v. Southam Inc., [1984] 2 SCR 145 Facts Pursuant to s. 10(1) of the Combines Investigation Act, Lawson A. W. Hunter, Director of Investigation and Research of the Combines Investigation Branch authorized several Combines Investigation officers to enter and examine documents and other things at Southam premises in Edmonton "and elsewhere in Canada" The authorization was certified by a member of the Restrictive Trade Practices Commission pursuant to s. 10(3) of the Act o The authorization had a great scope The Canadian Charter was proclaimed after the authorization was made but before the actual search had begun Southam Inc. unsuccessfully sought an interim injunction pending trial of the question whether the search was in violation of s. 8 of the Charter The Alberta CA ordered all documents taken from the respondent's premises sealed as an interim measure and proceeded with the appeal Hunter appeals the Court's finding that s. 10(3), and, by implication, s. 10(1) of the Act were inconsistent with the Charter and therefore of no force or effect Issue: Did the legislation violate the protection from unreasonable search and seizure? Holding: Yes; the appeal was dismissed Dickson J The Charter is a purposive document which must be subjected to a purposive analysis Section 8 of the Charter guarantees a broad and general right to be secure from unreasonable searches and seizures which extends at least so far as to protect the right of privacy from unjustified state intrusion Its purpose requires that unjustified searches be prevented It is not enough that a determination be made, after the fact, that the search should not have been conducted; this can only be accomplished by a requirement of prior authorization Accordingly, prior authorization, where FEASIBLE, is a precondition for a valid search and seizure 45

Warrantless searches are PRIMA FACIE UNREASONABLE under s. 8 o The party seeking to justify a warrantless search bears the onus of rebutting the presumption of unreasonableness

The procedures established by s. 10(3), however, are constitutionally defective in two respects First, for the authorization procedure to be meaningful, it is necessary for the person authorizing the search to be able to assess the conflicting interests of the state and the individual in an entirely neutral and impartial manner o He must not be someone charged with investigative or prosecutorial functions under the relevant statutory scheme Second, reasonable and probable grounds, established upon oath, to believe that an offence has been committed and that there is evidence to be found at the place of the search, constitutes the minimum standard consistent with s. 8 of the Charter for authorizing searches and seizures o Subsections 10(1) and 10(3) of the Act do not embody such a requirement and therefore, do not measure up to the standard imposed by s. 8 of the Charter. The Court will not attempt to save the Act by reading into it the appropriate standards for issuing a warrant It should not fall to the courts to fill in the details necessary to render legislative lacunae constitutional Dickson J refrained from carrying out a s.1 analysis, because the parties had not submitted any evidence to support it Lecture notes This was one of the first cases to discuss the restriction of state action in individuals property interests What is being protected from intrusion is not property per se, but the reasonable expectation of privacy A proper warrant, wherever feasible, process is necessary to justify the states intrusion The conditions for a warrant to be constitutionally valid were also laid out in the case o The person granting it must be aware of the competing interests between the individual and the state o The assessment must be done in advance of the search o The search must be authorized by law, the law must be reasonable and must be executed reasonably o It must be given under oath Prior authorization is required, but all this case states about an exception to this, is that it is not necessary when it is not feasible Hunter recognized an exception to the warrant requirement: where it is not feasible to get one Under this case the search of any premise (including a person or vehicle) is whether the warrant was feasible Under Rao, however, the focus is on reasonableness, and a warrant is only a critical factor 46

o This seems to suggest that under the Rao standard courts are less likely to assert a warrant requirement for searches of vehicles A s.8 challenge against a search can be made against a search power (like Rao, Hunter and Noble) or against the way in which the search is exercised, as will be illustrated below R. v. Collins, [1987] 1 SCR 265 Facts The accused had been under surveillance by two members of the RCMP Drug Squad Having arrested her husband who was found in possession of heroin in his, a police officer approached her in a pub, laid hold of her identifying himself as a police officer, grabbed her throat and pulled her to the floor The officer directed her to let go of an object clenched in her hand, which was a balloon containing heroin The search was found to be unlawful and therefore unreasonable and in violation of s. 8 of the Charter but the evidence was nevertheless admitted because the accused failed to satisfy the judge that it should be excluded under s. 24(2) of the Charter The Court of Appeal unanimously dismissed the accused's appeal Issue: Was the search unreasonable? Holding: Yes; the appeal was allowed and a new trial was ordered Lamer J The accused bears the burden of persuading the court on a balance of probabilities that a Charter right has been infringed That appears from the wording of s. 24(1) and (2), and most courts which have considered the issue have come to that conclusion The burden of persuasion is the balance of probabilities The courts have also developed certain presumptions, in particular, the SCC held in Hunter v. Southam Inc. that a warrantless search was prima facie "unreasonable" o This shifts the burden of persuasion from the appellant to the Crown: once the appellant has demonstrated that the search was a warrantless one, the Crown has the burden of showing that the search was, on a balance of probabilities, reasonable A SEARCH WILL BE REASONABLE IF authorized by law, if the law itself is reasonable, and in the manner in which the search was carried out is reasonable As the accused did not challenge the constitutionality of s. 10(1) of the Act, the issues that remain to be decided here are whether the search was unreasonable because the officer did not come within s. 10 of the Act, or whether, while being within s. 10, he carried out the search in a manner that made the search unreasonable The Crown here was not able to prove the search reasonable because it did not establish under s. 10 of the Narcotic Control Act that the officer had reasonable and probable grounds for believing there were narcotics in the place where the person was searched

47

The nature of the belief will also determine whether the manner in which the search was carried out was reasonable o If the officer is lawfully searching a person whom he believes on reasonable grounds to be a "drug handler", then the "throat hold" would not be unreasonable Because the failure to establish the grounds for the search was due to an error by the trial judge, a new trial should be ordered if the evidence would be excluded on the record as it now stands. It is important to note that there is no blanket exception to the warrant requirement for motor vehicle searches, and in fact an assessment of reasonableness of the search may be carried out See R. v. D. (I.D) (1987) and R. v. Grant (1993) R. v. Edwards, [1996] 1 SCR 128 Facts The accused was convicted of possession of drugs for purposes of trafficking He had been suspected of drug dealing out of his car using a cellular phone and of keeping the drugs at his residence or at his girlfriend's apartment The police arrested him on a traffic offence Two officers later called at his girlfriend's apartment and gained her cooperation through a number of statements, some of which were lies and half-truths o Once inside, the accused's girlfriend directed them to the location of a significant cache of drugs. She was arrested a short time later but the charges against her were later dropped o At no time prior to being taken into custody was she advised of her right to refuse entry to the police or of her right to counsel o At the police station, she gave a statement naming the accused as the person who put the drugs in her apartment At trial and on appeal, the accused denied being the owner of the drugs The accused's appeal from conviction was dismissed with a dissenting opinion which found a reasonable expectation of privacy giving rise to the possibility of an infringement of his s. 8 Charter rights against unreasonable search or seizure Issue: Did Edwards hold a reasonable expectation of privacy against the search and seizure of his girlfriends place? Holding: No; the appeal was dismissed Cory J A reasonable expectation of privacy is to be determined on the basis of the totality of the circumstances, but without reference to the conduct of the police during the impugned search The factors to be considered may include o Presence at the time of the search o Possession or control of the property or place searched o Ownership of the property or place o Historical use of the property or item o The ability to regulate access o The existence of a subjective expectation of privacy 48

o The objective reasonableness of the expectation If an accused person establishes a reasonable expectation of privacy, the inquiry must proceed to the second stage, to determine whether the search was conducted in a reasonable manner The intrusion of the privacy rights of a third person may be relevant in the second stage of this analysis o The invasion of third party right, however, is not determinative of the unreasonableness of the search The accused had no privacy interest in the goods seized as he had denied that the drugs were his Furthermore, taking all the circumstances of the case into account, he demonstrated no expectation of privacy in his girlfriend's apartment, which was the only other relevant privacy interest o His girlfriend described him as "just a visitor" o He only had a few personal belongings in the apartment and did not contribute to rent o While he had keys to the apartment, he lacked the authority to regulate access to the premises The police conduct did not affect a personal right of the accused A claim for relief under s. 24(2) of the Charter can only be made by the person whose Charter rights have been infringed o Like all Charter rights, s. 8 is a personal right. It protects people and not places The reasonable expectation of privacy concept has worked well in Canada It has proved to be reasonable, flexible and viable, and should not be abandoned in favour of the discredited rule of automatic standing La Forest J While concurring with the majority in the result, disagreement with their reasons was expressed on the ground that their effect was to diminish drastically the public's interest in being left alone, guaranteed by s. 8 of the Charter, in a manner inconsistent with previous statements of this Court As I see it, the protection accorded by s. 8 is not in its terms limited to searches of premises over which an accused has a personal right to privacy in the sense of some direct control or property Rather the provision is intended to afford protection to all of us to be secure against intrusion by the state or its agents by unreasonable searches or seizures, and is not solely for the protection of criminals It is important for everyone, not only an accused, that the police or other state agents do not break into private premises without warrant. R. v. Tessling, [2004] 3 SCR 432 49

Facts The RCMP, relying on confidential information from two sources, was investigating whether the accused was involved in a marijuana growing operation The RCMP used an airplane equipped with a Forward Looking Infra-Red (FLIR) camera to overfly properties owned by the accused without first obtaining judicial warrant. FLIR technology records images of thermal energy or heat radiating from a building o The police had contacted hydro to determine whether there was an unusual use of electricity in the property, but that wasnt the case The RCMP were able to obtain a search warrant for the accuseds home based on the results of the FLIR image coupled with information supplied by two informants that pointed to the fact that he accused was growing and trafficking marijuana In the house, the RCMP found a large quantity of marijuana and several guns. The accused was charged with a variety of drug and weapons offences At trial, he unsuccessfully argued that the FLIR overflight was a violation of his right to be free from unreasonable search and seizure guaranteed by s. 8 of the Charter, and was convicted The Court of Appeal set aside the convictions Issue: Does the use of Forward Looking Infra-Red violate the right to unreasonable search and seizure? Holding: No; the appeal was allowed Binnie J The freedom from unreasonable search and seizure is fundamental to the relationship between the state and the citizen Few things are as important to our way of life as the amount of power allowed the police to invade our home, privacy and even bodily integrity S. 8 of the Charter creates for everyone certain areas of personal autonomy where the state, including the police, cannot trespass At the same time, social and economic life creates competing demands; the community wants privacy but it also insists on protection Thus s. 8 of the Charter accepts the validity of reasonable searches and seizures. The difficult issue is where the reasonableness line should be drawn The distinction between informational and territorial privacy is of assistance in the current factual situation o Whereas the CA treated the FLIR imaging as equivalent to a search of the home, it is more accurately characterized as an external surveillance o FLIR is not equivalent to entry but provides information o Everything shown in the FLIR photograph exists on the external surfaces of the building and, in that sense, FLIR records only information exposed to the public o FLIR heat profile did not expose any intimate details of the accuseds lifestyle or part of his core biographical data When one considers the totality of the circumstances, the use of FLIR technology did not intrude on the reasonable sphere of privacy of the accused 50

Patterns of heat distribution on the external surfaces of a house are not a type of information in which, objectively speaking, the accused had a reasonable expectation of privacy Comments FLIR technology did not reveal specific information about the lifestyle and personal choices of the individual, so there is no expectation of privacy In Tessling the SCC distanced itself from the US Supreme Court in Kyllo v. United States (2001), where FLIR imaging was unconstitutional The SCC does express two caveats: o FLIR information alone is insufficient to obtain a search warrant o If FLIR technology gets better, the constitutional issues will be reconsidered R. v. Kang-Brown, 2008 SCC 18 Facts An RCMP officer involved in a special operation designed to detect drug couriers at bus stations spotted an individual who seemed suspicious The officer eventually approached the accused, identified himself and told him that he was not in any trouble and was free to go at any time He asked him if he was carrying narcotics and the accused said no The officer then asked to look in the accuseds bag, but when went to touch the bag the accused pulled it away, looking nervous At that point, the officer signaled another officer with a sniffer dog to approach and the dog sat down, indicating the presence of drugs in the bag The accused was arrested for possession of and/or trafficking in drugs, he was searched and drugs were found on his person and in his bag The trial judge found that the accused was neither arbitrarily detained nor unlawfully searched and entered a conviction o She held that the odours from the bag, which emanated freely in a public transportation facility, did not constitute information in which the accused had a reasonable expectation of privacy and that s. 8 of the Charter The Court of Appeal upheld the conviction Issue: Was the sniffer dogs search lawful? Holding: No; the appeal was allowed Lebel J (majority) The use of sniffer dogs constitutes a search within the meaning of s. 8 of the Charter, and absent justified authority for such a search in a statute or at common law, the sniffer-dog search breached s. 8 It is undisputed that the search was not specifically authorized by statute In determining whether the police were authorized at common law to conduct the search in fulfilment of their general duty to investigate crime, the threshold for the exercise of police powers should not be lowered to one of reasonable suspicion since, to do so, would impair the important safeguards found in s. 8 against unjustified state intrusion 51

The existing and well-established STANDARD OF REASONABLE AND PROBABLE GROUNDS should be applied In this case, the search did not meet this standard

Any perceived gap in the present state of the law on police investigative powers arising from the use of sniffer dogs is a matter better left for Parliament When rights and interests as fundamental as personal privacy and autonomy are at stake, the constitutional role of the Court suggests that the creation of a new and more intrusive power of search and seizure should be left to Parliament to set up and justify under a proper statutory framework Binnie J A sniff amounts to a s. 8 search because of the significance and quality of the information obtained about concealed contents, whether such contents are in a suspects belongings or carried on his or her person Because of the minimal intrusion, contraband-specific nature and pinpoint accuracy of a sniff, a proper balance between an individuals s. 8 rights and the reasonable demands of law enforcement would be struck by permitting such sniff searches on a REASONABLE SUSPICION standard without requiring prior judicial authorization Sniffer dogs have been in common use by police forces in Canada for the last 30 years or more. If the police have lawful authority to use sniffer dogs only when they already have reasonable grounds to believe contraband is present, sniffer dogs would be superfluous and unnecessary The issue raised by this appeal is not the existence of a police power to investigate crime using sniffer dogs, but the extent to which the use of such animals is permitted by s. 8 of the Charter It is emphatically the duty of the courts, not Parliament, to resolve the issue of Charter compliance The reasonable suspicion standard is not a new juridical standard called into existence for the purposes of this case Suspicion is an expectation that the targeted individual is possibly engaged in some criminal activity A reasonable suspicion means something more than a mere suspicion and something less than a belief based upon reasonable and probable grounds Because sniffer-dog searches are conducted without prior judicial authorization, the afterthe-fact judicial scrutiny of the grounds for the alleged reasonable suspicion must be rigorous It is common ground that what occurred at the bus station was a warrantless search, and therefore presumptively unreasonable However, had the dog-sniff search been based on reasonable suspicion, the dogs positive alert would have given the police the grounds to proceed on the spot with a warrantless search 52

The sniff in this case was an unreasonable search since the RCMP officer did not have grounds for reasonable suspicion at the time the dog was called Given the dog alerts to the odour of narcotics, not to their actual presence, the arrest of the accused in this case was premature The police should first have confirmed the presence of narcotics by a hand search of the bag If reasonable cause had existed, and given the positive alert and the dogs history of accuracy, the RCMP would have been entitled to perform such a verification search on the spot and without prior judicial authorization Deschamps J (dissenting) The use of a sniffer dog to check the accuseds bag in a public bus terminal on the basis of a reasonable suspicion that evidence of an offence would be discovered was proper and did not constitute an unreasonable search or seizure The search in this case was justified on the basis of the reasonable suspicion standard This standard can be applicable only where there are circumstances that serve as safeguards against unreasonable intrusions on privacy and ensure a balance that affords proper protection A reasonable suspicion standard may be sufficient where the investigative technique is relatively non-intrusive and the expectation of privacy is not high To determine whether the reasonable suspicion standard is met in a given case, the totality of the circumstances must be considered The sniffer dogs intrusion on the accuseds right to informational privacy suggests that the accused had an objectively reasonable expectation of privacy, yet nevertheless, the accuseds objectively reasonable expectation of privacy in this case was not high The search in the present case was conducted in a public place and it was only minimally intrusive The police did not use sniffer dogs randomly at the bus terminal where the appellant was searched They had obtained the permission of the terminals management to do so and it is after having observed the appellant in this public place that the police determined that he was a person of interest Bastarache J (dissenting) Although the accused did have a reasonable expectation of privacy in his luggage, it was significantly reduced owing to the location at which the search occurred In a bus depot, a passenger is voluntarily using the terminal to access a public mode of transportation, and he or she is aware that the state has an interest in ensuring that that transportation system is both secure and not being used to further criminal activity The requisite balancing of individual rights with the state interest in preventing and investigating 53

crime supports a finding that a sniff search of luggage using a police dog will not be in violation of s. 8 of the Charter where the police are acting on a reasonable suspicion about the committal of a crime The reduced expectation of privacy at public terminals, the minimal intrusion caused by the search itself, and the effectiveness of sniffer-dog searches all support a standard of reasonable suspicion. Lecture notes How does Kang-Brown fair in light of Tessling o The dogs identify particularly whether you have drugs or not, which could be part of your lifestyle, while FLIR technology only indicates heat consumption patterns, which in itself is not an offence Binnie J states that if you required reasonable and probable grounds to carry out the sniffer search, the sniffer dogs become redundant o His standard instead in the reasonable suspicion, which in this case was not met given the attitude of the defendant Lebel J would leave the creation of a new police power to Parliament o Binnie Js answer to this is that since police powers have been extended under other circumstances, not doing it do here could be inconsistent and courts would become a space where police powers are extended depending on the feeling of the court R. v. A.M., 2008 SCC 19 Facts The police accepted a long-standing invitation by the principal of a high school to bring sniffer dogs into the school to search for drugs o The police had no knowledge that drugs were present in the school and would not have been able to obtain a warrant to search the school The sniffer dog reacted to one of the unattended backpacks lined up against a wall o Without obtaining a warrant, the police opened the backpack and found illicit drugs The student who owned the backpack was charged with possession of cannabis marihuana and psilocybin for the purpose of trafficking At trial, the accused brought an application for exclusion of the evidence, arguing that his rights under s. 8 of the Charter had been violated The trial judge allowed the application, finding two unreasonable searches: the search conducted with the sniffer dog and the search of the backpack, and he ordered an acquittal The CA upheld the acquittal Issue: Was the sniffer dogs search lawful? Holding: No; appeal dismissed Lebel J (majority) Since there was no authority in the statutes or at common law for the sniffer-dog search in this case, the search violated s. 8 of the Charter Students are entitled to privacy in a school environment For the reasons stated in R. v. Kang- Brown, courts should not attempt to craft a legal framework of general application for the use of sniffer dogs in schools 54

Binnie J The dog sniff amounts to a search within s. 8 of the Charter The information provided when the dog is trained to alert to the presence of controlled drugs permits inferences about the precise contents of the source that are of interest to the police The subject matter of the sniff is not public air space, it is the concealed contents of a backpack, which is a personal Teenagers may have little expectation of privacy from the searching eyes and fingers of their parents, but they expect the contents of their backpacks not to be open to the random and speculative scrutiny of the police Although a warrantless sniffer-dog search is available where reasonable suspicion is demonstrated, the sniffer-dog search of the students belongings in this case violated their Charter rights under s. 8 The dog-sniff search was unreasonably undertaken because there was no proper justification While the sniffer-dog search may have been seen by the police as an efficient use of their resources, and by the principal of the school as an efficient way to advance a zero-tolerance policy, these objectives were achieved at the expense of the privacy interest (and constitutional rights) of every student in the school The Charter weighs other values, including privacy, against an appetite for police efficiency Where there are grounds of reasonable suspicion, the police should not have to take their suspicions to a judicial official for prior authorization to use the dogs in an area where the police are already lawfully present All searches do not have the same invasive and disruptive quality and prior judicial authorization is not a universal condition precedent to any and all police actions characterized as searches given that the touchstone of s. 8 is reasonableness Account must be taken in s. 8 matters of all the relevant circumstances including the minimal intrusion, contraband-specific nature and high accuracy rate of a fly-by sniff The warrantless search is, of course, presumptively unreasonable If the sniff is conducted on the basis of reasonable suspicion and discloses the presence of illegal drugs, the police may confirm the accuracy of that information with a physical search, again without prior judicial authorization The importance of proper tests and records of particular dogs will be an important element in establishing the reasonableness of a particular sniffer-dog search An important concern for the court is therefore the number of false positives Moreover, the sniff does not disclose the presence of drugs, but it discloses the presence of an odour that indicates either the drugs are present or may have been present In sniffer-dog situations, the police are generally required to take quick action guided by on-the55

spot observations In circumstances where this generally occurs, it is not feasible to subject the sniffer dogs sniff to prior judicial authorization Both the subject and his suspicious belongings would be long gone before the paperwork could be done The youth court judge here noted that the evidence of the drugs existed independently of the Charter violation and that its admission, being non-conscriptive, would not affect trial fairness However, given the fact that the speculative sweep in this case appears to be the standard practice of municipal police forces in Ontario, the youth court judge concluded that the Charter must not be seen as something to be swept away in the interests of expediency The court will not interfere with balance of competing values struck by the youth court judge or his exclusion of the evidence R. v. Wong, [1990] 3 SCR 36 Facts At issue was the admissibility of police video surveillance evidence of a large-scale illegal gambling operation in a hotel The Ontario CA held that the protection under s.8 of the Charter was not available as there was no reasonable expectation of privacy, given that 30 to 35 people had been invited to gamble illegally in the room The SCC held that this reasoning could not be reconciled with Duarte Issue: Is the reasonable expectation of privacy diminished by the fact that someone is committing a crime? Holding: No La Forest J The question should NOT be framed in terms of whether people engaging in illegal activities in a hotel room have a reasonable expectation of privacy, but whether in our society, people who retire to a hotel room and close the door behind them have a reasonable expectation of privacy Otherwise, no only those engaging in criminal activities will risk warrantless surveillances, but also all members of society renting rooms in a hotel S. 8 of the Charter affords us a protection against warrantless video surveillances in a hotel, just as these would be unlawful in our own homes The very reason we rent such rooms is to obtain a private enclave [La Forest J also held that it was not appropriate for courts to authorize video surveillance] Part IV.1 of the Code is designed to set strict limits on the ability of the agents of the state to intercept private oral communications Courts would be forgetting their role as guardians of our fundamental liberties if they were to usurp the role of Parliament and purport to give their sanction to video surveillance by adapting for that purpose a code of procedure dealing with an altogether different 56

surveillance technology It is for Parliament, and Parliament alone, to set out the conditions under which law enforcement agencies may employ video surveillance technology in their fight against crime Moreover, the same holds true for any other technology which the progress of science places at the disposal of the state in the years to come Lecture by Sarah Henningsson - Provincial prosecutor Fact pattern Flier goes out to Montreal police which has the picture of a black individual in his midtwenties who is wanted for home invasion, and is likely to be seen around the Lachine area An officer who is on patrol around Lachine sees someone who fits the description of the person in the flier o In his mind he is the person He stops the person and asks him to identify himself Verifications are then made by the officer and he finds out that a warrant is out for his arrest, but as he tries to arrest him he runs away Later on another officer sees a person who fits the same physical description riding a bike o He asks him his name and he says the same last name as the other person o He then grabs him by the arm and tells him that he is under arrest o As soon as he is arrested he says that they have the wrong person, and that the person they are looking for is his brother o He does not have any identification, however He arrests him anyway because he feels that he has reasonable and probable grounds and in his mind he is the person he was looking for o He searches him as incidental to arrest and finds crack cocaine and $ 280 cash and a cell phone o He is also arrested for possession for the purpose of trafficking At the police station they find that he is in fact the brother of the person they were originally looking for For the arrest to be lawful the officer had to believe that this person had committed or was about to commit a crime There is also an objective component based on information that would make the arrest lawful on reasonable and probable grounds The prosecutor argued that there were grounds for arrest and that the search should not be excluded The judge found that the arrest was illegal because although there were subjective grounds for the arrest, they were not objectively justifiable o The police officer should have gone back to his car and made verification of the information o Is the judge, however, imposing a standard of certainty for arrest, which in turn is contrary to Storrey? 57

Even if the search was illegal, the evidence should not be excluded according to 24(2) The argument here would be that the rocks would have been found anyway

Arrest
The arrest is a critical moment in the balance of power between the citizen and the State Across the spectrum of State intrusion on the individuals liberty and privacy, arrest is at the end as the most intrusive o For example, search incident to arrest allows more search powers than any other event in the common law o Police can use as much force as necessary in effecting an arrest As per R. v. Whitfield, one of the earliest sources of the description of arrest Arrest is the actual seizure or touching of a persons body or pronouncing the words that you are under arrest An arrest often involves a prolonged interference on someones freedom of movement and an intrusion into a persons privacy The powers of arrest are guided by a complex statutory regime Generally speaking, there is NOT an obligation to obtain arrest warrants, as is the case of searches

Citizens Power of Arrest


The power of citizen arrest is expressed in the Code and carefully circumscribed in s.494 s. 494 Criminal Code Arrest without warrant by any person (1) Any one may arrest without warrant (a) a person whom he finds committing an indictable offence; or (b) a person who, on reasonable grounds, he believes (i) has committed a criminal offence, and (ii) is escaping from and freshly pursued by persons who have lawful authority to arrest that person. Arrest by owner, etc., of property (2) Any one who is (a) the owner or a person in lawful possession of property, or (b) a person authorized by the owner or by a person in lawful possession of property, 58

may arrest without warrant a person whom he finds committing a criminal offence on or in relation to that property. Delivery to peace officer (3) Any one other than a peace officer who arrests a person without warrant shall forthwith deliver the person to a peace officer. Crown election offences are treated as indictable for the purpose of the law of arrest by virtue of interpretation Paragraph 2 delineates the special power of arrest of property owners and possessors

Power of Peace Officers


Peace officers, as described in s.2 of the Code, have more extensive powers s. 31 Criminal Code Arrest for breach of peace (1) Every peace officer who witnesses a breach of the peace and every one who lawfully assists the peace officer is justified in arresting any person whom he finds committing the breach of the peace or who, on reasonable grounds, he believes is about to join in or renew the breach of the peace. Giving person in charge (2) Every peace officer is justified in receiving into custody any person who is given into his charge as having been a party to a breach of the peace by one who has, or who on reasonable grounds the peace officer believes has, witnessed the breach of the peace. Notice that there is no restriction to an indictable offence here s. 495 Criminal Code Arrest without warrant by peace officer (1) A peace officer may arrest without warrant (a) a person who has committed an indictable offence or who, on reasonable grounds, he believes has committed or is about to commit an indictable offence; (b) a person whom he finds committing a criminal offence; or (c) a person in respect of whom he has reasonable grounds to believe that a warrant of arrest or committal, in any form set out in Part XXVIII in relation thereto, is in force within the territorial jurisdiction in which the person is found. Limitation (2) A peace officer shall not arrest a person without warrant for 59

(a) an indictable offence mentioned in section 553, (b) an offence for which the person may be prosecuted by indictment or for which he is punishable on summary conviction, or (c) an offence punishable on summary conviction, in any case where (d) he believes on reasonable grounds that the public interest, having regard to all the circumstances including the need to (i) establish the identity of the person, (ii) secure or preserve evidence of or relating to the offence, or (iii) prevent the continuation or repetition of the offence or the commission of another offence, may be satisfied without so arresting the person, and (e) he has no reasonable grounds to believe that, if he does not so arrest the person, the person will fail to attend court in order to be dealt with according to law. Consequences of arrest without warrant (3) Notwithstanding subsection (2), a peace officer acting under subsection (1) is deemed to be acting lawfully and in the execution of his duty for the purposes of (a) any proceedings under this or any other Act of Parliament; and (b) any other proceedings, unless in any such proceedings it is alleged and established by the person making the allegation that the peace officer did not comply with the requirements of subsection (2). The limitation set out in subsection 2 (mainly to summary convictions) is contingent on the circumstances described in subsection 2(e) What is the appropriate standard for a lawful arrest? R. v. Storrey, [1990] 1 SCR 241 Issue: Was the arrest unlawful and arbitrary? Holding: No; appeal dismissed Cory J Section 495(1) makes it clear that the police were required to have reasonable and probable grounds that the appellant had committed the offence of aggravated assault In the case of an arrest made without a warrant, it is even more important for the police to demonstrate that they have those same reasonable and probable grounds upon which they base the arrest The importance of this requirement to citizens of a democracy is self-evident, so as to not fall prey to the abuses and excesses of a police state; yet society also needs protection from crime This need requires that there be a reasonable balance between the individual's right to liberty and the need for society to be protected from crime 60

The Criminal Code then requires that an arresting officer must subjectively have reasonable and probable grounds on which to base the arrest, but this is not sufficient It must be objectively established that those reasonable and probable grounds did in fact exist That is to say a reasonable person, standing in the shoes of the police officer, would have believed that reasonable and probable grounds existed to make the arrest On the other hand, the police need not demonstrate anything more than reasonable and probable grounds, specifically they are not required to establish a prima facie case for conviction before making the arrest. Comments This case explains what reasonable grounds is as per s.495(1)(a) The court in Storrey appeared to be unaware that Parliament had deleted the words and probable from what is now s.495 Storrey did also say that a lawful arrest is not arbitrary The peace officers power to arrest anyone found committing a criminal offence is s.495(1)(b) was interpreted in R. v. Biron to mean apparently committing and offence R. v. Biron, [1976] 2 SCR 56 Facts The Montreal police made an authorized raid on a bar in search of illegal firearms and liquor Biron, who had been drinking, was at the bar while the raid was taking place o He refused to co-operate with the police, verbally abusing them and later refusing to his arrest He was charged with creating a disturbance in a public place by shouting and also for resisting a peace officer The accused was convicted of both offences but a trial de novo acquitted him for the offence of "creating a disturbance by shouting" on the ground that there was no evidence he had been shouting as was alleged in the information He was later acquitted by the Quebec CA on the charge of resisting a police officer His claim is that since he was acquitted of the charge of creating disturbance, the officer had no lawful reason to arrest him since he was committing no criminal offence, and as such he could not be guilty from resisting the police officer Issue: What does it entail to be found committing a criminal offence as per s.495(1)(b)? Holding: To be apparently committing an offence; appeal allowed Martland J Paragraph (a) of s. 450(1) permits a peace officer to arrest without a warrant under the situation where an indictable offence has already been committed or is expected to be committed The peace officer is not present at its commission, so she may have to rely upon information received from others The paragraph therefore enables her to act on her belief, if based on reasonable and probable grounds 61

Paragraph (b) applies in relation to ANY criminal offence and it deals with the situation in which the peace officer herself finds an offence being committed Her power to arrest is based upon her own observation Because it is based on her own discovery of an offence actually being committed there is no reason to refer to a belief based upon reasonable and probable grounds The validity of an arrest under this paragraph must be determined in relation to the circumstances which were apparent to the peace officer at the time the arrest was made The words "committing a criminal offence" are not to be construed in the manner indicated in the Pritchard case, whereby it is impossible to say that an offence is committed until the party arrested has been found guilty by the courts Instead, the wording used in paragraph (b) means that the power to arrest without a warrant is given where the peace officer herself finds a situation in which a person is APPARENTLY committing an offence In the present case Constable Maisonneuve observed an apparent offence being committed by Biron Laskin CJ (dissesnting) It is astonishing that a provision concerned with a constable's criminal or other responsibility, and which immunizes her in specified circumstances in respect of an arrest that she has made, should become the vehicle for providing a basis upon which an accused may herself be convicted of resisting the arrest [This is made in relation to s.25 of the Code which dictates when a police officer is justified in doing what she is authorized to do] If the word "apparently" is to be read into s. 450(1)(b) [now s.495(1)(b)], logical consistency demand that the word be read into s. 449(1)(a) [s.494(1)(a)] which empowers any person to arrest without warrant a person whom she "finds committing" an indictable offence On grounds of context when s. 449(1)(a) is read with s. 449(1)(b), the former could not possibly embrace arrest without warrant on apparency or on reasonable and probable grounds While a constable's burden is a heavy when she has to make an on-the-spot decision as to an arrest, we cannot go on a guessing expedition out of regret for an innocent mistake or a wrongheaded assessment The social and legal, and political, principle upon which our criminal law is based, namely, the right of an individual to be left alone, to be free of private or public restraint, save as the law provides otherwise The position as it relates to resistance to unlawful arrest was established at common law as early as 1709 Our law has not, as I understand it, deprived the citizen of her right to resist unlawful 62

arrest Her resistance may be at her own risk if the arrest proves to be lawful, but too must the police officer accept the risk of having effected a lawful arrest Lecture Notes Requiring police officers to have reasonable and probable grounds that the person committed the crime would entail asking them to be judges and determined whether the person has committed the crime or not Laskin CJ (dissenting) feels that loosening the standard for citizens powers of arrest could be problematic o It can also be said, that the bigger the scope if for s.495(1), the smaller it will be for s.495(2) o Loosening the standard to apparently committing an offence, would also affect all other sections which are based on this standard In Roberge v. R .(1983), Lamer J for the SCC interpreted Biron to mean that apparently committing was the same as having reasonable and probable grounds for believing an offence has been committed Because of this, a requirement of reasonable and probable grounds is now read into all forms of arrest powers

Constitutional Minimum Standards (Charter ss. 9 and 10(a))


There are also constitutional dimensions to the law of arrest s. 9 Canadian Charter of Rights and Freedoms Detention or imprisonment Everyone has the right not to be arbitrarily detained or imprisoned. s. 10 Canadian Charter of Rights and Freedoms Arrest or detention Everyone has the right on arrest or detention a) to be informed promptly of the reasons therefor; b) to retain and instruct counsel without delay and to be informed of that right; and c) to have the validity of the detention determined by way of habeas corpus and to be released if the detention is not lawful. The SCC has not given a clear pronouncement of what an arbitrary detention is There is no equivalent of Hunter that establishes what an arbitrary detention entails The Ontario CA held in Duguay that an unlawful arrest is not necessarily arbitrary under s. 9 of the Charter

63

R. v. Duguay (1985), 45 CR (3d) 140 Facts Following a break and enter at a residence, police were contacted and two experienced detectives attended at the victim's home The evidence indicated that prior to the victim leaving his home that night, he had noticed three young men drinking beer in the backyard of a neighbour's home The owner of the neighbouring house, after hearing a description of the three young men, identified one of them and made arrangements for the three to attend back at the premises When the three men arrived, they were arrested and put in a police cruiser As a result of questioning by one of the detectives, the police were directed to some of the stolen goods and the three accused all made inculpatory statements The trial judge found that the detectives could not honestly believe that they had reasonable and probable grounds to arrest the accused, and as such they had been subjected to arbitrary detention in violation of s. 9 of the Charter and he excluded all the evidence obtained following the arrest pursuant to s. 24(2) of the Charter The accused were acquitted Issue: Is an unlawful arrest necessarily arbitrary and contrary to s.9 of the Charter? Holding: No; appeal dismissed MacKinnon ACJO It cannot be that every unlawful arrest necessarily falls within the words "arbitrarily detained" The person making the arrest may honestly, though mistakenly, believe that reasonable and probable grounds for the arrest exist and there may be some basis for that belief; in those circumstances the arrest, though subsequently found to be unlawful, could not be said to be capricious or arbitrary On the other hand, the entire absence of reasonable and probable grounds for the arrest could support an inference that no reasonable person could have genuinely believed that such grounds existed In such cases, the conclusion would be that the person arrested was arbitrarily detained Between these two ends of the spectrum, the issue of whether an accused was arbitrarily detained will depend on two considerations 1. The particular facts of the case 2. The view taken by the court with respect to the extent of the departure from the standard of reasonable and probable grounds and the honesty of the belief and basis for the belief in the existence of reasonable and probable grounds on the part of the person making the arrest On the facts of the present case, the arrest or detention was arbitrary, being for quite an improper purpose They had neither grounds nor an honest belief that they had the necessary grounds 64

Zuber JA The detention that follows an arrest based on something less than reasonable and probable grounds is not necessarily arbitrary The arrest in this case was neither capricious nor random I have very serious doubts as to these two conclusions reached by the trial judge but since they are bound up with issues of fact and since the appeal by the Crown is dependent upon an issue of law alone, I am content to accept his conclusions on these two issues and rest my judgment entirely on the third and last issue, i.e., the propriety of excluding the evidence Lecture Notes The case seems to constitutionalize the Storrey standard of reasonable grounds for detention, but with some room for departure The court doesnt state whether this is the standard to be used in all types of detention In contrast to the position advanced in Duguay, the Saskatchewan Court held that an unlawful detention is necessarily arbitrary and contrary to s.9 of the Charter; see R. v. Iron (1987)

Reasons for Arrest


Common law requirements respecting the duty to provide the reasons for an arrest were laid down by the House of Lords in Christie v. Leachinsky (1947) The requirement does not means that technical an precise language must be used, but the matters is a matter of substance This pronouncement was followed by Canadian courts until a majority in the SCC held that the duty was exhaustively codified in s.29(2) of the Criminal Code; see Gamracy v. R. (1974) Section 10(a) of the Charter now establishes the right of everyone upon arrest or detention to be informed promptly of the reasons therefor R. v. Evans, [1991] 1 SCR 869 Facts The accused, a mentally challenged youth, was convicted of first degree murder in the brutal killings of two women Initially, the police thought his brother had committed the murders and arrested the appellant on a marijuana charge in the hope that he would be able to provide evidence against his brother During the course of the interrogation that followed, Evans became the prime suspect in the two murders o The police did not formally advise the accused that he was then being detained for murder o The police investigation was aggressive and marked by their lying about finding 65

the appellant's fingerprint at one of the murder scenes o Eventually incriminating statements were obtained from the appellant, and these statements formed virtually the entire basis of his conviction for the two murders An appeal to the Court of Appeal was dismissed The accused continued to appeal that his rights under ss. 7, 10(a) and 10(b) of the Charter were violated so that the resultant confessions should have been excluded pursuant to s. 24(2) of the Charter Issue: Was the accused properly informed of the reasons for his detention? Holding: Yes; the appeal was nonetheless allowed for other reasons McLachlin J The right to be promptly advised of the reason for one's detention embodied in s. 10(a) of the Charter is founded on the notion that one is not obliged to submit to an arrest if one does not know the reasons for it A second aspect of the right lies in its role as an adjunct to the right to counsel conferred by s. 10(b) of the Charter When considering whether there has been a breach of s. 10(a), it is the substance of what the accused can REASONABLY be supposed to have understood, rather than the formalism of the precise words used, which must govern The question is whether what the accused was told, viewed reasonably in all the circumstances of the case, was sufficient to permit him to make a reasonable decision to decline to submit to arrest, or alternatively, to undermine his right to counsel under s. 10(b) The accuseds response to the officer's statement that, while he had originally been arrested on marijuana charges, things had now taken "quite a change", indicates that the accused was aware that the focus of the questioning had changed and that he was then being questioned with respect to the killings It might, therefore, be argued that he was given the facts relevant to determining whether he should continue to submit to the detention Nor can any failure to comply with s. 10(b) be attributed to failure to advise the accused of the reasons why his detention and questioning was continuing These considerations suggest that the requirements of s. 10(a) were met in the case at bar Sopinka J (dissenting on the issues of s.10(a) but concurring on the result) The right to be informed of the true grounds for the arrest or detention is firmly rooted in the common law which required that the detainee be informed in sufficient detail that he or she "knows in substance the reason why it is claimed that this restraint should be imposed" (Christie v. Leachinsky) The purpose of communicating this information to the accused is to enable the person under arrest or detention to immediately undertake his or her defence, including a decision as to what response, if any, to make to the accusation In this case, the arresting officers were forewarned that they were dealing with a person of 66

subnormal intelligence, so it was incumbent on them to be scrupulous in ensuring that his rights were respected Instead, they concocted a ground for the arrest in order to question him about the involvement of his brother in the murders Having explicitly advised the appellant that he was in jeopardy for trafficking in narcotics, the arresting officers were obliged to disabuse him of this false information before seeking to elicit incriminatory evidence from him The accused, whose mental development was equated to that of a 14-year-old, should not have been required to deduce from the content of questions that the initial explicit reason for his arrest had shifted to a far more serious ground

Entry into Premises


The law is particularly vague as to whether authorities have the power to enter premises to arrest The Ouimet Report identified two distinct types of power The police officer can enter premises without a warrant to prevent the commission of an offence which could cause immediate and serious injury to a person, based on a belief grounded in reasonable and probable grounds The police officer can also enter premises without a warrant, by force if necessary, to effect the arrest of a person who has been found committing a serious crime, and who is being freshly pursued and seeks refuge in such premises Before 1997, there was no obligation for the police to subject their intentions to prior judicial scrutiny before arrest Feeney changed this when the arrest was carried out in the defendants premises R. v. Feeney, [1997] 2 SCR 13 Facts An old man was found beaten to death in his home The police learned that the victims truck had been found abandoned in a ditch near the crime scene A resident of the area had seen the accused walking away from the truck and carrying something in his hand, while another witness told the police that the accused had stolen a vehicle and crashed in the same location The police went to the house where the accused was staying and having received no answer at the door, they entered, roused the accused and took him to the front of the trailer for better lighting o The police arrested him after seeing blood on his shirt Following a caution with respect to the right to counsel but not the right to immediate counsel, the police asked the accused a couple of questions which he answered The accused's shirt was seized and he was taken to the police detachment where, before the accused had consulted with counsel, further statements were taken 67

The accused was convicted of second degree murder and his appeal to the British Columbia CA was unanimously dismissed o The CA was preoccupied that the blood in the shirt would be destroyed if the arrest had not been carried Issue: Was the police allowed to enter the accuseds premises to effect the arrest? Holding: No; appeal allowed Sopinka J Under the pre-Charter common law, a warrantless arrest following a forced entry into private premises is legal if a) The officer has reasonable grounds to believe that the person sought is within the premises b) Proper announcement is made c) The officer believes reasonable grounds for the arrest exist d) Objectively speaking, reasonable and probable grounds for the arrest exist Except in exigent circumstances, police should give notice of presence by knocking or ringing the doorbell, give notice of authority by identifying themselves as law enforcement police officers and give notice of purpose by stating a lawful reason for entry o Before forcing entry, police should, at minimum, request admission and have admission denied Neither the subjective nor objective requirements for arrest were met, rendering the arrest unlawful The arresting officer did not believe he had reasonable grounds to arrest prior to the forcible entry, until after he saw the blood-stained shirt A reasonable person, standing in the shoes of the officer, would not have believed that reasonable and probable grounds to make the arrest existed o Any finding that the subjective test is not met will generally imply that the objective test is not met, unless the officer is to be considered to have an unreasonably high standard The Landry test for warrantless searches, essentially a balancing between aiding the police in their protection of society on the one hand and the privacy interests of individuals in their dwellings on the other, no longer applies It must be adjusted to comport with Charter values which, notwithstanding the high value on the security and privacy of the home at common law, significantly increase the importance of the legal status of the privacy of the home In general, the privacy interest now outweighs the interest of the police and WARRANTLESS ARRESTS IN DWELLING HOUSES ARE PROHIBITED The case of Hunter was used in the analysis Generally a warrant is required to make an arrest in a dwelling house, just like warrantless searches are prima facie unlawful Nonetheless, there are exceptions with respect to the unreasonableness of warrantless searches for things o A warrantless search will respect s. 8 if authorized by law, and both the law and 68

the manner in which the search is conducted are reasonable o In cases of hot pursuit, the privacy interest must give way to the interest of society in ensuring adequate police protection An arrest warrant alone is insufficient protection of the suspect's privacy rights Privacy rights under the Charter demand that the police, in general, obtain prior judicial authorization of entry into the dwelling house in order to arrest the person If the Code currently fails to provide specifically for a warrant containing such prior authorization, such a provision should be read in While the absence of such a provision could have a profound influence on the common law power of arrest, its absence cannot defeat a constitutional right of the individual Once a procedure to obtain such prior authorization is created, the concern that suspects may find permanent sanctuary in a dwelling house disappears. Warrantless arrests in dwelling houses are in general prohibited Requiring a warrant prior to arrest avoids the ex post facto analysis of the reasonableness of an intrusion and invasive arrests without a basis of reasonable and probable grounds are prevented, rather than remedied after the fact To summarize, the following requirements must be met before an arrest for an indictable offence in a private dwelling is legal 1. A warrant must be obtained on the basis of reasonable and probable grounds to arrest and to believe the person sought is within the premises in question 2. Proper announcement must be made before entering 3. An exception to this rule occurs where there is a case of hot pursuit o Whether or not there is an exception for exigent circumstances generally has not been fully addressed by this Court. The arrest was unlawful both because the requirements for a warrantless arrest under s. 495 of the Code were not met, and, in any event, the police cannot make warrantless arrests in private dwellings unless exceptional circumstances exist (which were not present in this case) o Consequently, the entry into the trailer and the search and seizure of the accused's clothing violated s. 8 of the Charter LHeureux-Dub J (dissenting on this issues but concurring on the result) There were both subjective and objective grounds for the arrest The sole remaining factor to consider is whether a proper announcement was made before the police entered the premises o In some cases it would be contrary to common sense to announce the purpose of entry once it was clear that the person inside was refusing or unable to answer the request to enter o We are clearly faced with such a case here, since before entering, the officers were informed that the appellant was, in all likelihood, sound asleep It is clear that the power of arrest is a crucial part of law enforcement, and for that reason, it is 69

unrealistic to suggest that the police can never enter private premises without a warrant for the purposes of arrest Neither can I accept that it is only in circumstances of hot pursuit that the police are permitted to enter a dwelling house without a warrant for the purpose of arrest Preventing the removal or destruction of evidence is a legitimate law enforcement concern which warrants setting aside the strict rules concerning the sanctity of the home For all of these reasons, exigent circumstances were indeed present in the case at bar Where these circumstances exist, the common law authorizing entries onto private premises constitutes a "reasonable" entry for the purposes of s. 8 of the Charter Lecture Notes Sopinka J felt that the pre-Charter rule was not applicable because of the privacy interests that were now protected by the Charter On 1997, Parliament enacted new entry powers (see ss.529-529.3) including an exigent circumstances exception for warrantless entry (s. 529.3) s. 529 Criminal Code Including authorization to enter in warrant of arrest (1) A warrant to arrest or apprehend a person issued by a judge or justice under this or any other Act of Parliament may authorize a peace officer, subject to subsection (2), to enter a dwellinghouse described in the warrant for the purpose of arresting or apprehending the person if the judge or justice is satisfied by information on oath in writing that there are reasonable grounds to believe that the person is or will be present in the dwelling-house. Execution (2) An authorization to enter a dwelling-house granted under subsection (1) is subject to the condition that the peace officer may not enter the dwelling-house unless the peace officer has, immediately before entering the dwelling-house, reasonable grounds to believe that the person to be arrested or apprehended is present in the dwelling-house. s. 529.1 Criminal Code Warrant to enter dwelling-house A judge or justice may issue a warrant in Form 7.1 authorizing a peace officer to enter a dwellinghouse described in the warrant for the purpose of arresting or apprehending a person identified or identifiable by the warrant if the judge or justice is satisfied by information on oath that there are reasonable grounds to believe that the person is or will be present in the dwelling-house and that (a) a warrant referred to in this or any other Act of Parliament to arrest or apprehend the person is in force anywhere in Canada; (b) grounds exist to arrest the person without warrant under paragraph 495(1)(a) or (b) or section 672.91; or (c) grounds exist to arrest or apprehend without warrant the person under an Act of 70

Parliament, other than this Act. s. 529.2 Criminal Code Reasonable terms and conditions Subject to section 529.4, the judge or justice shall include in a warrant referred to in section 529 or 529.1 any terms and conditions that the judge or justice considers advisable to ensure that the entry into the dwelling-house is reasonable in the circumstances. s. 529.3 Criminal Code Authority to enter dwelling without a warrant (1) Without limiting or restricting any power a peace officer may have to enter a dwelling-house under this or any other Act or law, the peace officer may enter the dwelling-house for the purpose of arresting or apprehending a person, without a warrant referred to in section 529 or 529.1 authorizing the entry, if the peace officer has reasonable grounds to believe that the person is present in the dwelling-house, and the conditions for obtaining a warrant under section 529.1 exist but by reason of exigent circumstances it would be impracticable to obtain a warrant. Exigent Circumstances (2) For the purposes of subsection (1), exigent circumstances include circumstances in which the peace officer (a) has reasonable grounds to suspect that entry into the dwelling-house is necessary to prevent imminent bodily harm or death to any person; or (b) has reasonable grounds to believe that evidence relating to the commission of an indictable offence is present in the dwelling-house and that entry into the dwelling-house is necessary to prevent the imminent loss or imminent destruction of the evidence. In Godoy, the SCC recognized a common law power to enter premises to investigate a disconnected 911 call R. v. Godoy, [1999] 1 SCR 311 Facts Two police officers received a call from radio dispatch concerning a 911 emergency call originating from the accused's apartment in which the line had been disconnected before the caller spoke Along with two back-up officers they arrived at the accused's apartment and knocked on the door o The accused partially opened the door and when asked if things were all right inside responded that there was no problem o One of the officers asked if they could enter the apartment to investigate but the accused tried to close the door o The officer prevented him from shutting the door and entered the dwelling 71

o As soon as they got inside, he heard a woman crying. They accused's common law wife in their bedroom and the officer observed considerable swelling above her left eye, which she stated was the result of the accused hitting her Based on these observations, the accused was placed under arrest for assaulting his wife The trial judge dismissed the charge, holding that the officers' entry into the accused's apartment was unauthorized and that therefore all subsequent actions of the police, including the arrest of the accused, were illegal The Ontario Court (General Division) allowed the Crown's appeal and ordered a new trial The Court of Appeal upheld that decision Issue: Was a warrantless entry and ensuing arrest following a disconnected 911 call lawful? Holding: Yes; appeal dismissed Lamer CJ Public policy clearly requires that the police ab initio have the authority to investigate 911 calls, but whether they may enter dwelling houses in the course of such an investigation depends on the circumstances of each case The accepted test for evaluating the common law powers and duties of the police was set out in Waterfield If police conduct constitutes a prima facie interference with a person's liberty or property (which is the case of the forcible entry in the case at bar), the court must consider two questions 1. Does the conduct fall within the general scope of any duty imposed by statute or recognized at common law? 2. Does the conduct, albeit within the general scope of such a duty, involve an unjustifiable use of powers associated with the duty? The common law duties of the police (statutorily incorporated in s. 42(3) of the Ontario Police Services Act) include the protection of life The police duty to protect life is engaged whenever it can be inferred that the 911 caller is or may be in some distress, including cases where the call is disconnected before the nature of the emergency can be determined The importance of the police duty to protect life warrants and justifies a forced entry into a dwelling in order to ascertain the health and safety of a 911 caller There was no other reasonable alternative to ensure that the disconnected caller received the necessary assistance in a timely manner While residents have a recognized privacy interest within the sanctity of their home, the public interest in maintaining an effective emergency response system is obvious and significant enough to merit some intrusion on a resident's privacy interest However, the intrusion must be limited to the protection of life and safety; the police do not have further permission to search premises or otherwise intrude on a resident's privacy or property. As for Feeney, this case was concerned solely with when the police can enter a dwelling without a 72

warrant to make an arrest, thus, the reasoning in Feeney does not apply to the case at bar, which is unconcerned with powers of arrest

Meaning of Arrest
The leading case of what constitutes an arrest is now Latimer R. v. Latimer, [1997] 1 SCR 217 Facts The accused was the father of Tracy, a severely disabled child who suffered from extreme cerebral palsy and was quadriplegic o Tracy was in constant pain, and despite the administration of medication, experienced five or six seizures a day Tracy died while in the care of the accused, who advised the RCMP by telephone that she had passed away in her sleep o An autopsy found signs consistent with poisoning, and tests then indicated that Tracy's blood was saturated with carbon monoxide The RCMP began to treat the matter as a homicide investigation and eventually the accused was told that he was being detained for investigation into the death of his daughter o He was informed of his right to retain and instruct counsel without delay, of the availability of Legal Aid duty counsel, and of his right to remain silent o He was then taken to the police station, where he made a full confession The accused was convicted of second-degree murder o His appeal was dismissed by the CA The accused appealed, inter alia, that he had been arbitrarily detained in his farm and that his confession to the police should have been excluded Issues: 1. Was the accused arbitrarily detained contrary to s.9 of the Charter? 2. Did the failure to inform the accused that he had been "arrested" and that he could be charged with murder violate s. 10(a) of the Charter? Holding: 1. No; 2. No; the appeal was allowed for other reasons Lamer CJC 1. The RCMP officers who attended at the Latimer farm put Mr. Latimer under de facto arrest The de facto arrest was entirely lawful because it was based on reasonable and probable grounds that Mr. Latimer had taken his daughter's life A de facto arrest which is lawful cannot be an arbitrary detention for the purposes of s. 9 The appellant's strongest argument is that no arrest occurred because the officers deliberately chose not to arrest Mr. Latimer Notwithstanding what the intention of the officers may have been, their CONDUCT had the effect of putting Mr. Latimer under arrest o In R. v. Whitfield, Judson J. held that an arrest consists either of (i) the actual 73

seizure or touching of a person's body with a view to his detention, OR (ii) the pronouncing of "words of arrest" to a person who submits to the arresting officer With regards to this, what matters is the substance of what the accused can reasonably be supposed to have understood, rather than the formalism of the precise words used On the facts of this case, a de facto arrest occurred through the use of words that conveyed clearly that Latimer was under arrest, the conduct of the officers, and Mr. Latimer's submission to the authority of the officers Mr. Latimer was told that he was being detained, and that he would be taken back to North Battleford to be interviewed The police officers informed him of his right to silence and his right to counsel They accompanied him back into his house while he changed his clothes, telling him that they were doing so because he was now in their custody Finally, at no point did Mr. Latimer protest or resist the police - he submitted to the authority of the arresting officers The fact that a de facto arrest occurred, however, is not sufficient to dispose of the matter, because of the potential that his arrest was unlawful However, it is not necessary to address that question, because Mr. Latimer's arrest was entirely lawful, and failing an attack against the legislative provision which authorized the arrest, I do not see how a lawful arrest can contravene s. 9 of the Charter for being arbitrary. The trial judge made a specific finding that reasonable grounds for the arrest of Mr. Latimer existed, and I see no reason to disturb that finding Subjectively, despite the fact that the officers decided not to arrest Mr. Latimer, it is clear that they believed that they had reasonable grounds to arrest him Objectively, the reasonable person in the position of the arresting officer would have concluded there were reasonable grounds for arrest 2. Section 10(a) of the Charter provides the right to be informed promptly of the reasons for one's arrest or detention The purpose of this provision is to ensure that a person "understand generally the jeopardy" in which he or she finds himself or herself There are two reasons why the Charter lays down this requirement (see R. v. Evans) 1. Because it would be a gross interference with individual liberty for persons to have to submit to arrest without knowing the reasons for that arrest 2. Because it would be difficult to exercise the right to counsel protected by s. 10(b) in a meaningful way if one were not aware of the extent of one's jeopardy There is no doubt that Mr. Latimer was not told that he was under "arrest", nor was he explicitly told that he could be charged with murder However, when considering whether there has been a violation of s. 10(a), one must look beyond the exact words used 74

On the facts of this case, I have no doubt that the trial judge was right in finding that Mr. Latimer understood the basis for his apprehension by the police and hence the extent of his jeopardy

Appearance Notice
As part of The Bail Reform Bill one of the techniques for discouraging arrests adopted was a new provision for appearance notices s. 496 Criminal Code Issue of appearance notice by peace officer Where, by virtue of subsection 495(2), a peace officer does not arrest a person, he may issue an appearance notice to the person if the offence is (a) an indictable offence mentioned in section 553; (b) an offence for which the person may be prosecuted by indictment or for which he is punishable on summary conviction; or (c) an offence punishable on summary conviction. Indictable offences under s. 553 are those under absolute jurisdiction of provincial courts s. 497 Criminal Code Release from custody by peace officer (1) Subject to subsection (1.1), if a peace officer arrests a person without warrant for an offence described in paragraph 496(a), (b) or (c), the peace officer shall, as soon as practicable, (a) release the person from custody with the intention of compelling their appearance by way of summons; or (b) issue an appearance notice to the person and then release them. Exception (1.1) A peace officer shall not release a person under subsection (1) if the peace officer believes, on reasonable grounds, (a) that it is necessary in the public interest that the person be detained in custody or that the matter of their release from custody be dealt with under another provision of this Part, having regard to all the circumstances including the need to (i) establish the identity of the person, (ii) secure or preserve evidence of or relating to the offence, (iii) prevent the continuation or repetition of the offence or the commission of another offence, or (iv) ensure the safety and security of any victim of or witness to the offence; or (b) that if the person is released from custody, the person will fail to attend court in order to be dealt with according to law. Where subsection (1) does not apply 75

(2) Subsection (1) does not apply in respect of a person who has been arrested without warrant by a peace officer for an offence described in subsection 503(3). Consequences of non-release (3) A peace officer who has arrested a person without warrant for an offence described in subsection (1) and who does not release the person from custody as soon as practicable in the manner described in that subsection shall be deemed to be acting lawfully and in the execution of the peace officers duty for the purposes of (a) any proceedings under this or any other Act of Parliament; and (b) any other proceedings, unless in any such proceedings it is alleged and established by the person making the allegation that the peace officer did not comply with the requirements of subsection (1). s. 498 Criminal Code Release from custody by officer in charge (1) Subject to subsection (1.1), if a person who has been arrested without warrant by a peace officer is taken into custody, or if a person who has been arrested without warrant and delivered to a peace officer under subsection 494(3) or placed in the custody of a peace officer under subsection 163.5(3) of the Customs Act is detained in custody under subsection 503(1) for an offence described in paragraph 496(a), (b) or (c), or any other offence that is punishable by imprisonment for five years or less, and has not been taken before a justice or released from custody under any other provision of this Part, the officer in charge or another peace officer shall, as soon as practicable, (a) release the person with the intention of compelling their appearance by way of summons; (b) release the person on their giving a promise to appear; (c) release the person on the persons entering into a recognizance before the officer in charge or another peace officer without sureties in an amount not exceeding $500 that the officer directs, but without deposit of money or other valuable security; or (d) if the person is not ordinarily resident in the province in which the person is in custody or does not ordinarily reside within 200 kilometres of the place in which the person is in custody, release the person on the persons entering into a recognizance before the officer in charge or another peace officer without sureties in an amount not exceeding $500 that the officer directs and, if the officer so directs, on depositing with the officer a sum of money or other valuable security not exceeding in amount or value $500, that the officer directs. Exception (1.1) The officer in charge or the peace officer shall not release a person under subsection (1) if the officer in charge or peace officer believes, on reasonable grounds, (a) that it is necessary in the public interest that the person be detained in custody or that the matter of their release from custody be dealt with under another provision of this Part, 76

having regard to all the circumstances including the need to (i) establish the identity of the person, (ii) secure or preserve evidence of or relating to the offence, (iii) prevent the continuation or repetition of the offence or the commission of another offence, or (iv) ensure the safety and security of any victim of or witness to the offence; or (b) that, if the person is released from custody, the person will fail to attend court in order to be dealt with according to law. Where subsection (1) does not apply (2) Subsection (1) does not apply in respect of a person who has been arrested without warrant by a peace officer for an offence described in subsection 503(3). Consequences of non-release (3) An officer in charge or another peace officer who has the custody of a person taken into or detained in custody for an offence described in subsection (1) and who does not release the person from custody as soon as practicable in the manner described in that subsection shall be deemed to be acting lawfully and in the execution of the officers duty for the purposes of (a) any proceedings under this or any other Act of Parliament; or (b) any other proceedings, unless in any such proceedings it is alleged and established by the person making the allegation that the officer in charge or other peace officer did not comply with the requirements of subsection (1). s. 499 Criminal Code Release from custody by officer in charge where arrest made with warrant (1) Where a person who has been arrested with a warrant by a peace officer is taken into custody for an offence other than one mentioned in section 522, the officer in charge may, if the warrant has been endorsed by a justice under subsection 507(6), (a) release the person on the persons giving a promise to appear; (b) release the person on the persons entering into a recognizance before the officer in charge without sureties in the amount not exceeding five hundred dollars that the officer in charge directs, but without deposit of money or other valuable security; or (c) if the person is not ordinarily resident in the province in which the person is in custody or does not ordinarily reside within two hundred kilometres of the place in which the person is in custody, release the person on the persons entering into a recognizance before the officer in charge without sureties in the amount not exceeding five hundred dollars that the officer in charge directs and, if the officer in charge so directs, on depositing with the officer in charge such sum of money or other valuable security not exceeding in amount or value five hundred dollars, as the officer in charge directs.

Interrogation Right to Counsel and Arrest Informational duties


77

The law relating to interrogation presents a combination of three bodies of law The common law confessions rule The right to counsel in s. 10(b) of the Charter The right to silence in s. 7 of the Charter The order to instruct accused individuals of their right to counsel is an underdeveloped area of the law in the Code Appears to overlap with s. 29(2) of the Code which requires police to give reasons for arrest o McLachlin J discusses in Evans that the right to have the reasons of an arrest given is an adjunct right to the right to counsel What are the justifications behind the right to counsel? Allow people to understand their rights, including their right to counsel Protect individuals against self-incrimination, false confessions or other injustices There are three basic questions linked to the right to counsel The triggers What are the police supposed to do to uphold this right What is state supposed to do to uphold it

Right to counsel (Charter s. 10(b))


s. 2(c) Canadian Bill of Rights Construction of law 2. Every law of Canada shall, unless it is expressly declared by an Act of the Parliament of Canada that it shall operate notwithstanding the Canadian Bill of Rights, be so construed and applied as not to abrogate, abridge or infringe or to authorize the abrogation, abridgment or infringement of any of the rights or freedoms herein recognized and declared, and in particular, no law of Canada shall be construed or applied so as to ... (c) deprive a person who has been arrested or detained (i) of the right to be informed promptly of the reason for his arrest or detention, (ii) of the right to retain and instruct counsel without delay, or (iii) of the remedy by way of habeas corpus for the determination of the validity of his detention and for his release if the detention is not lawful; s. 10 Canadian Charter of Rights and Freedoms Arrest or detention 10. Everyone has the right on arrest or detention a) to be informed promptly of the reasons therefor; b) to retain and instruct counsel without delay and to be informed of that right; and c) to have the validity of the detention determined by way of habeas corpus and to be 78

released if the detention is not lawful.

1. Triggering mechanisms
Under both the Bill and the Charter, the right to counsel is not absolute, but it is only available under arrest or detention Under the Bill, much of the debate in the courts consisted on whether a person who was subject to a breath test was under detention and thus subject to right to counsel The leading decision on this was Brownridge v. R. (1972) where the SCC held that the denial of a right to counsel constituted a reasonable excuse for failing to comply with a breath sample, as long as the accused is not asserting her right to counsel for the purpose of delay o This was backtracked in Chromiak v. R (1980), where the SCC held that a person subjected to a roadside test was not under detention o Every CA, except for the majority in the Saskatchewan CA (in Therens), held that the detention in Chromiak was still determining under the Charter o The SCC unanimously rejected Chromiak A textual interpretation of triggers would look at determining what arrest or detention mean A purposive interpretation looks into what are the reasons why a right of counsel is available and what type of situations ought to trigger these rights E.g. the unequal power relationship between the State and the accused R. v. Therens, [1985] 1 SCR 613 Facts The accused lost control of his car and it collided with a tree A police officer demanded that he provided breath samples for analysis pursuant to s. 235(1) of the Criminal Code The accused accompanied the officer to the police station, complied with the demand, and was subsequently charged with driving a motor vehicle while having an excessive blood alcohol level contrary to s. 236(1) At trial, the accused objected to the admission of the certificate of analysis and applied, pursuant to s. 24 of the Charter, for its exclusion on the ground that he had been denied the right to be informed, upon arrest or detention, of his right to retain and instruct counsel without delay o The trial judge allowed the application and dismissed the charge for lack of other evidence The majority of the Saskatchewan CA upheld the decision Issue: Did the roadside alcohol test constitute detention under s. 10 of the Charter? Holding: Yes; appeal dismissed Le Dain J (dissenting on the remedy)

79

The accuseds rights under s. 10(b) of the Charter were violated A person who complied with a demand, pursuant to s. 235(1) of the Code, to accompany a police officer to a police station and to submit to a breathalyzer test is "detained" within the meaning of s. 10 of the Charter and that person is therefore entitled to be informed of his right to retain and instruct counsel without delay The word "detention" in s. 10 is directed to a restraint of liberty of varying duration other than arrest in which a person may REASONABLY require the assistance of counsel and might be prevented or impeded from retaining and instructing counsel without delay but for the constitutional guarantee In addition to the case of deprivation of liberty by physical constraint, there is also a "detention" within s. 10 when a police officer assumes control over the movement of a person by a demand or direction which may have significant legal consequence and which prevents or impedes access to counsel There must, however, be some form of compulsion or coercion Any criminal liability for failure to comply with a demand or direction of a police officer is sufficient to make compliance involuntary Under s. 235(2), a refusal to comply with a s. 235(1) demand without reasonable excuse is a criminal offence Notwithstanding any similarity to s. 10 of the Charter, the meaning of the word "detained" in s. 2(c) of the Canadian Bill of Rights as adopted in Chromiak was not determinative of the issue The premise that the framers of the Charter must be presumed to have intended that the words used by it should be given the meaning which had been given to them by judicial decisions at the time the Charter was enacted is not a reliable guide to its interpretation and application The Charter must use general language which is capable of development and adaptation by the court Estey J was content to simply state that the accused was detained within the meaning of s. 10(b) when the officers administered the breathalyzer Lecture Notes According to Le Dain J the person may be detained even when there are no criminal consequences, but if the person feels that she reasonably had no choice other than submit The uncertainty left in the judgment issued in Therens, where Le Dain J spoke for only half of the court, was cleared up in Thomsen Here the court held that someone under a roadside test (under what is now s. 254(2) is detained It later held that there was no right to counsel since the provision constituted a demonstrably justified reasonable limit on s. 10(b) Thomsen v. R, [1988] 1 SCR 640 Facts 80

A police officer engaged in spot checks of motor vehicles stopped the accused's vehicle because it had a defective headlight The officer detected an odour of alcohol on the accused's breath and made a formal demand that he provide a breath sample for the roadside screening device The accused refused and was charged with refusing to comply to give a breath sample At no time did the officer inform the accused that he had a right to retain and instruct counsel without delay Issue: Did the roadside alcohol test constitute detention under s. 10(b) of the Charter? Holding: Yes Le Dain J Restating the elements of detention that appear in Therens 1. In its use of the word "detention", s. 10 of the Charter is directed to a restraint of liberty other than arrest in which a person may reasonably require the assistance of counsel but might be prevented or impeded from retaining and instructing counsel without delay but for the constitutional guarantee 2. In addition to the case of deprivation of liberty by physical constraint, there is a detention within s. 10 of the Charter, when a police officer or other agent of the state assumes control over the movement of a person by a demand or direction which may have significant legal consequence and which prevents or impedes access to counsel 3. The necessary element of compulsion or coercion to constitute a detention may arise from criminal liability for refusal to comply with a demand or direction, or from a reasonable belief that one does not have a choice as to whether or not to comply 4. Section 10 of the Charter applies to a great variety of detentions of varying duration and is not confined to those of such duration as to make the effective use of habeas corpus possible The demand by the police in the case at bar fell under what constitutes detention The way in which the officer assumed control over the movement of the appellant was one which might have significant legal consequence o Given the criminal liability under s. 234.1(2) for refusal, without reasonable excuse, the situation was one in which a person might reasonably require the assistance of counsel The criminal liability for refusal also constituted the necessary compulsion or coercion to make the restraint of liberty a detention [It was later also held that failing to provide right to counsel was demonstrably justified under s.1] The obiter in Thomsen that detention includes psychological detention where the accused reasonably believes that the only choice is to comply with the police, continues to trouble courts The following decisions show that there has been a focus beyond the mind on the accused, in order to determine whether there is a detention within s.10 For example, the intention of the police officers when interviewing a person 81

R. v. Moran (1987), 36 CCC (3d) 225 Facts The accused was convicted of murder Amongst his grounds of appeal were that the trial judge had admitted certain statements made to the police in contravention of s. 10(b) A police officer telephone the accused, who was a friend of the deceased, and he agreed to come to a station where he made some exculpatory statements Some days later, when pressed by the police, he admitted he had lied and gave another exculpatory statement o At neither of these interviews was he advised of his right to counsel, but the trial judge held that he was not detained within the meaning of s. 10(b) at either interview His appeal was dismissed by the CA Issue: Did the questioning of the accused during the interviews at the station constitute a detention within s. 10(b)? Holding: No; appeal dismissed Martin JA As stared in R. v. Bazinet, acquiescence in a "demand" or "direction" is essential to constitute "psychological" detention When determining whether a person who subsequently is an accused was detained at the time she was questioned at a police station by the police, the following non-exhaustive list of factors is relevant 1. The precise language used by the police and whether the accused was given a choice or expressed a preference that the interview be conducted at the Police station, rather than at her home 2. Whether the accused was escorted to the police station by a police officer or came herself 3. Whether the accused left at the conclusion of the interview or whether she was arrested 4. The stage of the investigation, that is, whether the questioning was part of the general investigation of a possible crime or whether the police had already decided that a crime had been committed and that the accused was the perpetrator and the questioning was conducted for the purpose of obtaining incriminating statements 5. Whether the police had reasonable and probable grounds to believe that the accused had committed the crime being investigated 6. The nature of the questions: whether they were questions of a general nature designed to obtain information or whether the accused was confronted with evidence pointing to her guilt 7. The subjective belief by an accused that she is detained, although relevant, is not decisive, because the issue is whether she reasonably believed that she was detained Personal circumstances relating to the accused, such as low intelligence, emotional disturbance, youth and lack of sophistication are to be considered in determining the subjective belief As for the first interview, the officer testified that he asked the appellant whether he would prefer 82

to come to the police station or would prefer that the police come to see him At that time the police were conducting a general investigation of the circumstances of the deceased's death The issue of whether the accused was when he was interviewed again is more complex Even if, in a broad sense, the appellant may have been a suspect at that time, that fact alone is not determinative of the question whether he was detained The officer did not suggest that the appellant had killed the deceased nor did he attempt to obtain an admission from the appellant that he had killed the deceased Finally, after making the statement the appellant told the police: "I feel a lot better now"; He went home, and was not arrested until more than two months later R. v. Mickey (1988), 46 CCC (3d) 278 Facts The accused was convicted of a murder that took place near his residence He originally told the police that he had witnessed the murder and agreed to go to the station to make a statement Throughout the statement it became apparent to the police that he may be the perpetrator of the murder, so he turned the accused over t two other officers for interrogation o He was not advised of his s. 10(b) right o He made a number of statements on which the Crown later relied Issue: Was the accused under detention within s. 10(b) when he was interrogated by the two new officers? Holding: Yes; appeal allowed and a new trial ordered Macfarlane JA The status of the accused changed when it appeared that he might have knowledge which only the police and the assailant had The police officer on learning that fact, no longer treated him as a witness If he had treated him as a witness he would have taken a statement from him and let him go, but instead, he turned him over for questioning to experienced interrogators The appellant was detained from the moment the other two officers took charge of him He was no longer free to leave; he was a suspect and he was being detained at the police station for the purpose of having the police investigate a suspected offence Lecture Notes This case can be seen as shifting the attention in determining what detention means, from subjective feelings of the accused in to the purpose of the investigation R. v. Hawkins (1992), 14 CR (4th) 286 Facts A complain of sexual assault was made by the daughter of a friend of the accused The police contacted him requesting an interview and he chose to go to the police station He was advised of the nature of the complaint and his right to remain silent He agreed to make a written statement with some inculpatory remarks 83

The accused later testified that he was at no time given the impression that the complaint was serious and would amount to anything The accused was eventually convicted of sexual assault Issue: Was the written statement given in violation of s. 10(b)? Holding: Yes; appeal allowed Marshall JA While suspicion will be an important element in assessing the approach taken by an investigating officer, it is not in itself enough to substantiate a detention within the meaning of s. 10(b) A police officer investigating whether an offence has been committed is entitled to question anybody, whether suspected or not It is when these SUSPICIONS BECOME CRYSTALLIZED, and the investigator's approach to the encounter is changed from a questioning of the individual to an examination with an intent to charge her with the offence, that a detention must be deemed to have arisen engaging that person's rights under s. 10(b) The right to be informed of entitlement to counsel must arise at precisely the point where the individual is subject to "the coercive power of the State" It is at this moment that her liberty and security is placed in peril, and, the Charter mandates that the individual be informed not only of her right to silence but also be afforded the additional protection of being advised of the right to consult counsel The act creating the adversarial relationship and TRIGGERING these individual rights can be identified in the centering by the investigating officer upon the individual as the offender and the confining of the interview to attempts to discern her culpability and to elicit from the individual evidence to be used against her This pertains even though the person interviewed may be unconscious of any compulsion in the sense of feeling she has no choice but to respond o It will generally suffice to regard that, from the individual's point of view, an ambience of compulsion permeated the entire interview and it is not necessary to establish that the incriminating statement itself was specifically made because the person felt a lack of choice to do otherwise Lecture Notes The ruling seems to imply that a detention takes place when there is a reasonable suspicion in the mind of the police R. v. Feeney supra, [1997] 2 SCR 13 Facts While investigating a murder, the police went to the accuseds home and entered his residence having received no answer at the door After bringing the accused to better lighting where it was apparent that he was covered in blood, he was arrested Following caution with respect to the right o counsel, the police asked the accused some questions which he answered 84

He was eventually convicted Issue: Were the accuseds statements obtained in violation of s. 10(b)? Holding: Yes Sopinka J Detention began once the officer touched the appellant's leg and ordered him to rise, thus his s. 10(b) rights were violated at the time of his initial detention The accused was not given any caution at this time but only after the appellant had been escorted to the light, thus failing to satisfy the informational requirements of s. 10(b) Furthermore, the accused was not given an adequate opportunity to consult with counsel before being questioned LHeureux-Dub J (dissenting on this matter) The police are not obliged to read an accused his s.10(b) rights the instant he is detained or arrested The police must be permitted the latitude to assess and gain control of the situation and determine whether a potentially dangerous situation exists In addition, the officer was walking into a dark room and the appellant was sleeping, and surely, the officer was not supposed to read the appellant his rights while he was asleep The caution was given at the first reasonable opportunity The delay in this case between the time the officer grabbed the accuseds leg to wake him up and the time he was read his right to counsel was no more than a few minutes Notwithstanding the SCCs unanimous ruling in Hawkins, some courts continued to insist that the authority is Moran The SCCs interpretation of the content of the s.10 (b) guarantee has been dominated by the distinguishing approach between informational and implementation duties in Bartle R. v. Bartle, [1994] 3 SCR 173 Lamer CJ The purpose of the right to counsel guaranteed by s. 10(b) of the Charter is to provide detainees with an opportunity to be informed of their rights and obligations under the law and, most importantly, to obtain advice on how to exercise those rights and fulfil those obligations This opportunity is made available because, when an individual is detained by state authorities, she is put in a position of disadvantage relative to the state Not only has this person suffered a deprivation of liberty, but she may also be at risk of incriminating herself S. 10(b) of the Charter imposes the following duties on state authorities who arrest or detain a person 1. To inform the detainee of her right to retain and instruct counsel without delay and of the 85

existence and availability of legal aid and duty counsel 2. If a detainee has indicated a desire to exercise this right, to provide the detainee with a reasonable opportunity to exercise the right (except in urgent and dangerous circumstances) 3. To refrain from eliciting evidence from the detainee until she has had that reasonable opportunity (again, except in cases of urgency or danger) The first duty is an informational one The second and third duties are more in the nature of implementation duties and are not triggered unless and until a detainee indicates a desire to exercise her right to counsel The right to counsel under s. 10(b) is not absolute Unless a detainee invokes the right and is reasonably diligent in exercising it, the correlative duty on the police to provide a reasonable opportunity and to refrain from eliciting evidence will either not arise in the first place or will be suspended The rights guaranteed by s. 10(b) may be waived by the detainee, although the standard for waiver will be high, especially in circumstances where the alleged waiver has been implicit Moran and Therens still largely remain the authority with regards to the triggers to the right to counsel The difference with later cases lies on the importance given to the subjective feelings of the accused regarding the detention as compared to the subjective feelings of the police

2. Informational duties
Unlike the Bill, s. 10(b) of the Charter expressly confers the right to be informed of the right to retain and instruct counsel This is mandatory on arrest and detention Informational duties refer to stating to the accused of her right to counsel R. v. Brydges, [1990] 1 SCR 190 Facts The accused, a resident of Alberta, was arrested in Manitoba in connection with a murder which took place in Edmonton He was charged with second degree murder and informed without delay of his right to retain and instruct counsel The accused asked the investigating officer if they had Legal Aid in Manitoba because he could not afford a private lawyer o The officer, who was from Edmonton, answered that he imagined that they had such a system in Manitoba o The officer then asked the accused if he felt there was a reason for him to wanting to talk to a lawyer. The accused answered "Not right now, no" During the interrogation which followed, the accused made a number of statements 86

He later interrupted the questioning and requested a Legal Aid lawyer and the lawyer contacted by the police advised the accused not to say anything more and the interrogation ended At trial, the judge found that at the beginning of the interrogation the accused essentially requested the assistance of counsel and because the police did not assist the accused in exercising his right to counsel, by determining the availability of Legal Aid at that time, the accused's rights under s. 10(b) of the Charter were violated As a result, the accused was acquitted A majority of the CA set aside the acquittal and ordered a new trial Issue: Was s. 10(b) violated by failing to inform the accused of the availability of legal aid? Holding: Yes; appeal allowed Lamer J Once the accused requested the assistance of counsel it was incumbent on the police officer to facilitate contact with counsel by giving the appellant a reasonable opportunity to exercise his right to counsel There is a duty on the police to inform him of the existence of duty counsel and the ability to apply for Legal Aid It is not always the case that immediately upon detention an accused will be concerned about retaining the lawyer that will eventually represent him at a trial, if there is one Rather, one of the important reasons for retaining legal advice without delay upon being detained is linked to the protection of the right against self-incrimination The failure of the police to inform the appellant of the existence of Legal Aid or duty counsel at the time that he first indicated a concern about his ability to pay a lawyer, was a restriction on the appellant's right to counsel, in so far as the appellant was left with an erroneous impression of the nature and extent of his s. 10(b) rights It is consistent with the purpose underlying s. 10(b) of the Charter to impose a duty on the police to inform accused individual of the existing of legal aid in all cases of detention, regardless of whether the accused expresses a concern with affording an attorney or not Otherwise police officers would be put in the difficult position of having to judge, on the spot, whether a person has expressed concerns about affordability or whether there should be further inquiries made in this respect Before concluding, it is my view that in light of the imposition of the additional duty on the police as part of the information component of the s. 10(b) caution, a transition period is appropriate This transition period is needed to enable the police to properly discharge their new burden, more specifically to take into account the reality that police officers often use printed cards from which they read the caution given to detainees A period of thirty days from the date of this judgment is sufficient time for the police forces to react, and to prepare new cautions Lecture Notes Does this mean that it is now part of police procedure to advise of legal aid when making 87

reference to the right to counsel? R. v. Bartle (1994), 33 CR (4th) 1 Facts The accused was charged with impaired driving after he failed a breathalyzer test early in Saturday morning A section 10(b) Charter caution was read to him from a card, advising of the right to apply for free legal aid o The officer did not, however, refer to the fact that there was free and immediate preliminary legal advice available from duty counsel, who could be reached by calling a toll free number After being taken to the station he was twice asked if he wanted to speak to a lawyer, but once again he was not informed of this toll free number to free duty counsel On both occasions the accused declined Issue: Was the defendants right in s. 10(b) Charter violated by the absence of information about the free duty counsel? Holding: Yes; the appeal was allowed and the acquittal restored Lamer CJ It is critical that the information component of the right to counsel be comprehensive in scope and that it be presented by police authorities in a "timely and comprehensible" manner Unless they are clearly and fully informed of their rights at the outset, detainees cannot be expected to make INFORMED and MEANINGFUL choices and decisions about whether or not to contact counsel and, in turn, whether to exercise other rights, such as their right to silence (see Hebert) A person who does not understand his or her right cannot be expected to assert it Likewise, before an accused can be said to have waived her right to counsel, she must be possessed of sufficient information to allow her to make an informed choice as regards exercising the right In light of the rule that, absent special circumstances, police are not required to assure themselves that a detainee fully understands the s. 10(b) caution, it is important that the standard caution given to detainees be as instructive and clear as possible Particularly given that subsequent duties on the state are not triggered unless and until a detainee expresses a desire to contact counsel The purpose of the right to counsel would be defeated if police were only required to advise detainees of the existence and availability of Legal Aid and duty counsel after some triggering assertion of the right by the detainee A detainee is entitled under the information component of s. 10(b) of the Charter to be advised of whatever system for free, preliminary legal advice exists in the jurisdiction and of how such advice can be accessed

88

In the case at bar, section 10(b) required that the existence and availability of this duty counsel system and how to access it be routinely communicated by police in a timely and comprehensible manner to detainees The accused was not properly informed of his rights under s. 10(b) and as a result, he may have been misled about the nature and extent of his right to counsel The s. 10(b) caution that the appellant received, both at the roadside and at the police station, failed to convey the necessary sense of immediacy and universal availability of legal assistance, which the majority in Brydges said must be conveyed as part of the standard s. 10(b) warning in jurisdictions where such a service exists o Reference to Legal Aid was confusing in so far as it implied that free legal advice, while available, was contingent on applying for it once charged The 1-800 number, or at least the existence of a toll-free telephone number, should have been conveyed to the appellant upon his arrest at the roadside even though there were no telephones available Furthermore, the appellant did not waive his right to receive a caution that fully informed him of his right to counsel o The validity of a waiver of a procedural right is dependent upon it being clear and unequivocal that the person is waiving the procedural safeguard and is doing so with full knowledge of the rights the procedure was enacted to protect One thing that INFORMATIONAL DUTIES DO NOT REQUIRE, is that the accused in fact understands the right to counsel In Baig the SCC placed the onus on the accused to show that she did not understand the right The IMPLEMENTATION DUTIES, on the other hand, do require that the accused understands her right

2. Implementation duties
Laskins minority concurring judgment in Brownridge is still referred to in this respect, stating that the right to counsel raises a correlative obligation on the police to facilitate contact with counsel, such as providing the accused with a phone Some courts have held that privacy should be afforded by the police to the accused in contacting counsel, while the SCC in Jumaga v. R (1977), held that it is not a requirement The positive wording of s. 10(b) has been relied upon to distinguish Jumaga and hold that privacy must be afforded whether or not it is requested (see R. v. Playford (1987)) R. v. Manninen, [1987] 1 SCR 1233 Facts The accused was arrested for theft and possession of a stolen car and armed robbery The arresting officer read him his rights from a card twice because of a dismissive remark respondent had made following the first reading Even though the accused indicated that he was not going to say anything until he saw his 89

lawyer, the officers continued to question him He did not directly request to use the telephone and the officers did not volunteer the use of it o He did not speak to his lawyer until his lawyer called him at the police station that evening The trial judge, in convicting the accused, relied on a statement made by him in reply to a barbed question asked before his lawyer had made contact with him The trial judge held that, even if the right to counsel had been infringed, the admission of the statements would not bring the administration of justice into disrepute The CA unanimously allowed the accuseds appeal, quashed the convictions and ordered a new trial Issue: Was the accuseds right to counsel infringed? Holding: Yes; appeal dismissed Lamer J S.10(b) imposes at least TWO duties on the police in addition to the duty to inform the detainee of her rights 1. The police must provide the detainee with a reasonable opportunity to exercise the right to retain and instruct counsel without delay This means allowing her upon his request to use the telephone for that purpose if one is available 2. The police must cease questioning or otherwise attempting to elicit evidence from the detainee until she has had a reasonable opportunity to retain and instruct counsel The purpose of the right to counsel is to allow the detainee not only to be informed of her rights and obligations under the law but, equally if not more important, to obtain advice as to how to exercise those rights An example of this is the right to remain silent S. 10(b) requires at least that the authorities inform the detainee of her rights and not prevent her in any way from exercising them Also, where a detainee is required to provide evidence which may be incriminating and refusal to comply is punishable as a criminal offence, s. 10(b) also imposes a duty not to call upon the detainee to provide that evidence without first informing her of her s. 10(b) rights and providing him with a reasonable opportunity and time to retain and instruct counsel The first aspect of the right to counsel was clearly infringed in this case There was a telephone immediately at hand in the office, which the officers used for their own purposes o It was not necessary for the respondent to make an express request to use the telephone Of course, there may be circumstances in which it is particularly urgent that the police continue with an investigation before it is possible to facilitate a detainee's communication with counsel, yet there was no urgency in this case

90

The second aspect of the respondent's right to counsel was also infringed Immediately after the respondent's clear assertion of his right to remain silent and his desire to consult his lawyer, the police officer commenced his questioning as if the respondent had expressed no such desire The police proceed with their questioning of the detainee before providing him with a reasonable opportunity to retain and instruct counsel, but there was no such urgency in this case. The Crown contends that there was no infringement of the right to counsel because the respondent had waived his right by answering the police officer's questions While a person may implicitly waive their rights under s. 10(b), the standard will be very high (see Clarkson) The accuseds conduct in this case did not constitute an implied waiver of his right to counsel o He did not intend to waive his right, as he clearly asserted it at the beginning and at the end of the questioning o The respondent had the right not to be asked questions, and he must not be held to have implicitly waived that right simply because he answered the questions Otherwise, the right not to be asked questions would only exist where the detainee refused to answer Lecture Notes By talking to the police regardless of having asked for counsel, the accused was not waiving his rights In Manninen the accused had clearly asserted his right to counsel so it was not necessary to decide whether the police correlative duty only arises where such an assertion is made This issues was discussed in Baig Baig v. R., [1987] 2 SCR 537 Facts The accused was arrested for murder and promptly informed of his right to counsel When asked if he understood his right, the accused replied "How can you prove this thing?" At the police station, he made an inculpatory statement and the statement form indicated that the accused gave an affirmative answer to the question of whether he understood that he had the right to retain and instruct counsel without delay At trial, the trial judge ruled that the accused's right under s. 10(b) of the Charter had been violated and excluded the statement The accused was acquitted on a directed verdict The Court of Appeal quashed the acquittal and ordered a new trial Issue: Are the police correlative duties in relation to s. 10(b) triggered if they accused has not yet expressed a desire to exercise the right to counsel? Holding: No; appeal dismissed The Court 91

The judges agreed with Tarnopolsky J.A. in R. v. Anderson (1984) in that absent proof of circumstances indicating that the accused did not understand her right to retain counsel when she was informed of it, the onus has to be on her to prove that she asked for the right but it was denied or she was denied any opportunity to even ask for it No such evidence was put forth in this case Absent such circumstances, once the police have complied with s. 10(b), by advising the accused without delay of her right to counsel, there are no correlative duties triggered and cast upon them until the accused, if she so chooses, has indicated his desire to exercise her right to counsel Comments Can Baig be reconciled with Manninen? Should the right of counsel once invoked by a suspect, limit subsequent questioning by the police? Leclair and Ross v. R., [1989] 1 SCR 3 Facts Three youth were arrested at night and charged with break, enter and theft They were advised of their right to counsel and tried unsuccessfully to contact counsel by telephone at 2 am Leclair was asked if he wished to call another lawyer and he said no The police conducted a line-up an hour later and the accused were not advised that they were under no obligation to participate Issue: Were the implementation duties of the police successfully exercised? Holding: No Lamer J As expressed in Manninen s. 10(b) imposes at least two duties on the police in addition to the duty to inform detainees of their rights The first is that the police must give the accused or detained person who so wishes a reasonable opportunity to exercise the right to retain and instruct counsel without delay The second is that the police must refrain from attempting to elicit evidence from the detainee until the detainee has had a reasonable opportunity to retain and instruct counsel The police fulfilled neither duty in this case 1. Affording a Reasonable Opportunity It was highly unlikely that the accused would be able to contact their counsel outside the normal office hours The fact that Leclair did not want to call another lawyer cannot be viewed as a waiver of his right to retain counsel He merely asserted his right to counsel and to counsel of his choice As per R. v. Tremblay (1987), a detainee must be reasonably diligent in the exercise of the right to counsel, otherwise the correlative duties imposed on the police and set out in 92

Manninen are suspended Accused or detained persons have a right to choose their counsel and it is only if the lawyer chosen cannot be available within a reasonable time that the detainee or the accused should be expected to call another lawyer Once Leclair asserted his right to instruct counsel, and absent a clear indication that he had changed his mind, it was unreasonable for the police to proceed as if he had waived his right to counsel

As stated in Korponay v. Attorney General of Canada (1982), any WAIVER "is dependent upon it being clear and unequivocal that the person is waiving the procedural safeguard and is doing so with full knowledge of the rights the procedure was enacted to protect and of the effect the waiver will have on those rights in the process" In the case of Ross, there is no evidence that the police even asked whether he wanted to call another lawyer There was NO URGENCY justifying that the police proceed immediately and it cannot be said that the appellants had a real opportunity to retain and instruct counsel 2. Refraining from Taking Further Steps As held in Manninen, the police have a duty to cease questioning or otherwise attempting to elicit evidence from the detainee until she has had a reasonable opportunity to retain and instruct counsel This also means that, once an accused or detained person has asserted that right, the police cannot, in any way, compel the detainee or accused person to make a decision or participate in a process which could ultimately have an adverse effect in the conduct of an eventual trial until that person has had a reasonable opportunity to exercise that right In the case at bar, it cannot be said that the appellants had a real opportunity to retain and instruct counsel before the line-up was held Nor can it be said that there was any urgency or other compelling reason justifying proceeding so precipitously The Crown also submitted that there was no violation of the right to counsel because the appellants did not have the right to have their lawyers present during the line-up But, even if the appellants could not have their lawyers present during the line-up, this does not imply that counsel is of no assistance to a suspect o The accused could also have been told how a well-run line-up is conducted There is further no legal obligation to participate in a line-up Furthermore, that the accused did not refuse to participate in the line-up cannot by itself amount to a waiver of the right to counsel It would contradict the purpose of this right to conclude that a detained or accused person has waived the right to counsel simply by submitting, before being instructed by counsel, to precisely those attempts from which the police should refrain 93

R. v. Burlingham, [1995] 2 SCR 206 Facts The accused, who had been charged with one murder and was suspected in a second, was subjected to an intensive and often manipulative interrogation by the police o He was systematically questioned notwithstanding his stating repeatedly that he would not speak unless he could consult with his lawyer The police offered the accused a "deal" he would be charged with second degree murder if he provided the police with the location of the gun and other ancillary information related to that murder o When he refused to accept the "deal" without consulting his lawyer, the officers continued to badger him about the usefulness of his lawyer and informed him this "one-time" chance would be kept open only for the weekend -- the period when appellant's counsel was unavailable o He eventually agreed, despite his being advised by another lawyer not to talk to the police, and fulfilled his part of the deal by giving police a full confession, bringing them to the murder site, and telling them where the murder weapon had been thrown A misunderstanding arose as to the deal, since the accused understood that he would be allowed to plead not guilty to a charge of second degree murder whereas the Crown insisted that he would have to plead guilty to that charge o The trial judge found as a fact that the police officers had made an honest mistake The accused was charged with first degree murder The trial judge found that appellant's right to counsel had been breached and held that appellant's confession, his disclosure of the location of the weapon and his directions and gestures to the police were inadmissible o He, nonetheless, admitted the fact of finding the gun, the actual gun and the testimony of some witnesses The accused was convicted of the first degree murder and the CA affirmed that decision. Issue: Was the accuseds denied of his right to counsel? Holding: Yes; appeal allowed Iacobucci J The Supreme Court has consistently given a broad interpretation to s. 10(b) In the case at bar, there were several ways in which the appellant's right was denied 1. The police continually questioned him despite his repeated statements that he would say nothing absent consultation with his lawyer Section 10(b) requires, barring urgent circumstances, that the police refrain from attempting to elicit incriminatory evidence once a detainee has asserted his or her right to counsel 2. S. 10(b) specifically prohibits the police, as they did in this case, from belittling an accused's lawyer with the express goal of undermining the accused's confidence in and relationship with defence counsel

94

3. S. 10(b) was violated when the officers pressured the accused into accepting the "deal" without first having the opportunity to consult with his lawyer Allowing the appellant to call a random lawyer is, given the seriousness of the situation he faced and the circumstances of this case, insufficient for the officers to discharge their responsibilities under s. 10(b) Although it is clear that s. 10(b) does not guarantee an accused the right to the counsel of his or her choice at all times, in a situation such as the appellant's, the offer should have been made at a point in time when the accused's lawyer was available or the police should have kept it open to a point in time when the accused's counsel would reasonably be considered to be available As discussed in Evans, the police have the duty to advise a suspect of the right to counsel where there is a fundamental and discrete change in the purpose of an investigation which involves a different and unrelated offence or a significantly more serious offence than that contemplated at the time of the original instruction of the right to counsel S. 10(b) mandates the Crown or police, whenever offering a plea bargain, to tender that offer either to accused's counsel or to the accused while in the presence of her counsel, unless the accused has expressly waived the right to counsel. It is important to emphasize that, in the case at bar, there was no urgency to the matter Mere expediency or efficiency is not sufficient to create enough "urgency" to permit a s. 10(b) breach Lecture Notes One of the issues raised in this case was that when a plea bargaining is offered, the accused should be advised of her right again In R. v. Prosper (1994), the SCC held that there is no constitutional obligation on governments to provide a free duty counsel on arrest or detention Lamer J further held for the majority that once the right is asserted, s. 10(b) is violated where the accused is not given a reasonable opportunity to consult counsel, where the lack of availability of duty counsel is to be taken into consideration by the court

3. Waiver and duty to be reasonably diligent in exercise of right


Generally speaking, the Supreme Court has been generous to the accused when characterizing the issues of waiver of the right to counsel It is far less generous when insisting on the duty to assert the s. 10(b) right with reasonable diligence It seems unclear, however, which ruling will be adopted in each case, thus leading to inconsistencies There is a difference between the standards used in the right to counsel and the right to silence 95

The later uses an operating mind test The former involves an awareness of the consequences test

R. v. Clarkson, [1986] 1 SCR 383 Facts The accused was very intoxicated when she was charged with her husband's murder, as well as when she was given the customary police warning and informed of her right to counsel She said there was "no point" in having counsel and underwent police questioning while still drunk and very emotional The interrogation continued in spite of the efforts of an aunt to have it postponed and to convince appellant to stop talking until counsel was present The trial judge excluded inculpatory statements finding that she did not appreciate the consequences of making such statements The CA rejected the trial judge's test of admissibility and ordered a new trial Issue: Could the accused effectively waive her right to counsel while being intoxicated? Holding: No; appeal allowed Wilson J Given the concern for fair treatment of an accused which underlies the right to counsel in s. 10(b), it is evident that any alleged waiver of this right by an accused must be carefully considered and that the accused's awareness of the consequences of what she was saying is crucial As per Korponay v. Attorney General of Canada (1982), any waiver "is dependent upon it being clear and unequivocal that the person is waiving the procedural safeguard and is doing so with full knowledge of the rights the procedure was enacted to protect and of the effect the waiver will have on those rights in the process" US jurisprudence shows that an accused must knowingly intelligently and with a full understanding of the implications, waive her constitutional rights to counsel The US Supreme Court has gone so far as to indicate that not only must an accused person be cognizant of the consequences of waiving the constitutional right to counsel in a general way, but she must be aware of the legal specificities of her own case It is clear that the waiver of the s. 10(b) right by an intoxicated accused must pass some form of "AWARENESS OF THE CONSEQUENCES" test Any voluntary waiver in order to be valid and effective must be premised on a true appreciation of the consequences of giving up the right Unlike the confession itself, there is no room for an argument that the court in assessing such a waiver should only be concerned with the probative value of the evidence Rather, the purpose of the right, as indicated in Therens, is to ensure that the accused is treated fairly in the criminal process Application to the case 96

The trial judge found as a fact that the appellant's confession could not pass the "awareness of the consequences" test, so then presumably neither could the waiver of the s. 10(b) right to counsel At the very minimum it was incumbent upon the police to delay their questioning and the taking of the appellant's statement until she was in a sufficiently sober state to properly exercise her right to retain and instruct counsel or to be fully aware of the consequences of waiving this right Lecture Notes The standard of awareness of the consequences appears to be higher than the operating mind used in the right to silence cases Bartle and the companion cases also hold that a waiver of the informational component of s. 10(b) will have to be explicit and require a reasonable belief that the accused knew about the right to counsel and how to exercise it R. v. Smith, [1989] 2 SCR 368 Facts The accused was arrested at his home around 7:00 p.m., charged with robbery and informed of his right to retain and instruct counsel After various stops made to accommodate the accused, the police officers and the accused arrived at the police station at approximately 9:00 p.m The accused requested the opportunity to communicate with his lawyer and the police gave him a telephone and a telephone book o Because it was late and the only telephone number appearing in the telephone book was his lawyer's office number, the accused decided not to call and advised the police that he would contact his lawyer in the morning o The police suggested that he try to make the call but he refused He was placed in a police cell for approximately one hour and then taken to an interview room o The accused told the police that he would not answer questions concerning the robbery until he could speak with his lawyer, but the police pursued their questioning and tried to obtain a statement o The accused indicated in two other occasions that he wanted to speak to his lawyer but he finally made a statement, specifying that it was made "off the record" The trial judge ruled that the accused's rights under s. 10(b) had not been violated and admitted the statement The accused was subsequently convicted and the majority of the CA upheld the conviction Issue: Was the accused given reasonable opportunity to retain and instruct counsel? Holding: Yes; appeal dismissed Lamer J As per R. v. Tremblay (1987), the duties imposed on the police as stated in Manninen are suspended when the arrested or detained person is not reasonably diligent in the exercise of her rights

97

This limit on the right to counsel is essential because without it, it would be possible to delay needlessly and with impunity an investigation and even, in certain cases, to allow for an essential piece of evidence to be lost, destroyed or rendered impossible to obtain The rights set out in the Charter, and in particular the right to retain and instruct counsel, are not absolute and unlimited rights They must be exercised in a way that is reconcilable with the needs of society The situation would be very different in a case like R. v. Ross, if the accused had tried to contact her lawyer but had failed in her attempt In these circumstances, the accused would have been justified to ask or a delay until the opening of offices in the morning The burden of proving that it was impossible for the accused to communicate with his lawyer when the police offered him the opportunity to do so was on the accused An arrested or detained person who has had a reasonable opportunity to communicate with counsel but who was not diligent in the exercise of this right cannot, subsequently, require the police to suspend, one more time, the investigation or the questioning This does not apply, however, when the circumstances that exist when she asks subsequently to exercise the right to counsel are substantially different from those which existed when he had the opportunity to communicate with a lawyer (e.g. when she originally was accused of something and is later accused of another crime) LHeureux-Dub J The circumstances of time and place as well as the responsible behaviour of the police officers clearly gave the accused more than a reasonable opportunity to communicate with counsel of his choice, or at the very least, to obtain legal advice before answering the questions The accused elected not to avail himself of this opportunity Sopinka J In this case the accused was most casual in asserting his right to counsel The accused had been afforded a reasonable opportunity to retain and instruct counsel This determination is largely a question of fact What is a reasonable opportunity is determined by reference to all the circumstances of the case, including the action of the accused Whether an opportunity is reasonable must be judged in part in light of the diligence with which the accused seeks to avail herself of the right La Forest J (dissenting) Following Clarkson, it cant be disputed that the accused waived his right to counsel The accused positively and repeatedly asserted his desire to exercise his right to counsel Answering the questions put to him does not constitute a waiver of his right to counsel in 98

these circumstances It is clear that these answers were given after he had asserted his desire to speak with his lawyer It is also clear that he believed they were given "off the record", In these circumstances, it could not be said that there was an awareness of the consequences of speaking in the absence of his lawyer

The appellant did not have a reasonable opportunity to contact counsel He decided not to make the call thinking, quite reasonably that at that time of night there would be no one in the lawyer's office The evidence also discloses that the police actively dissuaded the accused from his resolve not to speak until he had talked with his lawyer The accused should be able to wait and get in touch with his lawyer, rather than with any lawyer If the investigation needed urgently to be pursued, the position might be different, but it cannot be said that there was any urgency in this case This case is governed by Ross There was no reason to proceed before the appellant had the opportunity to speak with counsel the next morning The only factor I can find in the police's favour in this case is that the appellant did not attempt to call his lawyer However, I cannot hold it against the appellant that he was willing to spend the night in jail when the likelihood was that he would not have been able to talk to his lawyer that night anyway

Right to Silence
This right has been very poorly defined in Canada and did not exist for a long time Until recently, the confession rule was the only standard restricting the admissibility of some incriminating statements The reasons behind exercising this right are the following Unequal power balance between the State and the accused Risk of a false confession Confession rule Comes from Ibrahim o Confessions made to persons in authority under promises or threats may be unreliable

99

In Hebert, the idea of voluntariness was expanded to include the reputation of the administration of justice On the burden of proof Voluntariness has to be proved by the Crown beyond a reasonable doubt Showing that the administration of justice was brought into disrepute has to be done by the accused, as any other Charter violation o Then the burden shifts to the Crown What is the Crown requested to show under the voluntary confession rule? (assuming that the person who heard the confession was a person in authority) See Oickle No threats of promises Operating mind o Does not require more than knowledge of what the accused is saying No oppression o Though Oickle set a very high bar for oppression No police trickery o This forms part of a distinct inquiry, which deals with the integrity of the justice system It seems like the first three deal with the reliability of the confession o The standard is the shock of the community o Police informants seem to be admissible within this high standard of trickery As for the right to silence under s.7 of the Charter Police persuasion is acceptable After detention o Voluntary statements to cellmates are allowed o Undercover officers to observe are allowed, but not to elicit information s. 7 Canadian Charter of Rights and Freedoms Life, liberty and security of person Everyone has the right to life, liberty and security of the person and the right not to be deprived thereof except in accordance with the principles of fundamental justice. R. v. Hebert, [1990] 2 SCR 151 Facts The accused was arrested on a charge of robbery and informed upon arrest of his right to counsel. At the police station, after consulting counsel, he advised the police that he did not wish to make a statement The accused was then placed in a cell with an undercover police officer posing as a suspect under arrest by police. The officer engaged the accused in conversation, during which the accused made various incriminating statements implicating him in the robbery Prior to trial, there was a voir dire to determine the admissibility of these statements 100

o The judge held that the accused's right to counsel under s. 10(b) of the Charter and his right to remain silent asserted under s. 7 of the Charter had been violated and excluded the statements pursuant to s. 24(2) of the Charter o The Crown offered no evidence, and the accused was later acquitted The Court of Appeal set aside the accused's acquittal and ordered a new trial. The Court found that the police conduct did not violate the accused's right to counsel or his right to remain silent Issue: Did a statement elicited by an undercover police officer once the accused expressed his intention not to speak to the police violate his Charter rights? Holding: Yes; the appeal was allowed and the acquittal was restored McLachlin J Section 7 of the Charter accords a detained person a pre-trial right to remain silent which extends beyond the narrow formulation of the confessions rule The rules relating to the right to remain silent and the privilege against self-incrimination, suggest that the scope of the right in the pre-trial detention period must be based on the fundamental concept of the suspect's right to freely choose whether to speak to the authorities or remain silent This concept, accompanied by a concern with the repute and integrity of the judicial process, is consistent with the right to counsel and the right against self-incrimination affirmed by the Charter. It is also consistent with the Charter's approach to the question of improperly obtained evidence under s. 24(2) and with the underlying philosophy and purpose of the procedural guarantees the Charter enshrines -- in particular in s. 7 Under s. 7, the state is not entitled to use its superior power to override the suspect's will and negate his choice to speak to the authority or to remain silent o The test to determine whether the suspect's choice has been violated is ESSENTIALLY OBJECTIVE The scope of the right to silence, however, does not go as far as to prohibit police from obtaining confessions in all circumstances The s. 7 right to silence retains the objective approach to the confessions rule and would the following limits o There is nothing that prohibits the police from questioning an accused or a suspect in the absence of counsel after he has retained counsel Police persuasion, short of denying the suspect the right to choose or of depriving him of an operating mind, does not breach the right to silence o The right applies only after detention o The right does not affect voluntary statements made to fellow cell mates o A distinction must be made between the use of undercover agents to observe the suspect, and the use of undercover agents to actively elicit information in violation of the suspect's choice to remain silent o Even where a violation of the suspect's right is established, the evidence may, where appropriate, be admitted. Only if the court is satisfied that its reception would be likely to bring the administration of justice into disrepute can the evidence be rejected under 101

s. 24(2) of the Charter o Where the police have acted with due care for the suspect's rights, it is unlikely that the statements they obtain will be held inadmissible. Here, the accused exercised his choice not to speak to the police and the police violated his right to remain silent under s. 7 of the Charter by using a trick to negate his decision. Section 1 of the Charter was inapplicable because the police conduct was not a limit "prescribed by law" within the meaning of that section The evidence obtained in breach of the accused's right under s. 7 should be excluded pursuant to s. 24(2) of the Charter, otherwise the reception of the evidence would render the trial unfair and the accused would be deprived of his presumption of innocence Lectures notes A confession is not voluntary when the behavior of the police is likely to bring the administration of justice into disrepute or when the person cannot exercise her choice to remain silent R. v. Oickle, [2000] 2 SCR 3 Facts Between Feb 94 and April 95, there were 8 fires in 4 buildings and 2 cars in Waterville, Nova Scotia One car belonged to the accuseds dad and one belonged to his fiance, Tanya Fires appeared to have been deliberately set, with the possible exception of Tanyas car Cops did extensive investigation including asking 7 or 8 people to sit for polygraphs. 5 or 6 did and passed and were removed from list of suspects Oickle agreed to sit for the test and went to a motel at 3 pm one day for it. The police audiotaped the events and Sgt Taker administered the test The accused was informed of rights, and that he could leave at any time. He was given pamphlet to review and signed a consent form In a lengthy pre-test interview, an exculpatory statement was taken, which formed the basis for the polygraph test The sergeant then did the test, which lasted only a few minutes. At its conclusion Oickle was told that he had failed. He was reminded of his rights and then questioned for more than an hour At one point he asked:what if I admit to the car? Then I call walk out of here and its over. The officer replied he could walk out whenever Time later Corporal Deveau took over and reminded Oickle of his rights. After 30-40 minutes, he confessed to torching Tanyas car He was emotionally distraught, the police took a written statement and he continued to deny involvement in any other fires The accused was then arrested, advised of his rights and taken to a police station. He was very upset and crying He was put in room and videotaped while questioned for longer, and after a while he said he was tired and wanted to go home and he was told that he was under arrest and couldnt do so Another Constable, Bogle, took over the interrogation, which had so far lasted 11 hours, 102

and he confessed to 7 of the 8 fires. He denied any involvement in his dads van He was very distraught. The police reminded him of his rights and took a written statement, which lasted until 1 am. He was then put in a cell to sleep at 2:45 am and at 6am Corp Deveau asked if he would agree to a re-enactment. He was taken around the town explaining how he set each fire He was charged with 7 counts of arson. At trial the voir dire was held by judge to determine if the statements and video reenactment were admissible. She held that they were voluntary and admissible and convicted on all counts The CA allowed the appeal on grounds that statements were not voluntary. They entered acquittals Issue: Where the statements voluntary? Holding: Yes; the appeal was allowed and the convictions restored Iacobucci J There are 2 strands to the confessions rule Under one approach any statement made by cops where there is an explicit promise or threat to accused should be excluded (as in Ibrahim). This gives the accused a negative right - i.e. the right not to be tortured or coerced into making a statement by threats or promises held out to him by a person who is and whom he subjectively believes to be a person in authority The decision in Hebert recognized a broader approach whereby the absence of violence was not dispositive of the case you still need the necessary mental element of deciding between alternative options The Charter does not subsume these common law rules regarding confessions. The confessions rule has a broader scope than the Charter The Charter also has a different standard and burden of proof. Remedies under the charter are different as well Confessions rule today is concerned with voluntariness, broadly defined This is rooted in a concern about reliability It has twin goals: protecting the accuseds rights while not unduly limiting societys need to investigate and solve crimes Both of these objectives must be borne in mind in an analysis One of the key concerns of the criminal justice system is that the innocent must not be convicted, so the confession rule prevents the solicitation of false confessions 5 categories of confessions Voluntary Stress-compliant confessions o The person will say anything to get rid of the stress The coerced compliant o Coercion by threats or promises to confess 103

o Most cases of false confessions come out of these o Most cases develop in relation to this Non-coerced persuaded o Take someone who does not recall the events well and lead her to confess to something didnt happen Someone with mental challenges, who was intoxicated at the time, etc o One of the ways to go about persuading someone to confess is through the fabrication of false facts o The would likely get the most false confessions from vulnerable people and those who are easily influenced to agree with others o You have to look at the strength of mind of the accused Coerced-persuaded

Confessions rule today must be applied contextually. The following factors should be considered by trial judges in reviewing confessions: Threats or promises o Reduced charge o Minimizing the seriousness of the crime o Offer psychological help o Moral or spiritual inducements Oppression o Denial of food, clothes, rest, medical attention, intimidating questioning o Has a large impact on false confessions Operating mind o Inspired by principle of voluntariness and the accused being aware of what she is saying Other police trickery o Shock to the community criteria (as per Lamer in Rothman) Because of the overall concern about not convicting the innocent, a confession will not admissible if it is made under circumstances that raise a reasonable doubt as to voluntariness Oppressive conditions and inducements can operate together to exclude confessions Trial judges must pay attention to all circumstances when making this determination In this case, there was nothing wrong with the confession and how it came about Oickle was fully aware of his rights at all times, he was never subjected to harsh, aggressive or overbearing interrogation and not deprived of food, sleep or drink and was not offered any improper inducements. With regards to the claim from CA that the cops minimized the seriousness of the crimes, this was not a problem. Any sort of package deal was raised by Oickle and not the cops. While the cops minimized the moral significance of the crimes, there was no suggestion that they would minimize the legal consequences thereof CA held that the cops improperly offered psychological help in return for a confession. Iacobucci points out that there was no suggestion that he would only be able to get help if 104

he confessed. Although there were moral inducements made, such as Oickle would feel better if he confessed, there were no implied threats in this. With regards to alleged threats against his fiance, although the cops did say they might have to interview her, she was not threatened with harm CA also found the use of the polygraph test problematic. Merely failing to tell an accused that the polygraph is inadmissible will not automatically produce an involuntary confession. Courts need to do a 2-stage process. (I) it should be excluded if the police deception shocks the community and (II) use of deception should be considered within the rest of the overall voluntariness analysis. In this case, the police made it clear what was admissible and what was not The police did exaggerate the validity of the polygraph but not to the extent that the respondent was overwhelmed by the polygraph results Lecture Notes Oickle has been seen as formally expanding the right to silence rule and defining the operating mind standard from Hebert o Yet the Court seems to place the bar very high for the accused to rely on this right The Court gave some examples of instances when the confessions may be thrown out o E.g. when a person is made a promise in relation to someone else o Telling a person that if they do not confess they will go to hell R. v. Singh, [2007] SCC 48 [Extra reading] Facts The accused was arrested for second degree murder in respect of the shooting death of an innocent bystander who was killed by a stray bullet while standing just inside the doorway of a pub The accused was advised of his right to counsel under s. 10(b) of the Charter and privately consulted with counsel During the course of two subsequent interviews with police, the accused stated on numerous occasions that he did not want to talk about the incident. The interviewing officer persisted in trying to get him to make a statement While the accused never confessed to the crime, he made a number of admissions which, when taken together with other evidence, later became probative of the issue of identification at trial On the voir dire to determine the admissibility of the statements made by the accused, the trial judge held that the admission came freely and did not result from the police systematically breaking down his operating mind or undermining his right to silence The probative value of the statements was held to outweigh their prejudicial effect and the trial judge thus ruled them admissible The accused was subsequently convicted by a jury The Court of Appeal upheld the trial judges ruling and affirmed the conviction In the CA and in this appeal to the SCC the accused did not contest the trial judges findings of fact nor his conclusion that the statements were voluntary; his appeal solely concerns the s. 7 Charter right to silence Issue: Did the admission of these statements violate the accuseds right to silence as per s. 7? 105

Holding: No; the appeal should be dismissed Charron J Although historically the confessions rule was more concerned with the reliability of confessions than the protection against self-incrimination, this no longer holds true in the post-Charter era The modern view of the confessions rule includes the right of the detained person to make a meaningful choice whether or not to speak to state authorities On the question of voluntariness, the focus is on the conduct of the police and its effect on the accuseds ability to exercise her free will o The test is an objective one, but the individual characteristics of the accused are obviously relevant considerations in applying this objective test o After detention, once the accused cannot walk away, she is in a more vulnerable position In the context of an interrogation of a detainee by an obvious person in authority, a finding of voluntariness will be determinative of the s. 7 issue In such circumstances, the confessions rule effectively subsumes the constitutional right to silence because the two tests are functionally equivalent o If a statement has survived a thorough inquiry into voluntariness, the accuseds Charter application alleging a violation of the pre-trial right to silence under s. 7 cannot succeed o Conversely, if circumstances are such that the accused can show on a balance of probabilities that the statement was obtained in violation of her constitutional right to remain silent, the Crown will be unable to prove voluntariness beyond a reasonable doubt It is not appropriate to impose a rigid requirement that police refrain from questioning a detainee who states that he or she does not wish to speak to police Under both the common law and Charter rules, police persistence in continuing an interview, despite repeated assertions by the detainee that he wishes to remain silent, may well raise a strong argument that the subsequently obtained statement was not the product of a free will to speak to authorities The trial judge in this case was very much alive to this risk. His ultimate judgment call on this issue is supported by the record and is entitled to deference Fish J The interrogator understood very well that the accused had chosen not to speak with the police but nonetheless disregarded the accuseds repeated assertions of his right to silence In his relentless pursuit of a confession no matter what, the interrogator urged the accused, subtly but unmistakably, to forsake his counsels advice While detainees who have asserted their right to silence are entitled to change their minds , they cannot be compelled to do so by the persistent disregard of that asserted choice

106

A confession may be voluntary under the common law rule and yet be obtained by state action that infringes s. 7 of the Charter A confession that meets these common law standards does not invariably represent a free and meaningful choice for the purposes of the Charter Why is there a doctrine of waiver to right to counsel, but not to the right to silence? According to McLachlin, in Hebert, in order to be able to waive a right, you need to be able to understand the full circumstances surrounding the waiver If there was a doctrine of waiver in the right of silence, this would undermine the power of the police to use undercover police officers or other types of trickery Unfortunately Singh did not resolve the discrepancies surrounding this issue The court found that the voluntary confessions rules and the right to counsel were the same

Bail Hearings

At some point the investigation of a crime would allow the State to lay charges against someone Here the State will try to find a way to compel the person to appear in court In lieu of arresting the person until the train, the state can offer a notice to appear or a recognizance A recognizance is an acknowledgment of the debt that person will owe if they do not appear in court After this procedure happens, an information will be laid in front of a judge, which establishes the reasons for the persons arrest The judge must receive this information as part of her administrative functions o She cannot amend it Only then can the judge confirm the processes issued by the police earlier o E.g. the requirement to appear Once the judge has issued this order, the accused will appear in court for arraignment or the first appearance o People who are not offered bail remain in custody until the trial o The term used in the Code is judicial interim release, though bail is more largely used commonly o Generally speaking the burden is on the Crown to prove that the person should not be released, though under some circumstances the onus can be reversed Depending on the offence, a decision will be reached by the judge The rational for the Bail Reform Act of 1961 was mainly twofold Minimize incidence of arrest Minimize detention before trial Research found that when people were being held without bail (independent of the seriousness of the offence) convictions were more common, as well as being sentenced to serve time in prison 107

o Pre-trial detention could lead to more accused people pleading guilty and agreeing to harsher sentences o Pre-trial detention may limit the ability of the accused to gather evidence and build her case o Someone who is incarcerated cannot earn an income, thus may not be able to afford a good lawyer Originally the onus was always on the Crown to show why the accused should be held in prison The Reform Act removed this presumption for accused individuals charged with specific offences Central criteria for judicial interim release are set out in s. 515(10) The presumption in s.515 is that the individual should be let out without any conditions, except if it is shown that conditions are necessary because of matters of: o Flight risk o Public safety o Public confidence on the administration of justice S. 515 (2) lists the types of bail orders that may made, depending on how onerous they are In terms of a money order that will need to be paid, the Code is silent as to its amount The onus is always on the Crown to show why a less restrictive means is not adequate

Intake Procedures
s. 503(1) Criminal Code Taking before justice A peace officer who arrests a person with or without warrant or to whom a person is delivered under subsection 494(3) or into whose custody a person is placed under subsection 163.5(3) of the Customs Act shall cause the person to be detained in custody and, in accordance with the following provisions, to be taken before a justice to be dealt with according to law: (a) where a justice is available within a period of twenty-four hours after the person has been arrested by or delivered to the peace officer, the person shall be taken before a justice without unreasonable delay and in any event within that period, and (b) where a justice is not available within a period of twenty-four hours after the person has been arrested by or delivered to the peace officer, the person shall be taken before a justice as soon as possible, unless, at any time before the expiration of the time prescribed in paragraph (a) or (b) for taking the person before a justice, (c) the peace officer or officer in charge releases the person under any other provision of this Part, or (d) the peace officer or officer in charge is satisfied that the person should be released from custody, whether unconditionally under subsection (4) or otherwise conditionally or 108

unconditionally, and so releases him. s. 504 Criminal Code In what cases justice may receive information Any one who, on reasonable grounds, believes that a person has committed an indictable offence may lay an information in writing and under oath before a justice, and the justice shall receive the information, where it is alleged (a) that the person has committed, anywhere, an indictable offence that may be tried in the province in which the justice resides, and that the person (i) is or is believed to be, or (ii) resides or is believed to reside, within the territorial jurisdiction of the justice; (b) that the person, wherever he may be, has committed an indictable offence within the territorial jurisdiction of the justice; (c) that the person has, anywhere, unlawfully received property that was unlawfully obtained within the territorial jurisdiction of the justice; or (d) that the person has in his possession stolen property within the territorial jurisdiction of the justice. s. 505 Criminal Code Time within which information to be laid in certain cases Where (a) an appearance notice has been issued to an accused under section 496, or (b) an accused has been released from custody under section 497 or 498, an information relating to the offence alleged to have been committed by the accused or relating to an included or other offence alleged to have been committed by him shall be laid before a justice as soon as practicable thereafter and in any event before the time stated in the appearance notice, promise to appear or recognizance issued to or given or entered into by the accused for his attendance in court. s. 506 Criminal Code Forms An information laid under section 504 or 505 may be in Form 2. s. 507 Criminal Code Justice to hear informant and witnesses public prosecutions (1) Subject to subsection 523(1.1), a justice who receives an information laid under section 504 by a peace officer, a public officer, the Attorney General or the Attorney Generals agent, other 109

than an information laid before the justice under section 505, shall, except if an accused has already been arrested with or without a warrant, (a) hear and consider, ex parte, (i) the allegations of the informant, and (ii) the evidence of witnesses, where he considers it desirable or necessary to do so; and (b) where he considers that a case for so doing is made out, issue, in accordance with this section, either a summons or a warrant for the arrest of the accused to compel the accused to attend before him or some other justice for the same territorial division to answer to a charge of an offence. Process compulsory (2) No justice shall refuse to issue a summons or warrant by reason only that the alleged offence is one for which a person may be arrested without warrant. Procedure when witnesses attend (3) A justice who hears the evidence of a witness pursuant to subsection (1) shall (a) take the evidence on oath; and (b) cause the evidence to be taken in accordance with section 540 in so far as that section is capable of being applied. Summons to be issued except in certain cases (4) Where a justice considers that a case is made out for compelling an accused to attend before him to answer to a charge of an offence, he shall issue a summons to the accused unless the allegations of the informant or the evidence of any witness or witnesses taken in accordance with subsection (3) discloses reasonable grounds to believe that it is necessary in the public interest to issue a warrant for the arrest of the accused. No process in blank (5) A justice shall not sign a summons or warrant in blank. Endorsement of warrant by justice (6) A justice who issues a warrant under this section or section 508 or 512 may, unless the offence is one mentioned in section 522, authorize the release of the accused pursuant to section 499 by making an endorsement on the warrant in Form 29. Promise to appear or recognizance deemed to have been confirmed (7) Where, pursuant to subsection (6), a justice authorizes the release of an accused pursuant to section 499, a promise to appear given by the accused or a recognizance entered into by the accused pursuant to that section shall be deemed, for the purposes of subsection 145(5), to have been confirmed by a justice under section 508. 110

Issue of summons or warrant (8) Where, on an appeal from or review of any decision or matter of jurisdiction, a new trial or hearing or a continuance or renewal of a trial or hearing is ordered, a justice may issue either a summons or a warrant for the arrest of the accused in order to compel the accused to attend at the new or continued or renewed trial or hearing. Subsection 4 shows that summons are preferred to a warrant of arrest unless necessary in the public interest o See also s. 515 s. 507.1 Criminal Code Referral when private prosecution (1) A justice who receives an information laid under section 504, other than an information referred to in subsection 507(1), shall refer it to a provincial court judge or, in Quebec, a judge of the Court of Quebec, or to a designated justice, to consider whether to compel the appearance of the accused on the information. Summons or warrant (2) A judge or designated justice to whom an information is referred under subsection (1) and who considers that a case for doing so is made out shall issue either a summons or warrant for the arrest of the accused to compel him or her to attend before a justice to answer to a charge of the offence charged in the information. Conditions for issuance (3) The judge or designated justice may issue a summons or warrant only if he or she (a) has heard and considered the allegations of the informant and the evidence of witnesses; (b) is satisfied that the Attorney General has received a copy of the information; (c) is satisfied that the Attorney General has received reasonable notice of the hearing under paragraph (a); and (d) has given the Attorney General an opportunity to attend the hearing under paragraph (a) and to cross-examine and call witnesses and to present any relevant evidence at the hearing. Appearance of Attorney General (4) The Attorney General may appear at the hearing held under paragraph (3)(a) without being deemed to intervene in the proceeding. Information deemed not to have been laid (5) If the judge or designated justice does not issue a summons or warrant under subsection (2), 111

he or she shall endorse the information with a statement to that effect. Unless the informant, not later than six months after the endorsement, commences proceedings to compel the judge or designated justice to issue a summons or warrant, the information is deemed never to have been laid. Information deemed not to have been laid proceedings commenced (6) If proceedings are commenced under subsection (5) and a summons or warrant is not issued as a result of those proceedings, the information is deemed never to have been laid. New evidence required for new hearing (7) If a hearing in respect of an offence has been held under paragraph (3)(a) and the judge or designated justice has not issued a summons or a warrant, no other hearings may be held under that paragraph with respect to the offence or an included offence unless there is new evidence in support of the allegation in respect of which the hearing is sought to be held. Subsections 507(2) to (8) to apply (8) Subsections 507(2) to (8) apply to proceedings under this section. Non-application informations laid under sections 810 and 810.1 (9) Subsections (1) to (8) do not apply in respect of an information laid under section 810 or 810.1. Definition of designated justice (10) In this section, designated justice means a justice designated for the purpose by the chief judge of the provincial court having jurisdiction in the matter or, in Quebec, a justice designated by the chief judge of the Court of Quebec. s. 508 Criminal Code Justice to hear informant and witnesses (1) A justice who receives an information laid before him under section 505 shall (a) hear and consider, ex parte, (i) the allegations of the informant, and (ii) the evidence of witnesses, where he considers it desirable or necessary to do so; (b) where he considers that a case for so doing is made out, whether the information relates to the offence alleged in the appearance notice, promise to appear or recognizance or to an included or other offence, (i) confirm the appearance notice, promise to appear or recognizance, as the case may be, and endorse the information accordingly, or (ii) cancel the appearance notice, promise to appear or recognizance, as the case 112

may be, and issue, in accordance with section 507, either a summons or a warrant for the arrest of the accused to compel the accused to attend before him or some other justice for the same territorial division to answer to a charge of an offence and endorse on the summons or warrant that the appearance notice, promise to appear or recognizance, as the case may be, has been cancelled; and (c) where he considers that a case is not made out for the purposes of paragraph (b), cancel the appearance notice, promise to appear or recognizance, as the case may be, and cause the accused to be notified forthwith of the cancellation. Procedure when witnesses attend (2) A justice who hears the evidence of a witness pursuant to subsection (1) shall (a) take the evidence on oath; and (b) cause the evidence to be taken in accordance with section 540 in so far as that section is capable of being applied. s. 515 Criminal Code Order of release (1) Subject to this section, where an accused who is charged with an offence other than an offence listed in section 469 is taken before a justice, the justice shall, unless a plea of guilty by the accused is accepted, order, in respect of that offence, that the accused be released on his giving an undertaking without conditions, unless the prosecutor, having been given a reasonable opportunity to do so, shows cause, in respect of that offence, why the detention of the accused in custody is justified or why an order under any other provision of this section should be made and where the justice makes an order under any other provision of this section, the order shall refer only to the particular offence for which the accused was taken before the justice. Release on undertaking with conditions, etc. (2) Where the justice does not make an order under subsection (1), he shall, unless the prosecutor shows cause why the detention of the accused is justified, order that the accused be released (a) on his giving an undertaking with such conditions as the justice directs; (b) on his entering into a recognizance before the justice, without sureties, in such amount and with such conditions, if any, as the justice directs but without deposit of money or other valuable security; (c) on his entering into a recognizance before the justice with sureties in such amount and with such conditions, if any, as the justice directs but without deposit of money or other valuable security; (d) with the consent of the prosecutor, on his entering into a recognizance before the justice, without sureties, in such amount and with such conditions, if any, as the justice directs and on his depositing with the justice such sum of money or other valuable security as the justice directs; or (e) if the accused is not ordinarily resident in the province in which the accused is in custody or does not ordinarily reside within two hundred kilometres of the place in which he is in custody, on his entering into a recognizance before the justice with or without 113

sureties in such amount and with such conditions, if any, as the justice directs, and on his depositing with the justice such sum of money or other valuable security as the justice directs. Power of justice to name sureties in order (2.1) Where, pursuant to subsection (2) or any other provision of this Act, a justice, judge or court orders that an accused be released on his entering into a recognizance with sureties, the justice, judge or court may, in the order, name particular persons as sureties. Alternative to physical presence (2.2) Where, by this Act, the appearance of an accused is required for the purposes of judicial interim release, the appearance shall be by actual physical attendance of the accused but the justice may, subject to subsection (2.3), allow the accused to appear by means of any suitable telecommunication device, including telephone, that is satisfactory to the justice. Where consent required (2.3) The consent of the prosecutor and the accused is required for the purposes of an appearance if the evidence of a witness is to be taken at the appearance and the accused cannot appear by closed-circuit television or any other means that allow the court and the accused to engage in simultaneous visual and oral communication. Idem (3) The justice shall not make an order under any of paragraphs (2)(b) to (e) unless the prosecution shows cause why an order under the immediately preceding paragraph should not be made. Conditions authorized (4) The justice may direct as conditions under subsection (2) that the accused shall do any one or more of the following things as specified in the order: (a) report at times to be stated in the order to a peace officer or other person designated in the order; (b) remain within a territorial jurisdiction specified in the order; (c) notify the peace officer or other person designated under paragraph (a) of any change in his address or his employment or occupation; (d) abstain from communicating, directly or indirectly, with any victim, witness or other person identified in the order, or refrain from going to any place specified in the order, except in accordance with the conditions specified in the order that the justice considers necessary; (e) where the accused is the holder of a passport, deposit his passport as specified in the order; (e.1) comply with any other condition specified in the order that the justice considers 114

necessary to ensure the safety and security of any victim of or witness to the offence; and (f) comply with such other reasonable conditions specified in the order as the justice considers desirable. Condition prohibiting possession of firearms, etc. (4.1) When making an order under subsection (2), in the case of an accused who is charged with (a) an offence in the commission of which violence against a person was used, threatened or attempted, (a.1) a terrorism offence, (b) an offence under section 264 (criminal harassment), (b.1) an offence under section 423.1 (intimidation of a justice system participant), (c) an offence relating to the contravention of subsection 5(3) or (4), 6(3) or 7(2) of the Controlled Drugs and Substances Act, (d) an offence that involves, or the subject-matter of which is, a firearm, a cross-bow, a prohibited weapon, a restricted weapon, a prohibited device, ammunition, prohibited ammunition or an explosive substance, or (e) an offence under subsection 20(1) of the Security of Information Act, or an offence under subsection 21(1) or 22(1) or section 23 of that Act that is committed in relation to on offence under subsection 20(1) of that Act, the justice shall add to the order a condition prohibiting the accused from possessing a firearm, cross-bow, prohibited weapon, restricted weapon, prohibited device, ammunition, prohibited ammunition or explosive substance, or all those things, until the accused is dealt with according to law unless the justice considers that such a condition is not required in the interests of the safety of the accused or the safety and security of a victim of the offence or of any other person. Surrender, etc. (4.11) Where the justice adds a condition described in subsection (4.1) to an order made under subsection (2), the justice shall specify in the order the manner and method by which (a) the things referred to in subsection (4.1) that are in the possession of the accused shall be surrendered, disposed of, detained, stored or dealt with; and (b) the authorizations, licences and registration certificates held by the person shall be surrendered. Reasons (4.12) Where the justice does not add a condition described in subsection (4.1) to an order made under subsection (2), the justice shall include in the record a statement of the reasons for not adding the condition. Additional conditions (4.2) Before making an order under subsection (2), in the case of an accused who is charged with 115

an offence referred to in subsection (4.3), the justice shall consider whether it is desirable, in the interests of the safety and security of any person, particularly a victim of or witness to the offence or a justice system participant, to include as a condition of the order (a) that the accused abstain from communicating, directly or indirectly, with any victim, witness or other person identified in the order, or refrain from going to any place specified in the order; or (b) that the accused comply with any other condition specified in the order that the justice considers necessary to ensure the safety and security of those persons. Offences (4.3) The offences for the purposes of subsection (4.2) are (a) a terrorism offence; (b) an offence described in section 264 or 423.1; (c) an offence in the commission of which violence against a person was used, threatened or attempted; and (d) an offence under subsection 20(1) of the Security of Information Act, or an offence under subsection 21(1) or 22(1) or section 23 of that Act that is committed in relation to an offence under subsection 20(1) of that Act. Detention in custody (5) Where the prosecutor shows cause why the detention of the accused in custody is justified, the justice shall order that the accused be detained in custody until he is dealt with according to law and shall include in the record a statement of his reasons for making the order. Order of detention (6) Notwithstanding any provision of this section, where an accused is charged (a) with an indictable offence, other than an offence listed in section 469, (i) that is alleged to have been committed while at large after being released in respect of another indictable offence pursuant to the provisions of this Part or section 679 or 680, (ii) that is an offence under section 467.11, 467.12 or 467.13, or a serious offence alleged to have been committed for the benefit of, at the direction of, or in association with, a criminal organization, (iii) that is an offence under any of sections 83.02 to 83.04 and 83.18 to 83.23 or otherwise is alleged to be a terrorism offence, (iv) an offence under subsection 16(1) or (2), 17(1), 19(1), 20(1) or 22(1) of the Security of Information Act, or (v) an offence under subsection 21(1) or 22(1) or section 23 of the Security of Information Act that is committed in relation to on offence referred to in subparagraph (iv), (b) with an indictable offence, other than an offence listed in section 469 and is not ordinarily resident in Canada, (c) with an offence under any of subsections 145(2) to (5) that is alleged to have been 116

committed while he was at large after being released in respect of another offence pursuant to the provisions of this Part or section 679, 680 or 816, or (d) with having committed an offence punishable by imprisonment for life under subsection 5(3), 6(3) or 7(2) of the Controlled Drugs and Substances Act or the offence of conspiring to commit such an offence, the justice shall order that the accused be detained in custody until he is dealt with according to law, unless the accused, having been given a reasonable opportunity to do so, shows cause why his detention in custody is not justified, but where the justice orders that the accused be released, he shall include in the record a statement of his reasons for making the order. Order of release (7) Where an accused to whom paragraph 6(a), (c) or (d) applies shows cause why the accuseds detention in custody is not justified, the justice shall order that the accused be released on giving an undertaking or entering into a recognizance described in any of paragraphs (2)(a) to (e) with the conditions described in subsections (4) to (4.2) or, where the accused was at large on an undertaking or recognizance with conditions, the additional conditions described in subsections (4) to (4.2), that the justice considers desirable, unless the accused, having been given a reasonable opportunity to do so, shows cause why the conditions or additional conditions should not be imposed. Idem (8) Where an accused to whom paragraph (6)(b) applies shows cause why the accuseds detention in custody is not justified, the justice shall order that the accused be released on giving an undertaking or entering into a recognizance described in any of paragraphs (2)(a) to (e) with the conditions, described in subsections (4) to (4.2), that the justice considers desirable. Sufficiency of record (9) For the purposes of subsections (5) and (6), it is sufficient if a record is made of the reasons in accordance with the provisions of Part XVIII relating to the taking of evidence at preliminary inquiries. Justification for detention in custody (10) For the purposes of this section, the detention of an accused in custody is justified only on one or more of the following grounds: (a) where the detention is necessary to ensure his or her attendance in court in order to be dealt with according to law; (b) where the detention is necessary for the protection or safety of the public, including any victim of or witness to the offence, having regard to all the circumstances including any substantial likelihood that the accused will, if released from custody, commit a criminal offence or interfere with the administration of justice; and (c) on any other just cause being shown and, without limiting the generality of the 117

foregoing, where the detention is necessary in order to maintain confidence in the administration of justice, having regard to all the circumstances, including the apparent strength of the prosecutions case, the gravity of the nature of the offence, the circumstances surrounding its commission and the potential for a lengthy term of imprisonment. Detention in custody for offence listed in section 469 (11) Where an accused who is charged with an offence mentioned in section 469 is taken before a justice, the justice shall order that the accused be detained in custody until he is dealt with according to law and shall issue a warrant in Form 8 for the committal of the accused. Order re no communication (12) A justice who orders that an accused be detained in custody under this section may include in the order a direction that the accused abstain from communicating, directly or indirectly, with any victim, witness or other person identified in the order, except in accordance with such conditions specified in the order as the justice considers necessary. The offences listed under s. 469 include treason, intimidating Parliament, and murder o Subsection 10 requires custody when an accused is charged with these o The procedure under s. 522 stipulates when an accused may be released when charged with these types of offences Subsection 6 reverses the onus and forces the defendant to justify release if the indictable offence (other than those listed in s. 469) was committed within the context of certain circumstances s. 516 Criminal Code Remand in custody (1) A justice may, before or at any time during the course of any proceedings under section 515, on application by the prosecutor or the accused, adjourn the proceedings and remand the accused to custody in prison by warrant in Form 19, but no adjournment shall be for more than three clear days except with the consent of the accused. Detention pending bail hearing (2) A justice who remands an accused to custody under subsection (1) or subsection 515(11) may order that the accused abstain from communicating, directly or indirectly, with any victim, witness or other person identified in the order, except in accordance with any conditions specified in the order that the justice considers necessary. s. 517 Criminal Code Order directing matters not to be published for specified period (1) If the prosecutor or the accused intends to show cause under section 515, he or she shall so state to the justice and the justice may, and shall on application by the accused, before or at any 118

time during the course of the proceedings under that section, make an order directing that the evidence taken, the information given or the representations made and the reasons, if any, given or to be given by the justice shall not be published in any document, or broadcast or transmitted in any way before such time as (a) if a preliminary inquiry is held, the accused in respect of whom the proceedings are held is discharged; or (b) if the accused in respect of whom the proceedings are held is tried or ordered to stand trial, the trial is ended. Failure to comply (2) Every one who fails without lawful excuse, the proof of which lies on him, to comply with an order made under subsection (1) is guilty of an offence punishable on summary conviction. s. 518 Criminal Code Inquiries to be made by justice and evidence (1) In any proceedings under section 515, (a) the justice may, subject to paragraph (b), make such inquiries, on oath or otherwise, of and concerning the accused as he considers desirable; (b) the accused shall not be examined by the justice or any other person except counsel for the accused respecting the offence with which the accused is charged, and no inquiry shall be made of the accused respecting that offence by way of crossexamination unless the accused has testified respecting the offence; (c) the prosecutor may, in addition to any other relevant evidence, lead evidence (i) to prove that the accused has previously been convicted of a criminal offence, (ii) to prove that the accused has been charged with and is awaiting trial for another criminal offence, (iii) to prove that the accused has previously committed an offence under section 145, or (iv) to show the circumstances of the alleged offence, particularly as they relate to the probability of conviction of the accused; (d) the justice may take into consideration any relevant matters agreed on by the prosecutor and the accused or his counsel; (d.1) the justice may receive evidence obtained as a result of an interception of a private communication under and within the meaning of Part VI, in writing, orally or in the form of a recording and, for the purposes of this section, subsection 189(5) does not apply to that evidence; (d.2) the justice shall take into consideration any evidence submitted regarding the need to ensure the safety or security of any victim of or witness to an offence; and (e) the justice may receive and base his decision on evidence considered credible or trustworthy by him in the circumstances of each case. Release pending sentence (2) Where, before or at any time during the course of any proceedings under section 515, the 119

accused pleads guilty and that plea is accepted, the justice may make any order provided for in this Part for the release of the accused until the accused is sentenced. Subsection 1 addresses the procedure during the bail hearing Subscetion2 discusses the issue of release pending evidence s. 519 Criminal Code Release of accused (1) Where a justice makes an order under subsection 515(1), (2), (7) or (8), (a) if the accused thereupon complies with the order, the justice shall direct that the accused be released (i) forthwith, if the accused is not required to be detained in custody in respect of any other matter, or (ii) as soon thereafter as the accused is no longer required to be detained in custody in respect of any other matter; and (b) if the accused does not thereupon comply with the order, the justice who made the order or another justice having jurisdiction shall issue a warrant for the committal of the accused and may endorse thereon an authorization to the person having the custody of the accused to release the accused when the accused complies with the order (i) forthwith after the compliance, if the accused is not required to be detained in custody in respect of any other matter, or (ii) as soon thereafter as the accused is no longer required to be detained in custody in respect of any other matter and if the justice so endorses the warrant, he shall attach to it a copy of the order. Discharge from custody (2) Where the accused complies with an order referred to in paragraph (1)(b) and is not required to be detained in custody in respect of any other matter, the justice who made the order or another justice having jurisdiction shall, unless the accused has been or will be released pursuant to an authorization referred to in that paragraph, issue an order for discharge in Form 39. Warrant for committal (3) Where the justice makes an order under subsection 515(5) or (6) for the detention of the accused, he shall issue a warrant for the committal of the accused. s. 520 Criminal Code Review of order (1) If a justice, or a judge of the Nunavut Court of Justice, makes an order under subsection 515(2), (5), (6), (7), (8) or (12) or makes or vacates any order under paragraph 523(2)(b), the accused may, at any time before the trial of the charge, apply to a judge for a review of the order. Notice to prosecutor 120

(2) An application under this section shall not, unless the prosecutor otherwise consents, be heard by a judge unless the accused has given to the prosecutor at least two clear days notice in writing of the application. Accused to be present (3) If the judge so orders or the prosecutor or the accused or his counsel so requests, the accused shall be present at the hearing of an application under this section and, where the accused is in custody, the judge may order, in writing, the person having the custody of the accused to bring him before the court. Adjournment of proceedings (4) A judge may, before or at any time during the hearing of an application under this section, on application by the prosecutor or the accused, adjourn the proceedings, but if the accused is in custody no adjournment shall be for more than three clear days except with the consent of the accused. Failure of accused to attend (5) Where an accused, other than an accused who is in custody, has been ordered by a judge to be present at the hearing of an application under this section and does not attend the hearing, the judge may issue a warrant for the arrest of the accused. Execution (6) A warrant issued under subsection (5) may be executed anywhere in Canada. Evidence and powers of judge on review (7) On the hearing of an application under this section, the judge may consider (a) the transcript, if any, of the proceedings heard by the justice and by any judge who previously reviewed the order made by the justice, (b) the exhibits, if any, filed in the proceedings before the justice, and (c) such additional evidence or exhibits as may be tendered by the accused or the prosecutor, and shall either (d) dismiss the application, or (e) if the accused shows cause, allow the application, vacate the order previously made by the justice and make any other order provided for in section 515 that he considers is warranted. Limitation of further applications

121

(8) Where an application under this section or section 521 has been heard, a further or other application under this section or section 521 shall not be made with respect to that same accused, except with leave of a judge, prior to the expiration of thirty days from the date of the decision of the judge who heard the previous application. Application of sections 517, 518 and 519 (9) The provisions of sections 517, 518 and 519 apply with such modifications as the circumstances require in respect of an application under this section. s. 521 Criminal Code Review of order (1) If a justice, or a judge of the Nunavut Court of Justice, makes an order under subsection 515(1), (2), (7), (8) or (12) or makes or vacates any order under paragraph 523(2)(b), the prosecutor may, at any time before the trial of the charge, apply to a judge for a review of the order. Notice to accused (2) An application under this section shall not be heard by a judge unless the prosecutor has given to the accused at least two clear days notice in writing of the application. Accused to be present (3) If the judge so orders or the prosecutor or the accused or his counsel so requests, the accused shall be present at the hearing of an application under this section and, where the accused is in custody, the judge may order, in writing, the person having the custody of the accused to bring him before the court. Adjournment of proceedings (4) A judge may, before or at any time during the hearing of an application under this section, on application of the prosecutor or the accused, adjourn the proceedings, but if the accused is in custody no adjournment shall be for more than three clear days except with the consent of the accused. Failure of accused to attend (5) Where an accused, other than an accused who is in custody, has been ordered by a judge to be present at the hearing of an application under this section and does not attend the hearing, the judge may issue a warrant for the arrest of the accused. Warrant for detention (6) Where, pursuant to paragraph (8)(e), the judge makes an order that the accused be detained in 122

custody until he is dealt with according to law, he shall, if the accused is not in custody, issue a warrant for the committal of the accused. Execution (7) A warrant issued under subsection (5) or (6) may be executed anywhere in Canada. Evidence and powers of judge on review (8) On the hearing of an application under this section, the judge may consider (a) the transcript, if any, of the proceedings heard by the justice and by any judge who previously reviewed the order made by the justice, (b) the exhibits, if any, filed in the proceedings before the justice, and (c) such additional evidence or exhibits as may be tendered by the prosecutor or the accused, and shall either (d) dismiss the application, or (e) if the prosecutor shows cause, allow the application, vacate the order previously made by the justice and make any other order provided for in section 515 that he considers to be warranted. Limitation of further applications (9) Where an application under this section or section 520 has been heard, a further or other application under this section or section 520 shall not be made with respect to the same accused, except with leave of a judge, prior to the expiration of thirty days from the date of the decision of the judge who heard the previous application. Application of sections 517, 518 and 519 (10) The provisions of sections 517, 518 and 519 apply with such modifications as the circumstances require in respect of an application under this section. s. 522 Criminal Code Interim release by judge only (1) Where an accused is charged with an offence listed in section 469, no court, judge or justice, other than a judge of or a judge presiding in a superior court of criminal jurisdiction for the province in which the accused is so charged, may release the accused before or after the accused has been ordered to stand trial. Idem (2) Where an accused is charged with an offence listed in section 469, a judge of or a judge 123

presiding in a superior court of criminal jurisdiction for the province in which the accused is charged shall order that the accused be detained in custody unless the accused, having been given a reasonable opportunity to do so, shows cause why his detention in custody is not justified within the meaning of subsection 515(10). Order re no communication (2.1) A judge referred to in subsection (2) who orders that an accused be detained in custody under this section may include in the order a direction that the accused abstain from communicating, directly or indirectly, with any victim, witness or other person identified in the order except in accordance with such conditions specified in the order as the judge considers necessary. Release of accused (3) If the judge does not order that the accused be detained in custody under subsection (2), the judge may order that the accused be released on giving an undertaking or entering into a recognizance described in any of paragraphs 515(2)(a) to (e) with such conditions described in subsections 515(4), (4.1) and (4.2) as the judge considers desirable. Order not reviewable except under section 680 (4) An order made under this section is not subject to review, except as provided in section 680. Application of sections 517, 518 and 519 (5) The provisions of sections 517, 518 except subsection (2) thereof, and 519 apply with such modifications as the circumstances require in respect of an application for an order under subsection (2). Other offences (6) Where an accused is charged with an offence mentioned in section 469 and with any other offence, a judge acting under this section may apply the provisions of this Part respecting judicial interim release to that other offence. The offences listed under s. 469 include treason, intimidating Parliament, and murder o Subsection 2 lays down the procedure under which an accused may be released when charged with these types of offences (satisfying the reversed onus imposed on her) s. 795 Criminal Code Application of Parts XVI, XVIII, XX and XX.1 The provisions of Parts XVI and XVIII with respect to compelling the appearance of an accused before a justice, and the provisions of Parts XX and XX.1, in so far as they are not inconsistent 124

with this Part, apply, with such modifications as the circumstances require, to proceedings under this Part. This section appears under Part XXVII: Summary Convictions

Show cause hearings


The new legislations emphasis on release is nowhere more evident than in s. 515 Previously the accused would have to petition for her release Now the legislation requires that the accused should be release without conditions, unless the prosecutor shows cause why the release should not be ordered o The section follows a ladder effect R. v. Thompson (1972), 18 CRNS 102 (BC SC) Facts The accused lost control of his car and it collided with a tree Issue: Did the roadside alcohol test constitute detention under s. 10 of the Charter? Holding: Yes; appeal dismissed Anderson J Whereas under the old system the onus was on an accused to show that he was entitled to be released on bail, the new legislation places on the Crown the burden of justifying the detention of an accused The legislation as a whole is to be liberally interpreted so as to carry out the intention of Parliament The fact that a person has no roots in the community is not, of itself, a sufficient ground for denying a release without conditions, nor is the question of deterrence relevant in considering release pending trial A justice who makes an order refusing to release an accused is required to give his reasons therefor, both for the protection of the accused and for the assistance of a reviewing court Lecture Notes At common law, only flight risk and public safety were reasons for denying bail The third ground in Canada is anomalous amongst developed nations o Does not exist in the UK or the US Before 1967, there were two grounds o Compelling appearance o Public interest It can then be argued that the third ground that exists now derives from this second former legal ground

125

Grounds for Detention


Re Powers and R. (1973), 20 CRNS 23 (Ont. HC) Lerner J Under s. 457(7), detention for the purpose of ensuring attendance in court includes consideration of such things as residence, fixed place of abode, employment or occupation, marital and family status, previous criminal record, proximity of close friends and relatives, character witnesses, facts relating to the allegations of the offence, personal history or vitae In the instant case the accused was alleged to have committed a series of drug offences while on bail such that it could not be said that she would likely commit an offence involving serious harm or interference with the administration of justice The expression, "detention is necessary in the public interest", in s. 457(7)(b), is a separate ground Public interest" includes the "public image" of the Criminal Code, the bail amendments, the apprehension and conviction of criminals, deterrence of crime and public protection The application of the bail provisions should not appear to be a mockery The public must be allowed to feel safe and secure The plain lessons of human experience must be recognized and applied to these bail procedures. Lectures notes The court interprets public interest as something including deterrence and people not feeling that the application of our criminal laws are a mockery R. v. Graham, Ont. Dist. Ct., August 2 1990 Fleury J In our modern society, where all sectors have become aware of the prevalence of sexual abuse, it is clearly in the public interest to discourage any type of sexual abuse The context of reasonable bail as outlined in the Charter must be understood in the context of the present provisions of the Criminal Code It is clear that there are circumstances where there is no substantial likelihood of the commission of a criminal offence which might still justify a detention order in the public interest The principles outlined in Powers have not changed despite the advent of the Charter Lectures notes Here the court interprets public interest as something based on how much the particular offence is considered a social problem R. v. Lamothe (1990), 77 CR (3d) 236 126

Facts The accused was charged with complicity after the fact in a case of murder Under s. 522 of the Code, he had to convince the court that his detention was not required for either of the reasons in s. 515(10) The Superior Court was satisfied that his detention was not required to assure his presence in court, but concluded that he should be detained because of the probability that he would be convicted of a very serious crime The accused appealed. Issue: Does the seriousness of the crime justify a detention in the public interest? Holding: No; appeal allowed Baudoin J In applying s. 515(10) a judge has a wide discretionary power because of the importance of the circumstances of each case in the decision-making process However, this discretionary power must be exercised judicially, taking into account the fundamental principles of criminal law, and particularly the rights guaranteed by the Charter The right to be presumed innocent should not be considered only to decide the question of guilt or innocence The same right applies to s. 520, 521 or 522, and does not vary according to the nature of the crime charged Even if s. 522 shifts the burden of proof, the accused is still presumed innocent Perron authoritatively defined the notion of public interest in that context o That criterion implies two aspects: the perception of the public if the accused is released from custody, and its reaction The perception of the public is often negative, sometimes emotionally so, towards criminals However, the criminal law has an educative goal to achieve The judge must take into account the perception of the INFORMED PUBLIC who understand that the right to be presumed innocent at every stage of the criminal process is not a theoretical notion, but is the price to pay for living in a free and democratic society The judge should also take into account the reaction the informed public would have if the accused were released from custody before trial Would that release discredit the administration of justice? Lectures notes The court recognizes that it would not want to be entertaining public views when they are not aware of the purpose of the Bail Reform Act and that bail should not be use as a means of denunciation The court lists some criteria which could be relevant to interpreting what the reasonable response of the public may be o As for recidivism, what is at stake here is the public perception of recidivism and not public safety 127

s. 11(e) Canadian Charter of Rights and Freedoms Proceedings in criminal and penal matters Any person charged with an offence has the right ... e) not to be denied reasonable bail without just cause R. v. Morales, [1992] 3 SCR 711 Issues: 1. Did the criterion of public interest as a basis for pre-trial detention violate s. 11(e) of the Charter? 2. Is the public safety component unconstitutional? Holding: 1.Yes 2. No; appeal allowed Lamer CJC The criterion of "public interest" violates s. 11(e) of the Charter because it authorizes detention in terms which are vague and imprecise As per R. v. Nova Scotia Pharmaceutical Society, the doctrine of vagueness is a consideration both as a principle of fundamental justice under s. 7 and in applying s. 1 It is a principle of fundamental justice that laws may not be too vague Vagueness can also be raised under s. 1 of the Charter in limine, on the basis that an enactment is so vague as not to satisfy the requirement that a limitation on Charter rights be "prescribed by law" A law will be found unconstitutionally vague if it so lacks in precision as not to give sufficient guidance for legal debate There cannot be just cause for denial of bail within the meaning of s. 11(e) if the statutory criteria for denying bail are vague and imprecise Nova Scotia Pharmaceutical Society identified two rationales for the doctrine of vagueness, namely fair notice to the citizen and limitation of law enforcement discretion While fair notice is not relevant to a provision like s. 515(10)(b) which does not prohibit conduct, limitation of law enforcement discretion is still a relevant factor o Is the statute so pervasively vague that it permits a standardless sweep allowing law enforcement officials to pursue their personal predilections? Since pre-trial detention is extraordinary in our system of criminal justice, vagueness in defining the terms of pre-trial detention may be even more invidious than is vagueness in defining an offence A provision does not violate the doctrine of vagueness simply because it is subject to interpretation Nevertheless, the term "public interest" is incapable of framing the legal debate in any meaningful manner or structuring discretion in any way 128

Nor would it be possible to give the term "public interest" a constant or settled meaning

This violation is not justified under s. 1 Even if the term "public interest" is capable of passing the threshold test under s. 1 of being a limit which is "prescribed by law", it cannot be justified under the Oakes test o It can be justified under the first branch of the Oakes test given its objectives: 1) prevent those who have been arrested from committing criminal offences; 2) prevent those who have been arrested from interfering with the administration of justice o However, it does not meet the proportionality branch of the Oakes test First, there is no rational connection between the measure and the objectives of preventing crime and preventing interference with the administration of justice Second, the measure does not impair rights as little as possible Third, there is no proportionality between the effects of the measure and its objectives Gonthier J (dissenting in part) It is first significant to recognize the general sense of the phrase public interest, which is a reference to the special set of values which are best understood from the point of view of the aggregate good and are of relevance to matters relating to the well-being of society The Charter is an expression of the fundamental values which direct the public interest The concept of public interest is indeed broad but it is not meaningless, nor is it vague At issue are the reasons for detention, for which the Charter requires a just cause This implies two elements: (1) a cause or reason and (2) proportionality between the reason and the deprivation of liberty that makes the cause "just" Under s. 515(10), two main elements exist in relation to the operation of the public interest criterion One of them is a requirement of necessity The other is the content of the considerations which may be included within the public interest criterion o The concept of public interest is broader than that of protection or safety of the public, and includes interests which may not be properly included within the categories of public health or safety o The aim of avoiding interference with the administration of justice is one such example o Other examples of a public interest could be the protection of the accused from suicide or from the actions of others, the prevention of activities which involve the possession of or dealing 129

in small quantities of illegal narcotics, or the preparation of reports for the court which require the presence of the accused o Public interest is capable of encompassing circumstances which have not been foreseen One must be wary of using the doctrine of vagueness to prevent or impede State action in furtherance of valid social objectives, by requiring the law to achieve a degree of precision to which the subject-matter does not lend itself A delicate balance must be maintained between societal interests and individual rights As stated by Lamer CJC flexibility and vagueness are not synonymous The former is a quality necessary to the administration of justice and s.515(10)(b) provides for this Its dual requirements of public interest and necessity which itself predicates a public interest of a serious nature have meaning, give rise to legal debate and, though broad, are not vague but provide an adequate framework and limit for the exercise of judicial discretion Lectures notes This is the first time that the court carefully considered what the purpose of the former grounds for pre-trial detention was, in particular the secondary ground in the old legislation The purpose of the secondary ground according to Lamer J was to prevent those being arrested from interfering with the administration of justice One thing in which the majority and minority seem to agree is that deterrence and denunciation does not play a role in determining bail Five years after Morales Parliament amended the Criminal Code, including an amendment to s. 515 S. 515(10) now consists of three circumstances which justify the pre-trial detention of the accused o Flight risk o Public safety o Maintaining public confidence in the administration of justice This third criteria, however, includes the words on any other just cause which was in fact deemed unconstitutional (see Hall) R. v. Hall (2002), 4 CR (6th) 197 Facts In 1999, a woman's body was found with 37 wounds to her hands, forearms, shoulder, neck and face. It also appeared that her assailant had tried to cut off her head The murder caused significant public concern and a general fear that a killer was at large Based on compelling evidence the accused was charged with first degree murder and applied for bail 130

The bail judge held that pre-trial detention was not necessary "to ensure ... attendance in court" nor for the "safety of the public" o He denied bail, however, under s. 515(10)(c) in order "to maintain confidence in the administration of justice" A superior court judge dismissed the accused's habeas corpus application challenging the constitutionality of s. 515(10)(c) The CA affirmed the decision Issue: Does s. 515(10)(c) violate s. 11(e) of the Charter? Holding: No; appeal dismissed McLachlin CJC The portion of s. 515(10)(c) permitting detention "on any other just cause being shown" is unconstitutional The impugned phrase confers an open-ended judicial discretion to refuse bail and is inconsistent with both s. 11(e) of the Charter and the presumption of innocence It is a fundamental principle of justice that an individual cannot be detained by virtue of a vague legal provision o Parliament must lay out narrow and precise circumstances in which bail can be denied The impugned phrase is not justified under s. 1 of the Charter as its generality impels its failure of the proportionality branch of the Oakes test The next phrase in s. 515(10)(c) "without limiting the generality of the foregoing" is also void since it only confirms the generality of the preceding phrase The balance of s. 515(10)(c), which authorizes the denial of bail in order "to maintain confidence in the administration of justice", is valid It provides a basis for denying bail not covered by s. 515(10)(a) and (b) Although the circumstances in which recourse to this ground may not arise frequently, when they do it is essential that a means of denying bail be available because public confidence is essential to the proper functioning of the bail system and the justice system as a whole Denial of bail "to maintain confidence in the administration of justice" having regard to the factors set out in s. 515(10)(c) complies with s. 11(e) of the Charter This ground is narrower and more precise than the old public interest ground which was struck down as vague in 1992 and provides an intelligible standard for debate and for the exercise of discretion The means chosen do not go further than necessary to achieve Parliament's purpose of maintaining public confidence in the bail system and the justice system as whole Parliament has hedged the provision with important safeguards: a judge can only deny bail if satisfied that in view of the four specified factors and related circumstances, a REASONABLE MEMBER of the community would be satisfied that denial of bail is necessary to maintain confidence in the administration of justice The provision is not overbroad but strikes an appropriate balance between the rights of 131

the accused and the need to maintain justice in the community. The appropriate remedy in this case is to sever the phrase "on any other just cause being shown, and without limiting the generality of the foregoing" The balance of s. 515(10)(c) can stand alone as a functioning whole without doing damage to Parliament's intention Iacobucci J (dissenting) At the heart of a free and democratic society is the liberty of its subjects and our justice system must minimize unwarranted denials of liberty In the criminal law context, this freedom is embodied generally in the right to be presumed innocent until proven guilty and specifically in the right to bail Fear that a bail judge will be unable to protect the public without s. 515(10)(c) is without reasonable foundation There is no evidence that the bail system was lacking in any way before the introduction of the provision in 1997 Section 515(10)(c) must be assessed as a whole Yet even if these components are considered independently, neither can withstand constitutional scrutiny It is impossible to hold that the phrase "any other just cause" provides for the denial of bail under a narrow set of circumstances The phrase is deliberately open-ended and is vaguer than the old "public interest" ground Nor does the "confidence in the administration of justice" component of s. 515(10)(c) provide a sufficiently precise standard The specific factors listed in s. 515(10)(c) provide little more than a facade of precision It is impermissibly vague because of the failure to establish a plausible and valid ground for denying bail that would serve the proper administration of the bail system and that is not already covered under the more specific grounds in s. 515(10)(a) and (b) Section 515(10)(c) essentially revives the old "public interest" ground and invokes similarly vague notions of the public image of the criminal justice system The bail judge erred in considering the subjective fears of the public after determining there was no risk of flight nor any threat to the public The problem with s. 515(10)(c) is that it allows the subjective fears of the public and illinformed emotional impulses extraneous to the bail system to form a sole basis for denying bail Section 515(10)(c) cannot be saved under s. 1 of the Charter It does not address a pressing issue and fails the proportionality tests The only available remedy is to strike down s. 515(10)(c) in its entirety 132

Although the section may encompass instances where bail could be denied without offending s. 11(e), the Court cannot sufficiently narrow the scope of that section without seriously intruding into the legislative sphere Even if the administration of justice portion of the provision were constitutional, s. 515(10)(c) could not be read down to include only this portion o The second component of s. 515(10)(c) is but an example of denying bail "on any other just cause" o To convert a single example of an explicitly open-ended rule into the rule itself substantially changes the significance of the provision and contravenes Parliament's clear intention not to limit the generality of "any other just cause" Lecture notes A criticism of the confidence in administration of justice criteria is that it adds many elements to the bail determination which go beyond the basic issues of public safety and flight risk o Proving these may require witnesses and a vast amount of evidence There seems to be a trend in the jurisprudence to use the third ground to deny bail (s. 515(10)(c)) sparingly For some examples of where this ground has been invoked see p.631 of textbook Most judgment have place emphasis on McLachlins acceptance that the reasonable person, in making an assessment of the need to maintain public confidence, must be properly informed about the philosophy of the legislative provision, Charter values and the actual circumstance of the case

3. The Judge Must Decide


Under the previous legislation, judges were to inquire into the sufficiency of sureties and make a decision based on the merits of the case Empirical studies showed that a decision-making process was not followed in most cases, but rather an inflexible procedure of cash or property in advanced based on the nature of the crime While the new provisions provide for judicial review on these matters, and the legislature has tried to be as specific as possible, the success of the system depends on whether or not it has support at the grass-roots levels Do judges under the new system make real decisions in determining bail on their own, or like before, they simply abide by the wishes of the police and prosecutors? R. v. Major (1990), 76 CR (3d) 104 (Ont. Dist. Ct.) Facts The judge consented to a detention order Judge Charron found this to be in error Charron DJC

133

When an accused is brought before a justice of the peace under s. 524, it is quite clear that the justice, of the peace has a duty to inquire into the alleged misconduct, make a finding in this respect and then make an appropriate order as set out in the section The facts relied upon by the Crown may well be presented by way of admission where the accused so consents but evidence must be heard in order to support a finding by the presiding justice An order must be based on such finding and cannot simply be based on a bare consent without more It is clear that the justice of the peace has a judicial function to perform in the determination of appropriate conditions and that function cannot be delegated to the prosecutor The justice of the peace must exercise his or her discretion as set out in the Code

4. Reasonable Conditions on Release


A judge must not impose so stringent condition on release, so that it amounts, in fact, to detention The conditions imposed must be reasonable See Re Keenan and R., 1979 o The conditions imposed must relate to the criteria for detention in s. 515(10 R. v. Anoussis (2008) Criminal Chamber - Court of Quebec Accused was charged with causing bodily harm The Crown was asking for conditions made on the judicial interim release o The first condition asked for was a cash deposit o Healy J felt that this was not required and encouraged defence counsels to raise objections to these conditions more often Cash deposits are meant to be exceptional The word bail was abandoned and replace with judicial interim release because of its pecuniary connotations One view, however, is that this provides the easiest way for the accused to be let out on interim release What if the accused agrees with the cash deposit? Should this override the ladder approach to determining conditions for judicial interim release? Healy J also talks about the unequal treatment that may result from making cash deposits the basis of judicial interim release While judges must intervene, there is a reason to respect the choices made by some particular defendants

5. Charter Standards

134

While one of the cornerstones of the Bail Reform Act of 1976 was to place the onus on the Crown to justify a detention, the legislature created a number of situations where the onus would be placed on the accused to justify release Pearson deals with one of these situations, where the accused is charged with trafficking drugs under the Controlled Drugs and Substances Act (then the Narcotic Control Act) R. v. Pearson, [1992] 3 SCR 665 Issue: Does s. 515(6)(d) violate ss. 7, 9, 11(d) or 11(e) of the Charter? Holding: No; appeal allowed Lamer CJC There is just cause for the denial of bail under s. 515(6)(d) Bail is denied only in a narrow set of circumstances S. 515(6)(d) applies only to a very small number of offences, all of which involve the distribution of narcotics The section does not deny bail for all persons who are charged with these offences, but rather denies bail only when these persons are unable to demonstrate that detention is not justified The denial of bail is necessary to promote the proper functioning of the bail system and is not undertaken for any purpose extraneous to the bail system S. 515(6)(d) establishes a set of special bail rules in circumstances where the normal bail process is incapable of functioning properly The unique characteristics of the offences subject to s. 515(6)(d) suggest that those offences are committed in a very different context than most other crimes o Most offences, unlike the trafficking of narcotics, are not committed systematically o There is also a marked danger that an accused charged with these offences will not appear for trial In the Court of Appeal, Proulx J.A. expressed concern about the scope of s. 515(6)(d) and he felt that it is inequitable to treat a person who distributes a few joints of marijuana in the same manner as a person running a sophisticated network to traffic cocaine While these concerns are legitimate, these lesser offenders will normally have no difficulty justifying their release and obtaining bail McLachlin J (dissenting in part) S. 515(6)(d) section is very broad It applies to everyone who commits the specified offences In addition, the offences are very broad 135

o As a result, s. 515(6)(d) catches not only large-scale drug dealers, but the friend who shares a joint of marijuana at a party or gives it to a friend for safekeeping o All trafficking is repugnant and hence criminally punishable, but when the issue is denial of bail, the different dangers associated with the different types of trafficking, may require different treatment While there may be "just cause" for denying bail to persons charged with serious, large-scale or commercial trafficking, the reasons offered by Lamer CJC do not apply to other traffickers His reasons for a just cause apply only to the organized commercial trafficker o The "small-time" trafficker may not be motivated by money or profit, nor be a participant in a "highly sophisticated commercial setting" o Neither is she very likely to flee justice S. 515(6)(d) is not "demonstrably justified" in a free and democratic society While avoiding repeat offences and escaping justice are important objectives, the section goes further than is necessary to achieve those objectives Other jurisdictions, like the United States and Australia, distinguish between major and minor trafficking Lecture notes The court seems to agree that the presumption of innocence in 11(d) of the Charter requires a burden on the Crown to justify pre-trial detention The Report of the Commission on Systemic Racism in the Ontario Criminal Justice System (1995) suggest that the practical effect of presuming detention for persons charged with trafficking narcotics results in racism and the disproportionate detention of small-scale offenders R. v. Cantave, Court o f Quebec Criminal Chamber September 9 2008 Man in possession of two loaded firearms and had been ordered to not be in possession of firearms before Healy J finds that pre-trial detention was justified under the second and third ground given the gravity of these types of offences

Disclosure and Discovery


Except for narrow exceptions, it is generally not obliged for the accused to disclose her case to the Crown The main focus is on the obligations for the Crown, justified as a matter of fairness There is a distinction between disclosure and production Disclosure cases involved whether the Crown properly shared the evidence in its possession 136

Production cases involve potentially relevant materials not in the possession of the Crown, but in the hands of so-called third parties

The charging instrument given the accused an outline of the case he will have to meet at trial An amendment to the Code in 1985 now provides for pre-hearing conferences s. 625.1 Criminal Code Pre-hearing conference (1) Subject to subsection (2), on application by the prosecutor or the accused or on its own motion, the court, or a judge of the court, before which, or the judge, provincial court judge or justice before whom, any proceedings are to be held may order that a conference between the prosecutor and the accused or counsel for the accused, to be presided over by the court, judge, provincial court judge or justice, be held prior to the proceedings to consider the matters that, to promote a fair and expeditious hearing, would be better decided before the start of the proceedings, and other similar matters, and to make arrangements for decisions on those matters. Mandatory pre-trial hearing for jury trials (2) In any case to be tried with a jury, a judge of the court before which the accused is to be tried shall, before the trial, order that a conference between the prosecutor and the accused or counsel for the accused, to be presided over by a judge of that court, be held in accordance with the rules of court made under sections 482 and 482.1 to consider any matters that would promote a fair and expeditious trial. .

Disclosure
Royal Commission on the Donald Marshall Jr. Prosecution While the courtroom is adversarial, the Crown prosecutor must make sure the criminal justice system is itself fair Disclosure Anything less than complete disclosure by the Crown falls short of decency and fair play This is linked to the fundamental justice of the accused to make full answer and defence The principle of fair trial requires that the accused receive all information know to the Crown that might reasonably be considered useful to the accused The Crown should have a positive continuing duty to make disclosure to the defence It is immaterial whether the defence fails to request disclosure It is not realistic to assume that prosecutor does not have an interest in the outcome of the trial An obligation to disclose all relevant information is not inconsistent with the adversarial 137

system Whether or not to disclose information should not be a decision left to the discretion of the prosecution o A statutory obligation to disclose is necessary The law exists to protect society against lawbreakers and to safeguard the liberty and freedom of individuals against the power of the State R. v. Stinchcombe, [1991] 3 SCR 326 Facts The accused, a lawyer, was charged with breach of trust, theft and fraud A former secretary of his was a Crown witness at the preliminary inquiry, where she gave evidence apparently favourable to the defence After the preliminary inquiry but prior to trial, the witness was interviewed by an RCMP officer and a tape-recorded statement was taken. Later, during the course of the trial, the witness was again interviewed by a police officer and a written statement taken Defence counsel was informed of the existence but not of the content of the statements His requests for disclosure were refused During the trial defence counsel learned conclusively that the witness would not be called by the Crown and sought an order that the witness be called or that the Crown disclose the contents of the statements to the defence The trial judge dismissed the application and the accused was eventually convicted of breach of trust and fraud The Court of Appeal affirmed the convictions without giving reasons Issue: Is there a constitutional right to disclosure by the Crown as per s. 7 of the Charter? Holding: Yes; appeal allowed and a new trial ordered Sopinka J The Crown has a legal duty to disclose all relevant information to the defence The fruits of the investigation which are in its possession are not the property of the Crown for use in securing a conviction but the property of the public to be used to ensure that justice is done The obligation to disclose is subject to a discretion with respect to the withholding of information and to the timing and manner of disclosure Crown counsel has a duty to respect the rules of privilege and to protect the identity of informers Discretion must also be exercised with respect to the relevance of information The Crown's discretion is reviewable by the trial judge, who should be guided by the general principle that information should not be withheld if there is a reasonable possibility that this will impair the right of the accused to make full answer and defence The absolute withholding of information which is relevant to the defence can only be justified on 138

the basis of the existence of a legal privilege which excludes the information from disclosure This privilege is reviewable on the ground that it is not a reasonable limit on the right to make full answer and defence in a particular case Counsel for the accused must bring to the trial judge's attention at the earliest opportunity any failure of the Crown to comply with its duty to disclose of which counsel becomes aware This will enable the trial judge to remedy any prejudice to the accused if possible and thus avoid a new trial Initial disclosure should occur before the accused is called upon to elect the mode of trial or plead Subject to the Crown's discretion, all relevant information must be disclosed, both that which the Crown intends to introduce into evidence and that which it does not, and whether the evidence is inculpatory or exculpatory All statements obtained from persons who have provided relevant information to the authorities should be produced, even if they are not proposed as Crown witnesses Where statements are not in existence, other information such as notes should be produced o If there are no notes, all information in the prosecution's possession relating to any relevant evidence the person could give should be supplied Crown counsel was not justified in refusing disclosure here on the ground that the witness was not worthy of credit: whether the witness is credible is for the trial judge to determine after hearing the evidence The trial judge ought to have examined the statements Since the information withheld might have affected the outcome of the trial, a new trial was ordered A 2002 amendment to the Code requires defence counsel to give notice of an expert it intends to call, plus once the Crown has closed its case, a report on which the opinion will be based The Stinchcombe duty on the Crown is not absolute It is only triggered on request and subject to Crown discretion not to disclose irrelevant material or that which is subject to privilege

Production of Third Party Records


These new principles of disclosure have been highly controversial in cases of sexual assault or when medical records are involved In L.L.A. v. A.B. (1995), which was handed down with OConnor, the SCC unanimously decided that production should not be determined by class or case-by-case privilege (resembling the Wigmore criteria)\ The better approach was one of balancing competing Charter rights

139

R. v. OConnor, [1995] 4 SCR 411 Facts A bishop was charged with a number of sexual offences The charges were stayed when it became apparent that the Crown had not fully complied with a sweeping pre-trial order to disclose which included the information regarding the identity of psychiatrists or counsellors who had treated the complainants with respect to the allegations of sexual abuse, as well as the records of these therapy sessions Issue: Was there an obligation to disclose third party records from the complainants therapy? Holding: Yes; appeal allowed and a new trial ordered L'Heureux-Dub , La Forest, Gonthier and McLachlin saw the need to balance the accuseds right to a fair trial and full answer and defence with the complainants right to privacy and to equality without discrimination The majority determined that the accuseds right should be balanced with the complainants right to privacy under ss. T and 8 of the Charter The Court agreed that there should be a two-stage procedure but was divided 5-4 as to the precise test The majority decided that when the defence seeks information in the hands of a third party the onus should be on the accused to satisfy the judge that the information is likely to be relevant 1. In order to initiate the procedure the accused must bring an application setting out the grounds for production (this formal application can be waived by the court) Notice must be given to the third party in possession of these documents The meaning of relevance, in the context of disclosure, is whether the information might be useful to the defence o In the context of production the test for relevance should be higher, the trial judge must be satisfied that there is a REASONABLE possibility that the information is logically probable to an issue at trial or the competence of a witness o The majority disagreed with L'Heureux-Dub, however, that such records will only be relevant in rare cases o The majority gave an example of the therapy being relevant as to how the complaint unfolded, the use of therapy to influence memory and information bearing on credibility 2. Upon production of the documents, the judge should examine the records and determine whether, and to what extent, they should be produced to the accused Here, the judge must weigh the valuable and deleterious effects of a production order and consider whether non-production would constitute a reasonable limit on the ability of the accused to make full answer and defence For the minority, at the second stage of deciding whether the records should be produced to the accused, the court needed to decide in a manner that was respectful to Charter values of the right to privacy and equality 140

The majority was also on the view that the Crowns disclosure obligation in Stinchcombe was unchanged by the nature of the records where the records where in possession of the Crown LOST EVIDENCE CASES R.v. Carosella (1997) Since the Stinchcombe and OConnor was met, a stay should be granted as remedy for the violation of the accuseds right to make a full answer and defence as per s. 7 of the Charter R. v. La; R. v. Vu Seems like the SCC took a different approach when evidence was lost rather than destroyed When the evidence is lost, the Crown has a duty to explain the loss o When the court is satisfied that the evidence was not lost due to unacceptable negligence, the duty to disclose has not been breached As the relevance of evidence increases, so does the degree of care for its preservation A stay may be appropriate under exceptional circumstances where the loss of evidence is so prejudicial to the right to make full answer and defence BILL C-46 Following OConnor, Parliament passed legislation to restrict the production of records in sexual offence proceedings This is contained in ss. 278.1-278.9 The legislation largely reflects the minority position in OConnor This reform was held to be constitutional in Miills R. v. Mills, [1999] 3 SCR 668 Issue: Is the restriction to disclose the production of records in sexual offence proceedings constitutional? Holding: Yes McLachlin and Iacobucci JJ To challenge the constitutionality of the impugned legislation, the accused need not prove that the legislation would probably violate his right to make full answer and defence It is sufficient that he establish that the legislation is unconstitutional in its general effect Although the procedure governing the production of private records of complainants in sexual assault proceedings set out in Bill C-46 differs significantly from the OConnor regime, it does not follow that Bill C-46 is unconstitutional A posture of respect towards Parliament has been adopted by the courts and the relationship between the courts and the legislature should be one of dialogue 141

The courts do not hold a monopoly on the protection and promotion of rights and freedoms

Two principles of fundamental justice seem to conflict: the right to full answer and defence and the right to privacy Neither right may be defined in such a way as to negate the other and both sets of rights are informed by the equality rights at play in this context A contextual approach to the interpretation of rights should be adopted as they often inform, and are informed by, other rights at issue in the circumstances The right of an accused to make full answer and defence is crucial to ensuring that the innocent are not convicted The accuseds right to make full answer and defence must be understood in light of other principles of fundamental justice which may embrace interests and perspectives beyond those of the accused In this context, full answer and defence does not include the right to evidence that would distort the search for truth inherent in the trial process. An order for the production of records made pursuant to ss. 278.1 to 278.91 of the Criminal Code is a seizure within the meaning of s. 8 of the Charter The therapeutic relationship is one that is characterized by trust, an element of which is confidentiality The protection of the complainants reasonable expectation of privacy in her therapeutic records protects the therapeutic relationship and the mental integrity of complainants and witnesses The right to privacy is one which may be limited as reasonable searches and seizures are permitted by s. 8 of the Charter The accused will have no right to the records in question insofar as they contain information that is either irrelevant or would serve to distort the search for truth o On the other hand, the accuseds right must prevail where the lack of disclosure or production of the record would render him unable to make full answer and defence o Full answer and defence will be more centrally implicated where the information contained in a record is part of the case to meet or where its potential probative value is high o Privacy rights will be most directly at stake where a record concerns aspects of ones individual identity or where confidentiality is crucial to a therapeutic or trust-like relationship Equality concerns must also inform the contextual circumstances in which the rights of full answer and defence and privacy will come into play An appreciation of myths and stereotypes in the context of sexual violence is essential to delineate properly the boundaries of full answer and defence The definition of the records subject to Bill C-46 is not overly broad as the legislation only applies to records in which there is a reasonable expectation of privacy 142

This provision does not by itself deny access to documents to which the defence is constitutionally entitled Crown possession or control of such records cannot be equated with a total loss of any reasonable expectation of privacy It is constitutionally permissible for the Crown to be subject to different treatment, to different procedures, or even to end up with documents that the accused has not seen, as long as the accused can make full answer and defence and the trial is fundamentally Section 278.3(4) of the Criminal Code, which lists a series of assertions that cannot on their own establish that a record is likely relevant, does not violate s. 7 of the Charter The purpose of this provision is to prevent speculative and unmeritorious requests for production In s. 278.5(1) of the Criminal Code Parliament supplemented the likely relevant standard for production to the judge proposed in OConnor with the further requirement that production be necessary in the interests of justice Under the new provision a trial judge is required to consider the salutary and deleterious effects of production to the court on the accuseds right to make full answer and defence and on the rights to privacy and equality of the complainant or witness The criterion that production to the court be necessary in the interests of justice invests the trial judge with the discretion to consider the full range of rights and interests at issue before ordering production, in a manner scrupulously respectful of the requirements of the Charter In determining whether production is necessary in the interests of justice, the judge need not engage in a conclusive and in-depth analysis of each of the factors listed in s. 278.5(2) The provision merely requires that the judge take these factors into account Trial judges are only asked to take into account the factors listed in s. 278.5(2) and are not required to rule conclusively on each factor Trial judges are also not required to determine whether factors relating to the privacy and equality of the complainant or witness outweigh factors relating to the accuseds right to full answer and defence The inclusion of societal interest factors in the analysis does not alter the constitutional balance established in OConnor Lamer CJ (dissenting in part) While agree with the majoritys finding that Bill C-46 complies with ss. 7 and 11(d) of the Charter as it applies to the production of records in the possession of third parties, Lamer CJ took a different view of the legislative regimes approach to records in the hands of the Crown Bill C-46's treatment of records that form part of the case to meet tips the balance too heavily in favour of privacy to the detriment of the accuseds right to make full answer and defence

143

On the issue of equality, why was there no reference to the ten-part test for judging s. 15 set out in Law v. Minister of Human Resoirces Development (1999)?

Preliminary Inquiry
Prior to the trial of certain indictable offences, a judge will inquire into the charge and determine whether there is sufficient evidence to warrant placing the accused on trial The procedure for preliminary inquiries is set out in Part XVIII of the Code If the indictable offence is listed in s. 553 of the Code, the jurisdiction of the provincial court is absolute and does not depend on the choice of the accused, so there will be no preliminary inquiry If the offence is listed under s. 469 of the Code, the offence must be tried at the Superior Court, and a preliminary inquiry will likely be held With respect to all other indictable offences, if the accused elects to be tried in a provincial court, she will not have a preliminary inquiry s. 535 Criminal Code Inquiry by justice If an accused who is charged with an indictable offence is before a justice and a request has been made for a preliminary inquiry under subsection 536(4) or 536.1(3), the justice shall, in accordance with this Part, inquire into the charge and any other indictable offence, in respect of the same transaction, founded on the facts that are disclosed by the evidence taken in accordance with this Part.

Preferring the Indictment


The Criminal Code allows the Crown to jump over the preliminary inquiry and go straight to a trial; this is known as a "preferred indictment". The Code provides that following the preliminary inquiry, the Crown may prefer an indictment against the person ordered to stand trial See ss. 566 and 574 As per s. 574(1) the preferred indictment may be on o Any charge on which that person was ordered to stand trial; or o Any charge founded on the facts disclosed by the evidence taken on the preliminary inquiry, in addition to or in substitution for any charge on which that person was ordered to stand trial.

144

The power of the Crown to prefer an indictment under s. 574 if broader that the power of the Justice presiding at a preliminary inquiry

s. 566 Criminal Code Indictment (1) The trial of an accused for an indictable offence, other than a trial before a provincial court judge, shall be on an indictment in writing setting forth the offence with which he is charged. Preferring indictment (2) Where an accused elects under section 536 or re-elects under section 561 to be tried by a judge without a jury, an indictment in Form 4 may be preferred. What counts may be included and who may prefer indictment (3) Section 574 and subsection 576(1) apply, with such modifications as the circumstances require, to the preferring of an indictment pursuant to subsection (2). s. 574 Criminal Code Prosecutor may prefer indictment (1) Subject to subsection (3), the prosecutor may, whether the charges were included in one information or not, prefer an indictment against any person who has been ordered to stand trial in respect of (a) any charge on which that person was ordered to stand trial; or (b) any charge founded on the facts disclosed by the evidence taken on the preliminary inquiry, in addition to or in substitution for any charge on which that person was ordered to stand trial. Preferring indictment when no preliminary inquiry requested (1.1) If a person has not requested a preliminary inquiry under subsection 536(4) or 536.1(3) into the charge, the prosecutor may, subject to subsection (3), prefer an indictment against a person in respect of a charge set out in an information or informations, or any included charge, at any time after the person has made an election, re-election or deemed election on the information or informations. Preferring single indictment (1.2) If indictments may be preferred under both subsections (1) and (1.1), the prosecutor may prefer a single indictment in respect of one or more charges referred to in subsection (1) combined with one or more charges or included charges referred to in subsection (1.1). Consent to inclusion of other charges 145

(2) An indictment preferred under any of subsections (1) to (1.2) may, if the accused consents, include a charge that is not referred to in those subsections, and the offence charged may be dealt with, tried and determined and punished in all respects as if it were an offence in respect of which the accused had been ordered to stand trial. However, if the offence was committed wholly in a province other than that in which the accused is before the court, subsection 478(3) applies. Private prosecutor requires consent (3) In a prosecution conducted by a prosecutor other than the Attorney General and in which the Attorney General does not intervene, an indictment may not be preferred under any of subsections (1) to (1.2) before a court without the written order of a judge of that court. In addition, the Crown may prefer an indictment where a preliminary inquiry has not been held or where such has been held and the accused has been discharged This is sometimes referred to as direct indictment s. 577 Criminal Code Direct indictments Despite section 574, an indictment may be preferred even if the accused has not been given the opportunity to request a preliminary inquiry, a preliminary inquiry has been commenced but not concluded or a preliminary inquiry has been held and the accused has been discharged, if (a) in the case of a prosecution conducted by the Attorney General or one in which the Attorney General intervenes, the personal consent in writing of the Attorney General or Deputy Attorney General is filed in court; or (b) in any other case, a judge of the court so orders. There is no absolute right for the accused to be given a hearing or to submit representations before the preferement of a direct indictment and this does not violate s. 7 of the Charter However, the constitutionality of this was based on the fact that the accused is given adequate disclosure prior to trial

Preliminary inquiry as discovery vehicle


Skogman v. R, [1984] 2 SCR 93 Estey J The purpose of a preliminary hearing is to protect the accused from a needless and improper, exposure to public trial where the Crown is not in possession of evidence to warrant the continuation of the process In addition, in the course of the preliminary hearing has become a forum where the accused is afforded an opportunity to discover and to appreciate the case to be made against her at trial where the requisite evidence is found to be 146

The inquiry has become mainly a judicial examination of the justification and need for pretrial detention of the accused as well as an examination of the need for a trial itself Its primary purpose is to ascertain whether or not there is sufficient evidence to warrant the accused being placed upon his trial This is not a determination of whether or not the accused is guilty or not guilty The judges function is to ascertain whether or not there is sufficient evidence to induce the belief in the mind of a cautious person that the accused is probably guilty o Therefore, considerations of reasonable doubt have no application at this stage of the proceedings. From the point of view of defence counsel the preliminary hearing has another aspect It affords counsel an opportunity of ascertaining the nature and the strength of the case against her client and it may be likened in that respect to an Examination for Discovery

Test for committal: sufficiency of evidence


s. 548 Criminal Code Order to stand trial or discharge (1) When all the evidence has been taken by the justice, he shall (a) if in his opinion there is sufficient evidence to put the accused on trial for the offence charged or any other indictable offence in respect of the same transaction, order the accused to stand trial; or (b) discharge the accused, if in his opinion on the whole of the evidence no sufficient case is made out to put the accused on trial for the offence charged or any other indictable offence in respect of the same transaction. Endorsing charge (2) Where the justice orders the accused to stand trial for an indictable offence, other than or in addition to the one with which the accused was charged, the justice shall endorse on the information the charges on which he orders the accused to stand trial. Where accused ordered to stand trial (2.1) A justice who orders that an accused is to stand trial has the power to fix the date for the trial or the date on which the accused must appear in the trial court to have that date fixed. Defect not to affect validity (3) The validity of an order to stand trial is not affected by any defect apparent on the face of the 147

information in respect of which the preliminary inquiry is held or in respect of any charge on which the accused is ordered to stand trial unless, in the opinion of the court before which an objection to the information or charge is taken, the accused has been misled or prejudiced in his defence by reason of that defect. For many years, the test for committal at preliminary inquiry was held to be the same as that declared by the SCC in United States v. Sheppard (1976) The judge was not to weigh the evidence by assessing credibility, but assess whether a properly instructed jury would convict if it believed the evidence R. v. Nelles (1982), 16 CCC (3d) 97 (Ont. Prov. Ct) Vanek J In arriving at a decision whether or not to commit the accused for trial, it is not the function of a "justice" at a preliminary inquiry to WEIGH the evidence for the purpose of determining whether it is credible or trustworthy These matters fall within the sphere of the jury The justice must assess whether it is "sufficient to warrant committing the accused for trial A justice at a preliminary inquiry is bound to weigh the evidence in the sense of analyzing it and considering the inferences that may reasonably be drawn there from it for the purpose of judging the sufficiency of the evidence for committal In this sense, perhaps one might substitute for weighing the word assessing Comment Notice how the court here introduced the term weighing the evidence in relation to an assessment for committal; R. v. Arcuri, [2001] 2 SCR 828 Facts The accused was charged with first degree murder At the preliminary inquiry, the Crowns case was entirely circumstantial and the accused called two witnesses whose testimony was arguably exculpatory The preliminary inquiry judge rejected the accuseds contention that he must weigh the evidence and, after viewing the evidence as a whole, determined that the accused should be committed to trial for second degree murder The accuseds certiorari application was dismissed and that decision was affirmed by the CA Issue: Did the trial judge, in determining whether the evidence was sufficient to commit the accused, err in refusing to weigh the Crowns evidence against that of the accused? Holding: No; appeal dismissed McLachlin CJ The question to be asked by a preliminary inquiry judge under s. 548 of the Code is whether there is any evidence upon which a reasonable jury properly instructed could return a 148

verdict of guilty The task is essentially the same, in situations where the defence tenders exculpatory evidence, whether it be direct or circumstantial Where the Crown adduces direct evidence on all the elements of the offence, the case must proceed to trial, regardless of the existence of defence evidence, as the only conclusion that needs to be reached is whether the evidence is true However, where the Crowns evidence consists of, or includes, circumstantial evidence, the judge must engage in a limited weighing of the whole of the evidence (i.e. including any defence evidence) to determine whether a reasonable jury properly instructed could return a verdict of guilty In performing the task of LIMITED WEIGHING, the preliminary inquiry judge does not draw inferences from facts, nor does she assess credibility The judges task is to determine whether, if the Crowns evidence is believed, it would be reasonable for a properly instructed jury to infer guilt This task of limited weighing never requires consideration of the inherent reliability of the evidence itself It should be regarded, instead, as an assessment of the reasonableness of the inferences to be drawn from the circumstantial evidence Notwithstanding certain confusing language in Mezzo and Monteleone, nothing in this Courts jurisprudence calls into question the continuing validity of the common law rule in Shephard Comments Notice here the use of the concept of limited weighing United States v. Ferras, [2006] 2 SCR 77 Facts The US sought the extradition of the accused under the record of the case method provided for in ss. 32(1)(a) and 33 of the Extradition Act The records of the case submitted at their committal hearings consist of unsworn statements from law enforcement agents summarizing the evidence expected to be presented at each trial The US certified that the evidence is available for trial and is sufficient to justify prosecution under the law of the US The accused alleged that ss. 32(1)(a) and 33 infringe s. 7 of the Charter because they allow for the possibility that a person might be extradited on inherently unreliable evidence In both cases, the extradition judges rejected the constitutional objection and committed the accused for extradition The Court of Appeal upheld the decisions. Issue: Did tendering statements summarizing the evidence to be adduced at trial during the preliminary inquiry violate s. 7 of the Charter? 149

Holding: No; appeal dismissed McLachlin CJ On the majority view in Shephard, committal may be ordered in the absence of certification that the evidence is available for trial This raises particular concerns in an extradition context because the committal becomes the final judicial determination that sends the subject out of the country If the extradition judge possesses neither the ability to declare unreliable evidence inadmissible nor to weigh and consider the sufficiency of the evidence, committal for extradition could occur in circumstances where committal for trial in Canada would not be justified Section 29(1) of the Extradition Act, as discussed, requires the extradition judge to be satisfied that the evidence would justify committal for trial in Canada, had the offence occurred here Canadian courts in recent decades have adopted the practice of leaving a case or defence to the jury where there is any evidence to support it, and have discouraged trial judges from weighing the evidence and refusing to put a matter to the jury on the basis that the evidence is not sufficiently reliable or persuasive This may explain the conclusion in Shephard that the extradition judge has no discretion to refuse to extradite if there is any evidence, however scant or suspect, supporting each of the elements of the offence alleged This narrow approach to judicial discretion should not be applied in extradition matters, in my opinion It is important to note the differences between extradition hearings and domestic preliminary inquiries Both are pre-trial screening devices and both use the same test of sufficiency of evidence for committal: whether evidence exists upon which a reasonable jury, properly instructed, could return a verdict of guilty as in Shephard The new Act, however, does not maintain this close parallel in proceedings o Section 24(2) of the Act grants the extradition judge the same powers as a preliminary inquiry judge, but requires the judge to exercise those powers in a manner appropriate to the extradition context o A second difference comes from the different rules for admitting evidence Evidence adduced on extradition may lack the threshold guarantees of reliability afforded by Canadian rules of evidence o A third difference comes from the ability of extradition judges to grant Charter remedies To deny an extradition judges discretion to refuse committal for reasons of insufficient evidence would violate a persons right to a judicial hearing by an independent and impartial magistrate, a right implicit in s. 7 of the Charter where liberty is at stake It would deprive the judge of the power to conduct an independent and impartial judicial review of the facts in relation to the law, destroy the judicial nature of the hearing, and 150

turn the extradition judge into an administrative arm of the executive In so far as the majority view in the pre-Charter case of Shephard suggests a contrary view, it should be modified to conform to the requirements of the Charter

However, s. 32(1)(a) and (b) and s. 33 of the 1999 Act do not violate the right of a person sought under s. 7 of the Charter, because the requirements for committal of s. 29(1), properly construed, grant the extradition judge discretion to refuse to extradite on insufficient evidence such as where the reliability of the evidence certified is successfully impeached or where there is no evidence, by certification or otherwise, that the evidence is available for trial In R. v. Hynes (2001), the SCC (by a 5-4 majority) confirmed the position that a justice presiding a preliminary inquiry is not a court of competent jurisdiction to consider whether evidence obtained in violation of the Charter should be excluded McLachlin CJ, held that this would change the screening function of the preliminary inquiry and undermine the expeditious nature of the process Given the attitude of some defence lawyers to whack the complainant hard at the preliminary some have advocated the elimination of these hearings in cases involving vulnerable victims, such as cases of sexual assault In 2002 amendments to the Criminal Code imposed 3 major changes to preliminary hearings Apart from murder, where preliminary inquiries are still automatic, defence counsel must now request a preliminary (s. 536.4) o The decision then rests on the competence of the accuseds legal counsel Defence counsel must now indicate which issues they want addressed and provide a list of witnesses to be called Defence counsel may need to conduct preliminary inquiries on the basis of hearsay and other inadmissible evidence subject to a discretion in the justice to order attendance of the witness for cross-examination (ss. 540(7)-(9))

Constitutional Remedies
Any time after the election of mode of trial is made a pre-trial motion regarding constitutional remedies can be made Preliminary inquiries are not allowed to grant constitutional remedies Alternatively, there may be a voir dire during the trial regarding to constitutional remedies s. 24 Canadian Charter of Rights and Freedoms Enforcement of guaranteed rights and freedoms (1) Anyone whose rights or freedoms, as guaranteed by this Charter, have been infringed or denied may apply to a court of competent jurisdiction to obtain such remedy as the court considers appropriate and just in the circumstances. 151

Exclusion of evidence bringing administration of justice into disrepute (2) Where, in proceedings under subsection (1), a court concludes that evidence was obtained in a manner that infringed or denied any rights or freedoms guaranteed by this Charter, the evidence shall be excluded if it is established that, having regard to all the circumstances, the admission of it in the proceedings would bring the administration of justice into disrepute. As for courts of competent jurisdiction The trial court is a court of competent jurisdiction The preliminary inquiry is not A Superior Court is always a court of competent jurisdiction Section 24 is usually understood as a compromise between the American exclusionary rule and the former Canadian position derived from the common law (see Collins) Before, the accused would be entitled to criminal and civil penalties against law enforcement officials, but not the exclusion of evidence It appears that the legislature only picked one ground for exclusion of evidence: judicial integrity In R. v. Big M Drug Mart Ltd. (1985) the SCC held that any provincial court has the power to declare laws of no force an effect under s. 52(1) of the Charter Apart for this remedy for all courts, those of competent jurisdiction have wide remedies entrusted on them under s. 24 s. 52(1) Constitutional Act, 1982 The Constitution of Canada is the supreme law of Canada, and any law that is inconsistent with the provisions of the Constitution is, to the extent of the inconsistency, of no force or effect. The SCC has yet to pronounce on the range of appropriate and just remedies, though courts have resorted over time to dismissals, stays, costs, return of seized goods or even reduction of sentences In R. v. Therens (1983) the Saskatchewan CA held that the exclusion of evidence was an appropriate remedy under s. 24(1) The SCC in 1985, however, held in this same case s. 24(2) is the sole basis for the exclusion of evidence in a case of a Charter right infringement Despite Therens, the SCC has since then recognized an uncertain discretion to exclude evidence to ensure a fair trial under ss. 7, 11(d) and even s. 24 (1) o See R. v. Harrer (1995) and R. v. White (1999) In R. v. Buhay (2003) there is an obiter by Arbour J in recognition of a common law discretion to exclude evidence obtained in circumstances that would lead to an unfair trial or if the prejudicial effect outweighs the probative value

Exclusion of evidence
152

Prior to the Charter courts followed English common law in holding that illegality in obtaining evidence generally has no bearing on its admissibility The leading case of this was A.G. Que. V. Begin (1955) where blood test that proved intoxication where admissible despite having been obtained from the accused without warning him that they could be used against him The key issue to determine was whether the evidence was relevant or not o That is whether its probative value outweighed its prejudicial effect In 1984, the UK Parliament passed the Police and Criminal Evidence Act which reconstructed the voluntary confession rule so that evidence admitted by oppression would be inadmissible in court In the US there is an automatic exclusionary rule of illegally obtained evidence o The fruit of the poisonous tree doctrine o Much of this derives from constitutional rights protected by the 4th Amendment and the evidence that civil and criminal penalties against police officers did not offer enough protection of these rights and deterrence of illegal police conduct R. v. Collins supra, [1987] 1 SCR 265 Facts [For the facts, see the summary of the case under the material on searches] Issue: When should evidence be excluded under s. 24(2) of the Charter? Lamer J S. 24(2) has adopted an intermediate position with respect to the exclusion of evidence obtained in violation of the Charter It rejected the American rule excluding all illegally obtained evidence and the common law rule that all relevant evidence was admissible regardless of the means by which it was obtained The phrase "if it is established that" places the burden of persuasion on the applicant to show that the evidence would bring the administration of justice into disrepute The standard of persuasion required can only be the civil standard of the balance of probabilities Whether the admission of the evidence would bring the administration of justice into disrepute is the applicable test Misconduct by the police in the investigatory process often has some effect on the repute of the administration of justice, but s. 24(2) is not a remedy for police misconduct at this stage The drafters of the Charter decided to prevent having the administration of justice brought into further disrepute by the admission of the evidence in the proceedings It will also be necessary to consider any disrepute that may result from the exclusion of the evidence Even though the inquiry under s. 24(2) will necessarily focus on the specific prosecution, it is the 153

long-term consequences of regular admission or exclusion of this type of evidence on the repute of the administration of justice which must be considered Since the concept of disrepute involves some element of community views, the test should be put figuratively in terms of the REASONABLE PERSON: would the admission of the evidence bring the administration of justice into disrepute in the eyes of the reasonable person, dispassionate and fully apprised of the circumstances of the case A judge's discretion under this test is thus not untrammelled, for she should not render a decision that would be unacceptable to the community, provided the community is not being wrought with passion or otherwise under passing stress due to current events S. 24(2) directs the judge to consider all the circumstances in determining whether the admission of evidence would bring the administration of justice into disrepute The courts have considered a number of factors, which list is not exhaustive 1. Certain factors are relevant in determining the effect of the admission of evidence on the fairness of the trial The trial is a key part of the administration of justice and its fairness is a major source of the repute of the system guaranteed by s. 11(d) of the Charter The factors relevant to this determination will include the nature of the evidence obtained as a result of the violation and the nature of the right violated and not so much the manner in which the right was violated An unfair trial may result with respect to cases where, after a violation of the Charter, the accused is conscripted against herself through a confession or other evidence emanating from her 2. A second group of factors relates to the seriousness of the Charter violation and therefore to the disrepute that will result from judicial acceptance of evidence obtained through that violation Relevant considerations include whether the violation it was deliberate, whether there were alternative investigative techniques, or whether the violation was motivated by necessity to prevent the loss or destruction of the evidence 3. The third group relates to the effect of excluding the evidence: exclusion of evidence essential to a charge because of a trivial breach of the Charter would result in an acquittal and would bring the administration of justice into varying degrees of disrepute directly proportionate to the seriousness of the charge The more serious the offence, however, the more damaging would be an unfair trial to the system's repute 4. A final factor, the availability of other remedies, is not relevant The threshold for exclusion under s. 24(2) is lower than that under the "community shock test" enunciated in Rothman v. The Queen Under s. 24(2), there will have been a constitutional violation as opposed to the absence of any unlawful behaviour as a result of the resort to tricks in Rothman The language of s. 24(2) further indicates a lower threshold 1. The French version of the text, which translates could bring the administration of 154

justice into disrepute, is less onerous than the more stringent English version, "would bring the administration of justice into disrepute", and consequently is preferable in that it better protects the right to a fair trial The evidence should be excluded notwithstanding the fact that the trial would not be rendered unfair by its admission or the fact that exclusion could bring the administration of justice into disrepute by allowing a person convicted at trial of a relatively serious offence to evade that conviction The administration of justice would be brought into even greater disrepute if the Court did not exclude the evidence and dissociate itself from the conduct of the police which, assuming it was based on mere suspicion, flagrantly and seriously violated the individual's rights The matter might be clarified at a new trial where the officer could explain his reasons for his actions Lecture notes While Lamer J states that police actions can bring the administration of justice into disrepute, the focus is to prevent bringing the administration of justice into further disrepute by not excluding the evidence o This highlights that the purpose of this exclusion is judicial integrity In terms of what would render a trial unfair, an important element is self-incrimination o This could result from a forced confession (or conscriptive evidence) or forcing the accused to provide something from her body which would not be otherwise obtainable o The admission of this kind of evidence would bring the administration of justice into disrepute In terms of the second branch the court may consider whether the police was acting in good faith or whether there were alternative means to obtaining the evidence without infringing the Charter violation In Hebert, Sopinka J discussed that when evidence is impugned because it violates the first set of factors in Collins (trial fairness) it cannot be saved by resorting to the second set of factors (seriousness of the offence) by showing that the police was not ill-intentioned This was upheld by the SCC in R. v. Elshaw (1991) The leading decisions on the exclusion of conscriptive evidence and the consideration of trial fairness is Stillman R. v. Stillman supra, [1997] 1 SCR 607 Facts [For the facts, see the summary of the case under the material on searches] Issue: When should conscriptive evidence be excluded under s. 24(2) of the Charter? Cory J A consideration of trial fairness is of fundamental importance; a conviction resulting from an unfair trial is contrary to our concept of justice 155

If after careful consideration it is determined that the admission of evidence obtained in violation of a Charter right would render a trial unfair then the evidence must be excluded without consideration of the other Collins factors

The primary aim and purpose of considering the trial fairness factor in the s. 24(2) analysis is to prevent an accused person whose Charter rights have been infringed from being forced or CONSCRIPTED to provide evidence in the form of confessions, statements or bodily samples for the benefit of the state When the trial fairness factor is being considered, it is necessary to classify the evidence as conscriptive or non-conscriptive based upon the manner in which the evidence was obtained The crucial element which distinguishes non-conscriptive evidence from conscriptive evidence is not whether the evidence may be characterized as real or not If the accused was not compelled to participate in the creation or discovery of the evidence, the evidence will be classified as non-conscriptive Its admission will not render the trial unfair and the court will proceed to consider the seriousness of the breach and the effect of exclusion on the repute of the administration of justice If the evidence, obtained in a manner which violates the Charter, involved the accused being compelled to incriminate herself either by a statement or by the use as evidence of her body or of bodily substances, it will be classified as conscriptive evidence It is repugnant to fair-minded people to think that police can without consent or statutory authority take or require an accused to provide parts of their body or bodily substances in order to incriminate themselves The recognition of the right to bodily integrity and sanctity is embodied in s. 7 of the Charter A subset of conscriptive evidence is derivative evidence This is a term frequently used to describe what is essentially conscriptive real evidence; it involves a Charter violation whereby the accused is conscripted against herself (usually in the form of an inculpatory statement) which then leads to the discovery of an item of real evidence If the evidence under consideration is classified as conscriptive, which in the case of statements includes derivative evidence, then it will be necessary to take the second step of the analysis and determine whether the admission of the evidence would render the trial unfair o As a general rule, this type of evidence will render the trial unfair, unless it would have been discovered in the absence of the unlawful conscription of the accused Either because an independent source of the evidence exists OR the discovery of the evidence was inevitable o The Crown must bear the onus of establishing discoverability on a balance of probabilities 156

o The Court would then proceed to consider the seriousness of the violation and the effect of the exclusion Trial fairness summary 1. Classify the evidence as conscriptive or non-conscriptive based upon the manner in which the evidence was obtained o If the evidence is non-conscriptive, its admission will not render the trial unfair and the court will proceed to consider the seriousness of the breach and the effect of exclusion on the repute of the administration of justice. 2. If the evidence is conscriptive and the Crown fails to demonstrate that the evidence would have been discovered by alternative means, then its admission will render the trial unfair o The Court, as a general rule, will exclude the evidence without considering the seriousness of the breach or the effect of exclusion 3. If the evidence is found to be conscriptive and the Crown demonstrates that it would have been discovered by alternative means, then its admission will generally not render the trial unfair o However, the seriousness of the Charter breach and the effect of exclusion on the repute of the administration of justice will have to be considered The accuseds bodily samples and impressions existed as real evidence but the police, by their words and actions, compelled the accused to provide evidence from his body The impugned evidence would not have been discovered had it not been for the conscription of the accused in violation of his Charter rights and no independent source existed by which the police could have obtained the evidence It follows that its admission would render the trial unfair and this finding is sufficient to resolve the s. 24(2) issue as the evidence must be excluded Cory J also found that the breach was also serious and would shock the consentience of the community On the issues of the tissue, the Court found that it should not be excluded The police did not force, or even request, a mucous sample from the accused, and although the police acted surreptitiously in disregard for the accuseds explicit refusal to provide them with bodily samples, the violation of the accuseds Charter rights with respect to the tissue was not serious The seizure did not interfere with the accuseds bodily integrity, nor cause him any loss of dignity The police could and would have obtained the discarded tissue, it was discoverable and the administration of justice would not be brought into disrepute LHeureux-Dub J (dissenting) The framework set out in Collins represents the proper approach to s. 24(2) and efforts since then to explain, clarify and refine or distinguish Collins, have only served to further muddy the waters The classification of evidence under the trial fairness aspect of the s. 24(2) analysis, in 157

terms of non-conscriptive real evidence and conscriptive evidence is an unfortunate development McLachlin J (dissenting) Evidence which affects the fairness of the trial must not inevitably be excluded under s. 24(2) o A proper consideration of all the circumstances demands a balancing of each set of factors set out in Collins The principle against self-incrimination does not apply to real evidence, except that which is derivative from compelled testimony, and there are different degrees of trial fairness o Depending on the degree of unfairness and countervailing circumstances, the fairness of the manner in which the evidence was obtained may or may not result in rejection of the evidence under s. 24(2) Lecture notes Stillman broadens evidence emanating from the accused from merely the use of statements, to the use of evidence emanating from the accuseds body or bodily substances o What matters if whether the evidence was obtained with or without the participation of the accused o Even that which was obtained with the participation of the accused can be admissible if it could have been discoverable otherwise Derivative evidence is real evidence which location is obtained from conscriptive evidence Does discoverability refer to evidence that could have been discoverable anyway or would have been discoverable anyway (which is the term employed by the court) Criticisms of this decisions point to the fact that conscriptive evidence is arbitrarily defined o This case could also lead to a perversion of the discoverability doctrine There is some debate as to whether the conscriptive categorization of Stillman should be narrowly applied to statements where the accused incriminates herself, or situations where police uses the body of the accused or forces the production of bodily samples R. v. Feeney supra, [1997] 2 SCR 13 Facts [For the facts, see the summary of the case under the material on searches] Issue: When should evidence be excluded under s. 24(2) of the Charter? Sopinka J Fairness of the trial The bloody shirt is non-conscriptive, and thus its admission does not go to trial fairness The shirt existed in a form useable by the state independent of any actions by the police and the accused was not compelled to incriminate himself by means of a statement, the use of the body, or the production of bodily samples 158

The statements obtained by the police in the trailer in violation of the appellants s. 10(b) rights are clearly conscriptive evidence, and self-incriminating The next question set out in Stillman is whether alternative legal means to obtain the conscriptive evidence existed o The statements were not discoverable, thus their admission would affect the fairness of the trial The shoes were observed by the police during the initial unconstitutional search of the trailer which violated s. 8, yet they are clearly non-conscriptive evidence, given that they are not compelled statements or bodily samples, and did not involve the use of the appellants body The admission of the shoes would thus not affect trial fairness The cigarettes were also observed by the police in the initial unconstitutional search of the trailer, yet their admission would not affect trial fairness The conscriptive statement at Williams Lake about the cash was a sufficient cause for obtaining the cash, stating as it did the location of the cash However, the statement was not a necessary cause of the taking of the cash, so the money was not conscriptive, derivative evidence The fingerprints were also taken in violation of the Charter and were conscriptive evidence It is important to note the distinction between the test for characterizing evidence as conscriptive, derivative evidence and the test for determining whether conscriptive evidence is discoverable Discoverability is concerned with whether a Charter breach was necessary to the discovery and obtaining of conscriptive evidence o If the conscriptive evidence would have been obtained even if the Charter had not been breached, the evidence is discoverable and its admission, despite the conscription of the accused, would not affect trial fairness The alternative means to obtain the evidence must comply with the Charter The derivative evidence inquiry, on the other hand, is directed at determining whether a piece of evidence should be viewed as having a conscriptive nature because of its intimate relationship with other conscriptive evidence o Evidence is derivative evidence if it would not have been obtained but for the conscriptive evidence o In analyzing this question, it is not relevant whether the means by which the evidence would have been discovered in the absence of the conscription were constitutional The unconstitutionality of the second search is a factor to be considered under other branches of the Collins test The evidence which does not affect trial fairness must be analyzed in light of the second and third branches of the Collins test

159

Seriousness of the Violation The violations were serious in the present case One of the indicia of seriousness is whether the violations were undertaken in good faith o One indication of bad faith is that the Charter violation was undertaken without any lawful authority o In the instant case, the police did not even have subjective belief in reasonable and probable grounds for the accuseds arrest Exigent circumstances did not exist in this case any more than they would exist in any situation following a serious crime After any crime is committed, the possibility that evidence might be destroyed is inevitably present o To tend to admit evidence because of the mitigating effect of such allegedly exigent circumstances would invite the admission of all evidence obtained soon after the commission of a crime Effect of Exclusion on the Repute of the Administration of Justice The admission of the conscriptive evidence would impact on the fairness of the trial and harm the repute of the administration of justice The other evidence, while not conscriptive, was obtained as the result of a very serious intrusion of the accuseds privacy rights Moreover, the evidence was associated with serious violations of the accuseds s. 10(b) rights, indicating a pattern of disregard for the Charter by the police The serious disregard for the appellants Charter rights in the case at bar suggests that the admission of the evidence would bring greater harm to the repute of the administration of justice than its exclusion The shirt, shoes, cigarettes and money were inadmissible under s. 24(2), along with the statements and the fingerprints Any price to society occasioned by the loss of a conviction in this case is fully justified in a free and democratic society which is governed by the rule of law LHeureux-Dub J (dissenting) The actions of the police did not breach the Charter If they had, the exigent circumstances of the case would make it so that the exclusion of the evidence would bring the administration of justice into disrepute Lamer CJ (dissenting) disagreed with the way in which Stillman was applied to the facts of the case, but not the principles laid out in it

160

In R. v. Grant the Ontario CA questioned the automatic exclusionary rule of evidence because it is conscriptive or derivative evidence Instead, they proposed looking into the other factors of the Collins test and these outweighed the justification for the exclusion of the evidence due to trial unfairness We are waiting to hear from the SCC on this issue In R. v. Buhay the court established that more attention was to be given to the last two factors of the Collins test

Judicial stay as abuse of process


Prior to the Charter, the courts developed a common law power for judges to stay criminal proceedings as an abuse of process, in order to control oppressive prosecutorial practices The rule was devised to deter private litigants from engaging in abuse of process The power was used sparingly, however In R. v. Rourke, the majority of the SCC seemed to reject any notion of this judicial power Rourke v. R., [1978] 1 SCR 1021 Facts An information was sworn against the accused alleging a kidnapping and robbery and a warrant of arrest was issued At the opening of the trial, the County Court judge granted a stay of the proceedings on the ground that the accused was prejudiced because of the delay in commencing the prosecution, a person who would have been a key witness having died in the interval An application for mandamus was dismissed by a judge of the Supreme Court of British Columbia on the basis that the trial judge had jurisdiction to do what he had done and, having exercised his discretion bona fide, his decision could not be revised The Court of Appeal unanimously held that inordinate delay was not a fact which would give discretion to the County Court judge to stay the proceedings, reversed the judgment of the Supreme Court and granted the mandamus Issue: Can judges stay a proceeding for inordinate prosecutorial delay? Holding: No; appeal dismissed Pigeon J There is no rule in our criminal law that prosecutions must be instituted promptly and ought not to be permitted to be proceeded with if a delay in instituting them may have caused prejudice to the accused Nor is there any general discretionary power in courts of criminal jurisdiction to stay proceedings regularly instituted because the prosecution is considered oppressive The absence of any provision in the Criminal Code contemplating the staying of an indictment by a trial judge or an appeal from such decision is a strong indication against the existence of any power to grant such stay Section 7(3) of the Criminal Code which preserves common law defences does not apply 161

to a discretionary stay of proceedings It would also be inconsistent with regular procedures of appeal to have a discretionary jurisdiction that could be exercised by superior court judges in criminal matters where the only possible appeal would be a direct appeal to the SCC under s. 41(1) of the Supreme Court Act Considerations of policy may not be of much importance in the application of explicit statutory provisions because policy decisions are essentially for Parliament Laskin CJC Every court having criminal jurisdiction has the power to stay proceedings which are an abuse of process or oppressive and vexatious However, absent any contention that the delay in apprehending the accused had some ulterior purpose, courts are in no position to tell the police that they did not proceed expeditiously enough with their investigation, and then impose a sanction of a stay when prosecution is initiated The time lapse between the commission of an offence and the laying of a charge cannot be monitored by Courts by fitting investigations into a standard mould or moulds Subject to such controls as are prescribed by the Criminal Code, prosecutions initiated a lengthy period after the alleged commission of an offence must be left to take their course and to be dealt with by the Court on the evidence, which judges are entitled to weigh for cogency as well as credibility o The Court can call for an explanation of any improper delay in prosecution and may be in a position, accordingly, to assess the weight of some of the evidence In this case, there is no basis, despite the allegations of prejudice, upon which abuse of process could be invoked Lectures notes Pigeon J saw that a stay could not be appealed like a defence, and this informed his decisions against recognizing the abuse of process R. v. Jewitt, [1985] 2 SCR 128 Facts The accused was charged with unlawfully trafficking a narcotic At trial, he admitted selling marijuana to an undercover police officer but only because he was persuaded to do so by a fellow employee who was a police informer The jury found there had been unlawful entrapment and the court therefore directed "a stay of proceedings" A majority of the Court of Appeal dismissed the Attorney General's appeal for want of jurisdiction Issues: 1. Is there a discretionary power at common law to stay proceedings in a criminal case for abuse of process? 2. Is a judicially entered stay of proceedings a "judgment or verdict of acquittal of a trial court", 162

from which the Crown may appeal to the Court of Appeal under s. 605(1)(a) of the Criminal Code? Holding: 1.Yes; 2. Yes; appeal allowed Dickson CJC 1. The discretionary power to stay proceedings for abuse of process The inherent jurisdiction of a superior court to stay proceedings which are an abuse of its process was recognized in Canada as early as 1886 In recent years, however, uncertainty has clouded the question whether Canadian courts, apart from powers given to the Attorney General under s. 508 of the Criminal Code, have a discretion to stay proceedings for abuse of process This decision in Rourke was seen by some as a death blow to the doctrine of abuse of process The breadth of the decision in Rourke has been the subject of differing views in various provincial appellate courts Abuse of process was most recently reviewed by the SCC in the case of Amato v. The Queen Here, the decisions of Osborn and Rourke were interpreted as being limited to the ratio of the judgments disposing of the issues arising on the facts of these cases Adopting the conclusion of the Ontario CA in R. v. Young (1984), "there is a residual discretion in a trial court judge to stay proceedings where compelling an accused to stand trial would violate those fundamental principles of justice which underlie the community's sense of fair play and decency and to prevent the abuse of a court's process through oppressive or vexatious proceedings" A caveat, however, is that this power which can be exercised only in the CLEAREST OF CASES 2. Whether a stay is a judgment or verdict of acquittal The stay of proceedings was tantamount to a judgment or verdict of acquittal and subject to appeal by the Crown pursuant to s. 605(1)(a) of the Criminal Code There is no logical reason why a decision to quash an indictment on a question of law should be considered a judgment or verdict of acquittal whereas a decision to enter a stay on a question of law should not The decision to stay was not based on procedural considerations, but rather on questions of law, and such decision was final Lectures notes In justifying the SCCs change in view Laskin CJC draws examples of cases where a stay was granted o Perhaps a better explanation would have been the Charter

163

R. v. Keyowski, [1988] 1 SCR 657 Facts The accuseds first two trials on a charge of criminal negligence causing death ended with the juries failing to agree on a verdict A third trial was stayed by the trial judge on the grounds that it would constitute both an abuse of process and a violation of s. 7 of the Charter The Crown's appeal was allowed by a majority of the Saskatchewan CA and a new trial ordered o The CA held that it was necessary for the accused to show prosecutorial misconduct Issue: When a series of trials have taken place is it necessary to show prosecutorial misconduct in order to grant a stay for abuse of process? Holding: No; the appeal was dismissed for other reasons Wilson J A stay of proceedings to remedy an abuse of process is available where the proceedings are "oppressive or vexatious", but such power can be exercised only in the "clearest of cases" To define "oppressive" as requiring misconduct or an improper motive would unduly restrict the operation of the doctrine of abuse of process Prosecutorial misconduct and improper motivation are two of many factors to be taken into account when a court is called upon to consider whether or not in a particular case the Crown's exercise of its discretion to re-lay the indictment amounts to an abuse of process Nevertheless the disagreement with the CA, the administration of justice would be best served in this case by allowing the Crown to proceed with the new trial The appellant had not demonstrated this to be one of those "clearest of cases" which would justify a stay o The charge is a serious one, the proceedings have not occupied an undue amount of time and the accused has not been held in custody o A third trial may indeed, stretch the limits of the community's sense of fair play but does not of itself exceed them. [Since the parties did not argue why they agreed that the common law doctrine of abuse of process was now subsumed in s. 7 of the Charter, this issue was left open] Lectures notes Other factors that may lead to abusive process are o Seriousness of the charge o Time spent in custody o Harm caused to the accused

R. v. OConnor, [1995] 4 SCR 411 Facts 164

Issues: 1. Is there a need to maintain a distinction between the common law doctrine of abuse of process and Charter requirements regarding abusive conduct? 2. Did the non disclosure by the Crown justify an order that the proceedings be stayed for abuse of process? Holding: 1. No; 2. No; appeal dismissed LHeureux-Dub J 1. The distinction between the doctrine of abuse of process and Charter requirements The modern resurgence of the common law doctrine of abuse of process began with the judgment in R. v. Jewitt The general test for abuse of process adopted in that case has been repeatedly affirmed The only instances in which there may be a need to maintain any type of distinction between the common law doctrine of abuse of process and the Charter regime regarding abusive conduct will be those cases in which the Charter, for some reason, does not apply yet where the circumstances nevertheless point to an abuse of the court's process Both individual rights to trial fairness and the general reputation of the criminal justice system are fundamental concerns underlying both the common law doctrine of abuse of process and the Charter While the Charter is certainly concerned with the rights of the individual, it is also concerned with preserving the integrity of the judicial system o Subsection 24(2) of the Charter gives express recognition to this dual role The principles of fundamental justice in s. 7 are largely inspired by values that are fundamental to our common law, on which the doctrine of abuse of process is also premised 165

A bishop was charged with a number of sexual offences Defence counsel obtained a pre-trial order requiring that the Crown disclose the complainants' entire medical, counselling and school records The accused later applied for a judicial stay of proceedings based on non-disclosure of several items o The Crown asserted that the non-disclosure of some of the medical records was due to inadvertence on their part o The trial judge dismissed the application for a stay, finding that the failure to disclose certain medical records had been an oversight o He concluded that while the conduct of the Crown was "disturbing", he did not believe that there was a "grand design" to conceal evidence, nor any "deliberate plan to subvert justice" On the second day of the trial, counsel for the accused made another application for a judicial stay of proceedings based largely on the fact that the Crown was still unable to guarantee to the accused that full disclosure had been made o The trial judge stayed proceedings on all four counts The CA allowed the Crown's appeal and directed a new trial

It is clear that abuse of process under the Charter also contemplates important individual interests One often cannot separate the public interests in the integrity of the system from the private interests of the individual accused Another justification for this convergence is that remedies less drastic than a stay of proceedings are available under s. 24(1) in situations where the "clearest of cases" threshold is not met but where it is proved, on a balance of probabilities, that s. 7 has been violated In this respect the Charter regime is more flexible than the common law doctrine of abuse of process There is no reason why such balancing cannot be performed equally, if not more, effectively under the Charter, both in terms of defining violations and in terms of selecting the appropriate remedy to perceived violations There is no one particular "right against abuse of process" within the Charter Depending on the circumstances, different Charter guarantees may be engaged Abuses may be best addressed by reference to ss. 7 and 11(b) of the Charter In addition, there is a residual category of conduct caught by s. 7 of the Charter o This residual category does not relate to conduct affecting the fairness of the trial or impairing other procedural rights enumerated in the Charter, but instead addresses the diverse and sometimes unforeseeable circumstances in which a prosecution is conducted in such a manner as to connote unfairness or vexatiousness of such a degree that it contravenes fundamental notions of justice 2. Was the stay justified? A stay of proceedings is only appropriate "in the clearest of cases", where the prejudice to the accused's right to make full answer and defence cannot be remedied or where irreparable prejudice would be caused to the integrity of the judicial system if the prosecution were continued Where life, liberty or security of the person is engaged in a judicial proceeding, and it is proved on a balance of probabilities that the Crown's failure to make proper disclosure to the defence has impaired the accused's ability to make full answer and defence, a violation of s. 7 will have been made out In such circumstances, the court must fashion a just and appropriate remedy, pursuant to s. 24(1) Although the remedy for such a violation will typically be a disclosure order and adjournment, there may be some extreme cases where the prejudice to the accused is irremediable, thus justifying a stay in these "clearest of cases" [The minority felt that the Crowns conduct was inappropriate and as such violated the accuseds right to full answer and defence. For Cory J, however, the Crowns actions did not justify a stay For Lamer CJ, the Crown violated fundamental principles of justice underlying the communitys sense of decency] 166

Lectures notes LHeureux-Dub J broadens the remedies that are available for an abuse of process; a stay is the most drastic remedy o There are less drastic remedies available under s. 24(1) of the Charter R. v. L.(W.K), [1991] 1 SCR 1091 Facts The accused was charged with 17 counts of sexual assault, gross indecency and assault relating to his stepdaughter and two daughters Since there was a 30 year delay in reporting the abuse, the trial judge ordered a stay on the basis that a present trial would be contrary to principles of fundamental justice The British Columbia CA reversed the stay Issue: Did the delay in reporting past abuse justify a stay? Holding: No; appeal dismissed Stevenson J Staying proceedings based on the mere passage of time would be the equivalent of imposing a judicially created limitation period for a criminal offence In Canada, except in rare circumstances, there are no limitation periods in criminal law The comments of Laskin C.J. in Rourke are equally applicable under the Charter An accused's rights are not infringed solely because a lengthy delay is apparent on the face of the indictment Courts cannot assess the fairness of a particular trial without considering the particular circumstances of the case A delay may even operate to the advantage of the accused, since Crown witnesses may forget or disappear The nature of sexual offences explain even further the delay that took place in reporting these offences If proceedings were to be stayed based solely on the passage of time between the abuse and the charge, victims would be required to report incidents before they were psychologically prepared for the consequences of that reporting Since OConnor , the SCC has continued to vacillate on its approach to abuse of process and in the application of the residual category for abuse of process In R. v. Neil (2002), Binnie J, for the SCC, left open the question of whether the common law doctrine of abuse of process can be invoked when there is no state action in question This may well be one of those situations alluded to by LHeureux-Dub J in OConnor, where the common law doctrine is not subsumed by the Charter, since the Charter regime regarding abusive conduct does not apply

Pleas and Plea Bargaining


167

S. 606 of the Criminal Code allows the accused plead guilty or not guilty Such a plea will be accepted by the court if it is proved that the accused has made the plea voluntarily and understood the nature of the charge Subsection 1.2 states that the failure of the court to verify the previous conditions does not necessarily vacate the plea s. 606 Criminal Code Pleas permitted (1) An accused who is called on to plead may plead guilty or not guilty, or the special pleas authorized by this Part and no others. Conditions for accepting guilty plea (1.1) A court may accept a plea of guilty only if it is satisfied that the accused (a) is making the plea voluntarily; and (b) understands (i) that the plea is an admission of the essential elements of the offence, (ii) the nature and consequences of the plea, and (iii) that the court is not bound by any agreement made between the accused and the prosecutor. Validity of plea (1.2) The failure of the court to fully inquire whether the conditions set out in subsection (1.1) are met does not affect the validity of the plea. Refusal to plead (2) Where an accused refuses to plead or does not answer directly, the court shall order the clerk of the court to enter a plea of not guilty. Allowing time (3) An accused is not entitled as of right to have his trial postponed but the court may, if it considers that the accused should be allowed further time to plead, move to quash or prepare for his defence or for any other reason, adjourn the trial to a later time in the session or sittings of the court, or to the next of any subsequent session or sittings of the court, on such terms as the court considers proper. Included or other offence (4) Notwithstanding any other provision of this Act, where an accused or defendant pleads not guilty of the offence charged but guilty of any other offence arising out of the same transaction, 168

whether or not it is an included offence, the court may, with the consent of the prosecutor, accept that plea of guilty and, if the plea is accepted, the court shall find the accused or defendant not guilty of the offence charged and find him guilty of the offence in respect of which the plea of guilty was accepted and enter those findings in the record of the court. Video links (5) For greater certainty, subsections 650(1.1) and (1.2) apply, with any modifications that the circumstances require, to pleas under this section if the accused has agreed to use a means referred to in those subsections.

R. v. Lessard (1976), 33 CRNS 16 (Ont. CA) Facts The complainant (eighty-three years of age) is the step-grandmother of the respondent who was charged of indecent assault The accused pleaded not guilty but did not testify in his own defence On the completion of the evidence, the trial judge found him guilty When the proceedings resumed for sentencing purposes, defence counsel moved to reopen the case stating that his earlier decision not to call the accused was based on the "mental state and condition of the accused" and that the accused was nervous and, therefore, it was not in his best interest at the time to give evidence The trial judge granted the request as an unusual exercise of discretion, and after hearing the testimony of the accused he acquitted him Issue: Was the trial judge allowed to reopen the case after the finding that the accused was guilty? Holding: Yes; appeal dismissed Martin JA The word "conviction" is equivocal It is often used both to denote the adjudication of guilt and to denote the adjudication of guilt together with the judgment of the Court upon the finding of guilt, that is, the sentence A judge exercising the functions of both judge and jury is not functus officio following a finding of guilt until she has imposed sentence or otherwise finally disposed of the case It has been recognized for long that a trial judge may permit an accused who has pleaded guilty to change his plea at any time before the imposition of sentence, notwithstanding the acceptance of the plea by the Court, because up to that time the proceedings have not been completed Where the Court has dismissed the charge it is, of course, functus officio Since the proceeding has terminated the Court is not empowered to reopen the case and vacate the acquittal In the case of a trial by jury, the trial judge has no power to set aside the verdict of the jury 169

and direct a rehearing where the verdict has been arrived at on disputed facts on a plea of not guilty The separation of the functions of the judge and the jury is incompatible with the existence of such a power in the trial judge The immunity of the jury's verdict from interference by the trial judge is seen in the fact that the trial judge, neither at common law nor under the Code, is empowered to set aside the verdict of a jury in a criminal case on the ground that it is against the evidence Since the trial Judge had a discretion to reopen the case and to permit the accused to give evidence, the exercise of his discretion, unless he has failed to exercise it judicially, is not subject to an appeal by the Crown which can be maintained only upon a "ground of appeal that involves a question of law alone" under s. 605(1)(a) of the Code Adgey v. R. (1973), 23 CRNS 298 (SCC) Facts The accused pleaded guilty to several charges of false pretences, a charge of fraud, and a charge of break, enter and theft After plea had been entered, the facts giving rise to each charge were related by a police officer and the accused was afforded an opportunity to explain He was was convicted. No request for the change of plea was made during the hearing The appellant appealed to the CA which allowed the appeal as to one of the charges and dismissed the appeal as to all other charges Issue: Having heard the accuseds explanation, did the judge err in failing to strike the guilty pleas and direct trial on the charges? Holding: No; appeal dismissed Dickson J There are two stages in the proceedings in which a trial judge has a discretion so far as accepting a plea of guilty is concerned 1. When the charge is read and the plea is entered 2. Second, following the hearing of evidence, if the judge chooses to hear evidence A trial judge is not bound, as a matter of law, in all cases to conduct an inquiry after a guilty plea has been entered If, however, evidence is heard, it may indicate that the accused never intended to admit to a fact which is an essential ingredient of the offence or that she may have misapprehended the effect of the guilty plea or never intended to plead guilty, in any of which the judge may in her discretion direct that a plea of not guilty be entered or permit the accused to change his plea to not guilty This discretion if exercised judicially will not lightly be interfered with An appellant can however later change her plea if she can satisfy the Appeal Court that there are valid grounds for his being permitted to do so. On reviewing the charges to which the appellant pleaded guilty neither the facts admitted nor the statements made following the guilty pleas justify disturbing the exercise of discretion by the trial 170

judge Laskin J (dissenting) The accused did not have counsel until, on his third appearance in Court and on his trial at that time, duty counsel stood with him The fact that the trial judge did not inquire if duty counsel had had an opportunity to consult with the accused and to ascertain the factual bases of the charges and whether the accused was willing to be represented by him should there be a plea of guilty was sufficient to require that the convictions be set aside and a new trial ordered The duty of a trial judge respecting an inquiry into the facts on a plea of guilty is to satisfy herself that the accused understands the nature of the charge and its consequences and is unequivocal in his plea of guilty, but this must be complemented by the duty of the Crown to adduce facts which, taken to be true, support the charge and conviction In the present case on the facts narrated by the Crown there was sufficient doubt on the elements of the offence to warrant striking the plea R. v. Moser (2002), 163 CCC (3d) 286 (Ont. SCJ) Facts Application by the accused to set aside guilty pleas Moser pleaded guilty to charges of assault with a weapon and forcible confinement He admitted that he had accosted a 16- year-old girl and dragged her into his apartment at knifepoint but disputed the fact that he had deliberately removed some of her clothing At the time of the plea, Moser was aware that the Crown intended to proceed with a dangerous offender application In the assessment after the pleas, Moser denied using a knife in the assault The psychiatrists found that there was considerable risk that Moser would re-offend violently or sexually Moser argued that he pleaded guilty because he did not want to put the girl through the ordeal of giving evidence Issue: Should the accuseds guilty pleas be set aside? Holding: No; appeal dismissed Hill J An accused's plea of guilt is a fundamentally significant step in the criminal trial process. The plea relieves the Crown of the burden to prove guilt beyond a reasonable doubt, the presumption of innocence, the right to silence, and the right to make full answer and defence to the charge are at an end Where the offender is represented by counsel in a guilty plea proceeding, the trial court has a discretion, as a matter of law, to hold no further inquiry into the factual circumstances alleged in support of the offences charged If the circumstances raise doubt as to the validity of the plea, the trial judge has a 171

discretion to undertake an inquiry, the extent of which will vary according to the particular facts of each case Where the accused is self-represented, common law notions of fairness, if not constitutional concerns, mandate more extensive judicial involvement in the plea process A guilty plea, to be considered valid, must have minimally sufficient characteristics in order to provide an assurance that the forfeiture of a trial is fair To be valid, the plea must be unequivocal - the circumstances should not be such that the plea was unintended or confusing, qualified, modified, or uncertain in terms of the accused's acknowledgement of the essential legal elements of the crime charged A plea of guilty must be voluntary in the sense that the plea is a conscious volitional decision of the accused Plea negotiations in which the prosecution pursues a plea of guilt in exchange for forgoing legal avenues open to it, or agrees not to pursue certain charges, do not render the subsequent plea involuntary What is unacceptable is coercive or oppressive conduct of others or any circumstance personal to the individual which unfairly deprives the accused of free choice in the decision not to go to trial Some circumstances calling into question the voluntariness of a guilty plea are Pressure from the court Pressure from defence counsel Incompetence of defence counsel Cognitive impairment or emotional disintegration of the accused Effect of illicit drugs or prescribed medications A guilty plea's validity depends on the plea being informed It is essential that the accused understand the nature of the charges faced, the legal effect of a guilty plea, and the consequences of such a plea The accused must generally know the jeopardy faced by way of possible punishment The prior experience of the accused in the criminal justice system is one factor weighing toward the validity of the accused's plea as she has had the opportunity to participate in the process In the absence of any circumstances in the record, or a challenge to the competence and professionalism of trial counsel, a trial judge is justified in drawing the inference that counsel took the necessary steps to ensure the client understood the nature and consequences of a guilty plea We must have finality to proceedings unless the demands of justice dictate otherwise: Exceptional circumstances calling into question the validity of a guilty plea warrant a judicious exercise of discretion to inquire into the legitimacy of the plea o Where the circumstances are made known to the trial court, the presiding justice is 172

empowered with a discretion to conduct an inquiry into whether the plea should be rejected or struck and the case proceed to trial o Statements made in the course of the court's inquiry into the facts supporting the allegations immediately following a plea of guilt, although not admitted by the Crown, may justify the court in rejecting the guilty plea At the same time, the court must be vigilant that an experienced criminal not be allowed to abandon a position when matters "did not play out as expected"

While the accused carries the burden of persuading the court the plea is invalid and ought to be withdrawn, the jurisprudence is not clear as to the standard of persuasion Some authorities advocate a balance of probabilities standard and other for a heavier one As for the present case Moser was attempting to avoid the potential impact of a dangerous offender finding on his sentencing He knew the seriousness of the dangerous offender application when he pled The plea was unequivocal, voluntary, and informed As well, Moser had participated in prior guilty plea proceedings He retained experienced criminal counsel He admitted that his consumption of cocaine at the time of the assault affected his recollection of the facts

Plea Bargaining
Plea bargaining is not something that is provided by the Code Before the 1990s courts were reluctant to enforce them Nevertheless, 80% of cases are typically resolved by a guilty plea It was finally agreed that they should be used in order to relieve institutional resources Certain promises are within the discretion of the prosecutor to offer for a guilty plea Plea bargaining may be formal or involve a hurried hallway discussion The are usually entered into following a discussion between Crown attorneys and the defence counsel but sometime, although less frequently, the judge is involved Klein, Lets Make a Deal From the offenders point of view, the kickback is generally the most beneficial kind of deal The benefits obtained by law enforcement officials are sometime more apparent than real Some offenders may not be discourage from engaging in criminal activities Others may resort to crime to finance their trials if left out on bail awaiting trial Information is the livelihood of any police department However, for many offender the mental and physical costs of becoming an informer outweigh the costs 173

Police often emerge as frequent and significant actors o They are the gatekeepers to the criminal justice system

The need for bargaining often involves crime prevention, law enforcement and the conservation of limited resources Ericson, Making Crime: A Study of Detective Work The bench in Canada has done little more than assert their disapproval of plea bargaining, giving the impression that it is sporadic rather than established and widespread In making their case to Crown attorneys, detectives rely upon several organizational elements which allow them to effect considerable control, one being their superior knowledge of the cases before them Crown attorney are likely to defer to detective in making deals for guilty pleas Ferguson and Roberts, Plea Bargaining: Directions for Canadian Reform The range of benefits offered in exchange for plea bargains include Reduction in charge Withdrawal of other charges Recommendation for a particular sentence The use of summary conviction Promise not to force trial by jury Promise not to charge friends or family of the accused Promise as to the place of imprisonment, type of treatment and time of parole Promise to arrange sentence before a lenient judge Promise not to oppose release on bail or while awaiting sentence Ministry of Attorney General for Ontario Crown Policy Manual (2005) In conducting resolution discussions, Crown counsel should: Attempt to balance the interests of the victim, protection of the public and the rights of the accused in the framework of optimal use of limited resources A proposed sentence should reflect the public interest and the gravity of offence Fundamental principles that are binding directives Crown counsel must not accept a guilty knowing that the accused is innocent Crown must not accept a guilty plea to a charge that can never be proven unless that fact is fully disclosed to the defence Crown counsel must not purport to bind the Attorney Generals right to appeal any sentence Unless for exceptional circumstances, Crown counsel must honours agreements reached Kipnis, Criminal Justice and the Negotiated Plea The appeal that plea bargaining has is rooted in our attitude towards bargaining in general Where both parties are satisfied with the terms of an agreement, it is improper to interfere 174

Judicial neutrality may be commendable were entitlements are being exchange, but in the criminal justice system what matters is not whether people receive what they have bargained for, but what they deserve

In its coercion of accused and the relaxation of conviction and sentencing standards, plea bargaining falls short of the justice we expect of our legal system While we need to alleviate economic injustices on the one hand and decriminalize some victimless crime, we also need to demand respect of our legal institutions, which plea bargaining does not accomplish In general, courts have been reluctant to give any legal effect to a plea bargain However, on occasion, an attempt by the Crown to repudiate the bargain will be considered an abuse of process R. v. D.(E.) (1990), 73 OR (2d) 758 Facts The accused was charged with two counts of having sexual intercourse with a female under 14 and two counts of having illicit sexual intercourse with the daughters of his common law spouse After a period of counselling, the complainants decided that charges against the accused should be pressed At the outset of the trial, the accused moved pursuant to s. 24 of the Charter for a stay of proceedings The trial judge found that the accused's rights under s. 11(b) and (d) of the Charter had not been violated so that there was no basis on which a remedy could be However, he went on to hold that to proceed against the accused in the circumstances would constitute an abuse of process and he invoked his inherent jurisdiction to order a stay of proceedings Issue: Did the trial judge err in staying the proceedings? Holding: Yes; appeal allowed Arbour JA The burden is on an accused to prove the abuse of process on a balance of probabilities The accused must show that allowing the state to proceed against him would violate the community's sense of fair play and decency or that his trial would be an oppressive proceeding Here, the trial judge held that it was offensive to the principles of fair play and decency to allow the complainants to change their minds three years after the police had informed the accused that charges would not be laid, especially since the accused had heeded the police warning not to contact them The police decision not to lay charges against the accused in 1984 could not be faulted, considering the complainants' reluctance to proceed The conduct of the complainants, likewise, could not be faulted 175

Absent prejudice to the accused or other circumstances leading to unfairness, the complainants willingness to lay charges at 22 offended neither common sense nor fair play To hold otherwise would require young victims of sexual abuse to speak up immediately or never be heard Several courts have found it to be an abuse of process for the Crown to renege on an agreement with the accused Typically, those cases involved "deals" in which the accused had genuinely compromised her position and made a real concession in anticipation of some reward, such as the abandonment of prosecution, so that the accused was prejudiced by the Crown reneging on the deal In this case, even if the discussion between the accused and the police officer in which the accused was warned to stay away from the family and told that no charges would be laid could be characterized as an agreement, the accused made no real concession and did not turn over evidence or in any way compromise his position He was not facing more serious charges now than he would have faced in 1984 There was no evidence that his life was disrupted, that his reputation had to be restored or that his working life was interfered with in the intervening period There was no evidence of trickery, improper motives or malice on the part of the police or the complainants The potential prejudice to the accused in making full answer and defence could be resolved at trial Law Reform Commission of Canada. (1975). Working paper no. 15: Criminal procedure: Control of the process Plea bargaining is now an established practice in Canada Much of the controversy surrounding it results from disagreement as what the practice is Here it is defined as a guilty plea by the accused in return for the promise of some benefit The plea bargaining is made possible by the accuseds right to plead guilty, while relinquishing her right to trial Objections to plea bargaining are overwhelming It detracts from the pursuit of legitimate goals of the criminal justice system, and destroys the appearance and reality of justice As long as it exist, the parties will adopt tactics to maximize their bargaining power Neither the public interest nor the interests of the parties can be properly served It is generally conduct in secret Justice should not be seen as something that can be purchased or negotiated at the bargaining table Plea bargaining is supported by some as a desirable method of achieving compassion and flexibility 176

It may a way to maintain a high proportion of guilty please o However, it is possible that the causal connection between plea bargaining and guilty pleas is exaggerated There are other ways to encourage guilty please The parties would be act more realistically if they didnt need to prepare for negotiations

Judicial participation in the plea bargaining is rare The commission thus recommends that plea bargaining be eliminated Guest Speakers Presentation Interests of the Crown in plea bargaining Protection of society and the public o Is the accused likely to reoffend? o Was there contribution of the victim to the wounds that she/he received? Within this, the interests of the victim and the interest of justice There is no case to win for the Crown, and one has to keep an open mind to the view of the defence If the accused is not represented, the Crown is very aware of not abusing its power during the bargaining process When the Crown authorizes a file it has already taken into consideration the evidence against the accused and the possibility of securing a conviction At the point of the preliminary inquiry, if the victim is absent, there is hardly any evidence o There is no person to identify the accused and document the crime The judge should follow a joint-submission from the Crown and the defence, unless it is unreasonable See R. v. GWC or Hollwick Before the 1990s plea agreements were frowned upon and not highly recognized by the courts as a legitimate practice Since then they have been recognized as a legitimate practice and part of the system of criminal justice Without it the system would be impractical The Karla Homolka case brought up many concerns related to plea bargaining o Yet, when the Crown entered the plea agreement for manslaughter and a 12 year sentence, there was almost no evidence against ether her or Paul Bernardo It is often true that pleas are just decisions made by the Crown and the defence in closed chambers, but their decisions are ultimately approved and made legitimate by a judge It is the judge who decided whether the plea is reasonable or not The judge ensures that the accused has made a free and voluntary decisions without fear of threats 177

Unfortunately, the media has given the plea bargaining process a very bad reputation

A guilty plea is a mitigating factor for sentencing purposes This is particularly the case when the accused is an informant of the criminal activities of others An important tool to use during the plea bargaining for the defence is to deal with an offence through a summary conviction instead of an indictable offence Another is to receive concurrent sentences as opposed to consecutive ones o This is appealing for the accused and recognizes the crime committed against each victim When entering the plea bargaining as a defence attorney, the first thing to consider is whether the accused recognizes some level of culpability Otherwise there is not ground to negotiate It is essential to have the consent of the accused to negotiate The criminal past of the accused S. 810 of the Code imposes conditions on the accused to be released in order to protect the victim

The Trial Process: Preliminary Considerations


When the types of decisions that are discussed here are made by the trial judge, appeals to these decisions are made as part of the appeal process of the ultimate verdict

Right to counsel at trial (Charter s. 7)


Before we discussed how there is no right to free counsel under s. 10(b) This deals with the period of arrest The traditional common law right to make full answer and defence is often the tool used by the trial judge to guarantee the right to counsel at trial Express provisions have long be in the Criminal Code (see ss. 650(3), 684 and 802) Barrette v. R., [1977] 2 SCR 121 Facts Charged with assaulting a peace officer the accused was sentenced to imprisonment for one year At his trial the accused filed an application for adjournment because his counsel was not present The judge denied the application on the grounds that the case dated back to the previous November and that counsel, who was occupied elsewhere, had not justified his absence 178

o The accused was therefore directed to proceed without the assistance of counsel The majority of the CA held that the accused, even though not represented by counsel, was given the opportunity to make a full defence and received a fair trial, and refused to order a new trial Issue: Should the accused be directed to proceed without a counsel at trial when his lawyer is unavailable? Holding: No; appeal allowed and a new trial ordered Pigeon J Despite that the fault of counsel constituted prima facie contempt of court, there was nothing which authorized the trial judge to presume the complicity of the accused or without any evidence to lay the blame for the fault of counsel on him The accused has the right to make full defence personally or by counsel (s. 577(3)) An adjournment necessary for the exercise of this right can only be refused for a reason based on established facts In the present case, the accused cannot be held responsible for the fact that too many cases are postponed because lawyers are not present Although the decision on an adjournment necessary for the exercise of this right is in the judges discretion, the judge must exercise this discretion judicially Her decision may thus be reviewed on appeal if it is based on reasons which are not well founded in law This right of review is especially wide when the consequence of the exercise of discretion was that someone was deprived of his rights, whether in criminal or in civil proceedings As to the English judgments where the Court refused to quash the conviction of accused persons deprived of the services of counsel, it must not be overlooked that at that time in England the quashing of a verdict by the CA meant the definitive acquittal of the accused Hence the tendency to uphold a conviction, despite an error of law, if there was no miscarriage of justice It cannot be said in the case at bar that the accused suffered no prejudice by being forced to defend himself without enjoying the assistance of counsel, and without being able to summon as a witness a person having knowledge of the incident which led up to the conviction When the case of the accused is such that she cannot defend herself without testifying, she is certainly in great need of assistance of counsel While it is true that counsel for the prosecution treated the accused with consideration, it cannot be concluded that he had a fair trial De Grandpre J (dissenting) As it must be determined whether a miscarriage of justice was perpetrated by the trial judge in the exercise of his discretion, and his decision was upheld by the CA, this SCC must interfere only if it is clear that the judgment a quo is based on an error of principle 179

The Court of Appeal of England has intervened in cases of this kind only when the fact that the accused was not represented by counsel might have constituted a denial of justice and have modified the result of the trial The accused has not convinced this Court that the presence of his lawyer would have changed the outcome, indeed to the contrary The right to the presence of counsel is a right which has limits, and the administration of justice requires that society be protected as well Deutsch v. Law Society of Upper Canada (1986), 47 CR (3d) 166 Issue: Do individuals possess a right under the Charter to be provided with a funded counsel to defend them at trial against criminal charges for an indictable offence? Holding: No; appeal dismissed Craig J The right to a funded counsel has not been entrenched by s. 7 and/or s. 11(d) of the Charter With the exception of the language provisions, most of the rights guaranteed by the Charter are expressed in negative terms in the sense that they require that the state refrain from certain activities To impose a constitutionally entrenched positive duty on the government to expend public funds on the defence of persons accused of crimes would require a specific guarantee in express language During the trial, the court was made aware of documentation that showed that there was a proposal to amend s. 10 of the Charter to include the obligation to supply counsel for those who could not afford it; this proposed amendment was defeated The Minutes the proceedings of the Special Joint Committee of the Senate and House of Commons on the Constitution of Canada become persuasive authority for the proposition that the Charter was not intended to en trench a right to funded counsel Under the common law the accused has a right to a fair trial and the trial judge is bound to ensure that an accused person receives a fair trial Where the accused faces possible imprisonment, pursuant to s. 7 of the Charter, the accused has an entrenched right not to be deprived of his liberty except in accordance with the principles of fundamental justice Also pursuant to s. 11(d) he has an entrenched right to a "fair and public hearing ..." The right to fundamental justice and a fair and public hearing includes the right to a fair trial There may be rare cases where legal aid is denied to an accused person facing trial, and where the trial judge is satisfied that, because of the seriousness and complexity of the case, the accused cannot receive a fair trial without counsel In such a case it seems to follow that there is an entrenched right to a funded counsel under the Charter 180

Nevertheless, this is not new; it is the same right enjoyed by an accused at common law

It may still be a difficult question for a trial judge to decide whether a funded counsel shall be appointed First, because an accused has an absolute right to proceed without counsel regardless of the complexity of the case Second, where an accused has been denied legal aid without error in law or jurisdiction on the part of the Legal Aid authorities, and/or where the accused refuses to comply with reasonable requirements of those authorities, his appearance in court without counsel may be construed as an exercise by him of his right to proceed without counsel It is unnecessary for the court to deal with administrative details, as these would be dealt with on a case by case basis through the Department of the Attorney General of Ontario Lecture Notes In arriving at the conclusion that right to free counsel is not constitutionally guaranteed, the court makes reference to legislative history and the refusal of Parliament to include this in the Charter despite human rights treaties R. v. Peterman (2004), 19 CR (6th) 258 Facts The accused was charged with two counts of arson Legal Aid Ontario issues him a legal certificate, and a lawyer who was four hours away and had acted for the accused before, agreed to act for the accused on the certificate The lawyers application for travel expenses to Legal Aid was denied given the availability of local counsel The court granted an application brought against Legal Aid to pay for the lawyers travel expenses, preparation time and junior counsel o The trial judge found the application reasonable and required for fair representation o She also ordered costs against the Crown Issue: Should the accused be entitled to receive from Legal Aid travel and other additional expenses for her lawyer? Holding: No; appeal allowed Rosenberg JA The state has a constitutional obligation to ensure that indigent accused receive a fair trial, and in many cases that means ensuring that the accused is represented by counsel Under the Ontario legal aid certificate system, it is because defence lawyers are willing to accept legal aid certificates that the state is able to fulfill its constitutional obligation The importance of the work that defence counsel perform for legal aid clients cannot be underestimated Nevertheless, the obligation for setting legal aid rates and policies relating to retention of out-of-town counsel and of junior or co-counsel lies with Legal Aid Ontario, not the court A criminal trial court has no jurisdiction to review those policies 181

A criminal trial court's jurisdiction rests solely on the obligation to ensure that an accused person receives a fair trial

In some cases, the court will be satisfied that if an accused is not represented by counsel, her right to a fair trial as guaranteed by ss. 7 and 11(d) of the Charter will be infringed If such an accused lacks the means to employ counsel privately, but has nevertheless been refused legal aid, the court can make an order staying the proceedings until the necessary funding for counsel is provided by the state As per Rowbotham "there may be rare circumstances in which legal aid is denied but the trial judge, after an examination of the means of the accused, is satisfied that the accused, because of the length and complexity of the proceedings or for other reasons, cannot afford to retain counsel to the extent necessary to ensure a fair trial" However, when a court makes a Rowbotham order, it is not conducting some kind of judicial review of decisions made by legal aid authorities; rather, it is fulfilling its independent obligation to ensure that the accused receives a fair trial Summary of state obligations regarding a right to counsel The Charter guarantees to a fair trial and fundamental justice mean that the state must provide funds so that an indigent accused can be represented by counsel where counsel is required to ensure that the accused person has a fair trial Further, within reason, the court will protect an accused's right to choose his or her counsel, as per common law principles Absent compelling reasons, such as a disqualifying conflict of interest or incompetence, the courts will not interfere with an accused's choice of counsel Further, the courts will avoid actions that result in accused persons being improperly or unfairly denied the opportunity to be represented by their counsel of choice However, the right of an accused person to be free of unreasonable state or judicial interference in her choice of counsel does not impose a POSITIVE obligation on the state to provide funds for counsel of choice There would appear to be two exceptions to this general proposition 1. In some unique situations it may be that an accused can establish that she can only obtain a fair trial if represented by a particular counsel o In those unusual circumstances, the court may be entitled to make an order to ensure that the accused is represented by that counsel 2. In unusual circumstances, the court may find that the accused simply cannot find competent counsel to represent him or her on conditions imposed by Legal Aid o Those cases will be rare In this case, there was never any dispute that the respondent required counsel to ensure that he had a fair trial and that he could not fully fund it He was therefore granted a contributory legal aid certificate The application judge was, again, not entitled to review the reasonableness of the 182

decisions made by Legal Aid o Her focus had to be on whether the respondent's right to a fair trial was imperiled because of the conditions under which he was being defended o There was no evidence to support a finding that the respondent's right to a fair trial was at risk o The respondent's case is not unique and it is not of an order of complexity that requires special counsel o This was also not a case where the accused could not otherwise obtain competent counsel The application judge could have adjourned the case to permit the respondent time to retain local counsel Alternatively, having regarded the circumstances, the application judge could have required the accuseds lawyer to proceed with the case since he was on the record and had undertaken to defend the respondent Lecture Notes When the court finds that not having a counsel present will lead to an unfair trial, the judge can issue a stay until the state can fund the counsel To prove that a lawyer must be funded by the state the accused must: o First establish that he has been denied legal aid and cannot afford a lawyer o That not having a counsel present will jeopardize the fair trial The court has to consider the seriousness of the charges The length and complexity of the case The ability of the accused to represent herself effectively at trial Courts now frequently acknowledge that the right to make full answer and defence protected under ss. 7 and 11(d) includes a right to effective assistance of counsel However, appeal courts have been reluctant to order a new trial on the basis of incompetence of counsel and often hesitate to second-guess tactical decisions made at trial When this matter reached the SCC in R. v. B.(G.D) the court stated that an appellant was required to establish incompetence and that a miscarriage of justice resulted o Incompetence was assessed based on the Strickland test, which employs a reasonable standard of professional assistance o Miscarriage of justice can take many forms and include procedural unfairness Where an accused in unrepresented, there are important obligations imposed on trial judges See R. v. Tran (2001) and R. v. Moghaddam (2006)

Adjournments
There are numerous provisions scattered across the Code dealing with judges adjournment powers S. 516 permits the adjournment of a bail hearing, and s. 537 the adjournment of a preliminary inquiry

183

With respect to indictable offences, s. 571 (relating to non-jury trials) and s. 645 (for jury trials) allow the judge to adjourn the trial from time to time S. 803 permits an adjournment on a matter of summary conviction

R. v. G. (J.C) (2004), 189 CCC (3d) 1 Facts The accused was charged with robbery and aggravated assault During trial the Crown sought an adjournment, because the complainant (who had been subpoenaed by ordinary mail) was not present in court o The Crown advised the judge that the complainant, who live with her parents, was missing The trial judge refused to grant the adjournment and the accused was acquitted Issue: Should an adjournment be granted when the complainant is unable to appear in court? Holding: Yes; appeal allowed Dalphond JA It is undisputed that whether an adjournment or a postponement should be granted or not is a discretionary matter for the trial judge o Such judicial discretion can however be reviewed on appeal if it has not exercised judicially o The test for appellate review is whether the trial judge has given sufficient weight to all relevant considerations o Of course, if the judgment is based on reasons that are not well founded in law, a court of appeal may also intervene Amongst the elements to be considered by a judge when asked to grant an adjournment of a criminal trial due to the absence of a witness are the following (R. v. Darville) 1. That the absent witness is a material witness in the case 2. That the party applying for an adjournment has been guilty of no laches or neglect in omitting to endeavour to procure the attendance of this witness 3. That there is a reasonable expectation that the witness can be procured at the future time to which it is sought to put off the trial It is also proper for a trial judge when asked for a postponement to consider other relevant circumstances such as the gravity of the charges, the number of previous postponements and the consequences of a postponement for the accused A trial judge errs in law by refusing a request for an adjournment without having given the party seeking it an opportunity to demonstrate that the conditions described above are met The decision whether or not to grant the adjournment must be made in the light of the realities of each case and shall be consistent with the interests of justice In the case at bar, the first criterion mentioned in Darville was met; her presence was critical to establish the case against the respondent As for the second criterion, whether the Crown negligent, if the trial judge was unsatisfied with the Crowns recital of the facts he should have said so and then ordered an 184

adjournment to provide the Crown the opportunity to bring before him proper evidence of what really happened o There ought to be no presumption that using a mailed subpoena constituted neglect or that it did not There was no evidence made before the trial judge that if the complainant had been personally served with a subpoena, she would not have run away o The Crown which had just found out about the disappearance of the complainant, could not then be blamed by the trial judge for not knowing why she was missing o The third criterion is the most difficult, but the trial judge erred in law by refusing the Crown's request for an adjournment without having given it an opportunity to demonstrate that this condition was met All that being said, was it nevertheless in the interests of justice to order an acquittal in this case? o No: the charges were serious, there was no prior postponement and the defence attorney did not mention any specific adverse consequence for the accused

Hilton JA (dissenting) The trial judge did not abuse his discretion in disposing of the case in the manner he did In this case, the trial judge focussed in the main on what he perceived to be the Crown's negligence in not having taken adequate steps to ensure the presence of the victim No one can seriously challenge that the Crown acts in accordance with the law when it sends a subpoena by ordinary mail o Undoubtedly, that practice may work in many instances, but it can hardly be doubted that using the ordinary mail to reach a 16-year old girl with a "particular" lifestyle and then not following up to insure the subpoena was received was conduct replete with risk As for the gravity of the charges, this is a two-way street The Crown did not consider that the gravity of the charges against the accused warranted any particular consideration when it came time to compel the attendance of its only witness to the trial The gravity of the charges also weighs on the accused, who had pleaded not guilty to them, and whose liberty was at risk once he had entered a plea of not guilty In addition, no party, be it the Crown or the accused, is entitled to a postponement simply because one has not been previously requested

Trial within a reasonable time (Charter s. 11(b))


Motions to stay proceedings for unreasonable delay are generally made to the trial judge, at the outset of trial

185

The judge must make a decision on unreasonable delay in light of the way the case has proceeded from the time of charge

s. 11(b) Canadian Charter of Rights and Freedoms Proceedings in criminal and penal matters Any person charged with an offence has the right ... b) to be tried within a reasonable time; The abuse of process is not created by the delay itself, but in some cases, the circumstances surrounding the delay Unreasonable delay has a different test than abuse of process Factors to consider when determining whether there has been an unreasonably long delay Length of delay Reasons for the delay o Conduct of the Crown o Systemic of institutional factors, like lack of resources The lack of institutional facilities, however, cannot be used to render the meaning of 11(d) meaningless The burden is on the Crown to show that institutional factors are at play Morin seems to focus less on burdens, and in some questions they are placed on the accused o The conduct of the accused An accused should not be allowed to deliberately prolong a case with dilatory applications If the delay acts in favour of the accused, even if this was not planned, it must be taken into account Prejudice to the accused A waiver of behalf of the accused o This is dealt in a similar way as the right to counsel The accused does not need to assert the right Any waiver must be clear and unequivocal in relation to the rights being waived o While the waiver can be implicit as of Askov and Morin, the accused must understand the consequences of the waiver R. v. Askov, [1990] 2 SCR 1199 Facts The accused were charged with conspiracy to commit extortion All counsel agreed on a date early in July 1984 for the preliminary hearing, but it could not be completed until September 186

A trial was then set for the first available date, in October 1985 The case could not be heard during that session, and was put over for trial to almost two years after the preliminary hearing When the trial finally began, appellants moved to stay the proceedings on the ground that the trial had been unreasonably delayed The trial judge found that the major part of the delay following appellants' committal stemmed from institutional problems and granted the stay The CA found: (1) no misconduct on the part of the Crown; (2) no indication of any objection by the appellants to any of the adjournments; and (3) no evidence of any actual prejudice to the appellants. It accordingly set aside the stay and directed that the trial proceed Issue: Should a stay be granted because of judicial delay? Holding: Yes; appeal allowed Cory J The right to be tried within a reasonable time, like other specific s. 11 guarantees, is primarily concerned with an aspect of fundamental justice guaranteed by s. 7 The primary aim of s. 11(b) is to protect the individual's rights and to protect fundamental justice for the accused A community or societal interest, however, is implicit in the section in that it ensures, first, that law breakers are brought to trial and dealt with according to the law and, second, that those on trial are treated fairly and justly A quick resolution of the charges also has important practical benefits, since memories fade with time, and witnesses may move, become ill or die Victims, too, have a special interest in having criminal trials take place within a reasonable time, and all members of the community are entitled to see that the justice system works fairly, efficiently and with reasonable dispatch The court should consider a number of factors in determining whether the delay in bringing the accused to trial has been unreasonable 1. The length of the delay 2. The explanation for the delay 3. Waiver 4. Prejudice to the accused The longer the delay, the more difficult it should be for a court to excuse it, and very lengthy delays may be such that they cannot be justified for any reason Delays attributable to the Crown will weigh in favour of the accused Complex cases, however, will justify delays longer than those acceptable in simple cases Systemic or institutional delays will also weigh against the Crown When considering delays occasioned by inadequate institutional resources, the question of how long a delay is too long may be resolved by comparing the questioned jurisdiction to others in the country 187

The comparison of similar and thus comparable districts must always be made with the better districts, not the worst

In all cases it will be incumbent upon the Crown to show that the institutional delay in question is justifiable Certain actions of the accused, on the other hand, will justify delays A waiver by the accused of his rights will justify delay, but the waiver must be informed, unequivocal and freely given to be valid Here, the delay of almost two years following the preliminary hearing was clearly excessive and unreasonable The Crown did not show that the delay did not prejudice the appellants, and nothing in the case was so complex or inherently difficult as to justify a lengthy delay The height of allowing stays for delays came in the aftermath of Askov Justice Arbour for the Ontario CA noted in R. v. Bennet (1991) that in one six-month period following Askov 34,000 charges had been stayed, dismissed or withdrawn Justice Cory stated in a conference in 1991 that while he knew Askov would have an impact, he never imagined the extent of this decision The purpose of the right to be tried within a reasonable time includes the right to security of the person (see Mills) There is also an increasing recognition to other matters like a fair trial In Morin some of the issues that were considered were Delays inherent to the case Actions of the accused Actions of the Crown Limits on institutional resources o There is some obligation on the Crown to commit resources to avoid an unreasonable delay, but one has to be careful when relying in unfair comparisons across different jurisdictions o The rapid and increasing population could be a reason to consider why a delay is justified under some circumstances While some case presumptions of unreasonable delay being due to prejudice are virtually irrebuttable o Morin sees no burden on the Crown at all o A lack of prejudice can act to justify deviations of the 10-month rule o What the court used to justify the lack of prejudice was the Crowns proposal to the accused to have the trial moved so that it would take place in two months, and her refusal was taken as an inference that she suffered no prejudice The case also brought up the issue that delays can in some cases act in favour of the accused o Many accused individuals act so as to avoid having a speedy trial 188

The majority also claims that it was able to answer the question of this appeal without resorting to burdens of proof

R. v. Morin, [1992] 2 SCR 771 Facts The accused was charged with impaired driving and with operating a motor vehicle while having a blood alcohol level which exceeded the legal limit She was released from custody that same day on a promise to appear When she appeared in Provincial Court, her counsel explicitly requested "the earliest possible trial date On her scheduled trial date the accused brought a motion to stay the proceedings pursuant to s. 24(1) of the Charter, arguing that the 14-month delay in bringing her to trial infringed her right to be tried within a reasonable time under s. 11(b) of the Charter The motion was dismissed and the accused was convicted on the "over 80" charge A stay was entered with respect to the impaired driving charge for unrelated reasons On appeal, the summary conviction appeal court also stayed the "over 80" charge on the basis that the accused had not been tried within a reasonable time The CA allowed the Crown's appeal and restored the conviction. Issue: Should a stay be granted because of judicial delay? Holding: No; appeal dismissed Sopinka J The primary purpose of s. 11(b) is the protection of the individual rights of accused persons: (1) the right to security of the person, (2) the right to liberty, and (3) the right to a fair trial The right to security of the person is protected by seeking to minimize the anxiety, concern and stigma of exposure to criminal proceedings The right to liberty is protected by seeking to minimize exposure to the restrictions on liberty which result from pre-trial incarceration and restrictive bail conditions The right to a fair trial is protected by attempting to ensure that proceedings take place while evidence is available and fresh. A secondary interest of society as a whole has also been recognized This interest is most obvious when it parallels that of the accused: society as a whole has an interest in seeing that citizens who are accused of crimes are treated humanely and fairly There is, as well, a societal interest that is by its very nature adverse to the interests of the accused: there is a collective interest in ensuring that those who transgress the law are brought to trial and dealt with according to the law The general approach to a determination of whether the s. 11(b) right has been denied is by a judicial determination balancing the interests which the section is designed to protect against factors which inevitably lead to delay The factors to be considered are (1) Length of the delay 189

(2) Waiver of time periods (3) Reasons for the delay, including (a) Inherent time requirements of the case (b) Actions of the accused (c) Actions of the Crown (d) Limits on institutional resources (e) Other reasons for delay (4) Prejudice to the accused The period to be scrutinized is the time elapsed from the date of the charge to the end of the trial If by agreement or conduct the accused has waived any part of this time period, the length of the period of delay will be reduced accordingly. An inquiry into unreasonable delay is triggered by an application under s. 24(1) of the Charter While the applicant has the legal burden of establishing a Charter violation, an evidentiary burden of putting forth evidence or argument on particular factors will shift depending on the circumstances of each case An inquiry into unreasonable delay should only be undertaken if the period is of sufficient length to raise an issue as to its reasonableness A case will only be decided by reference to the burden of proof if the court cannot come to a determinate conclusion on the facts presented to it . As well as the complexity of a case, all cases are subject to certain intake requirements and some cases must pass through a preliminary inquiry before reaching trial The court will also need to consider whether the actions of either the accused or the Crown have led to delay In considering the explanation for delay, account must be taken of the limits of institutional resources Institutional delay runs from the time the parties are ready for trial and continues until the system can accommodate the proceedings It is appropriate for this Court to suggest a guideline of between 8 and 10 months for institutional delay in Provincial Courts A guideline with respect to institutional delay after committal for trial in the range of 6 to 8 months was suggested in R. v. Askov, and is still apposite The application of the guideline will be influenced by the presence or absence of prejudice; the greater the prejudice, the shorter the acceptable period of institutional delay Prejudice may be inferred from the length of the delay The longer the delay, the more likely that such an inference will be drawn In this case the delay of 14 months is sufficient to raise the issue of reasonableness 190

In the jurisdiction in which this case arose, a period in the order of 10 months would not be unreasonable for systemic delay given the rapidly changing local conditions The accused led no evidence of prejudice and little or no prejudice is inferred from the delay as the accused appeared to be content with the pace of litigation In view of the strain on institutional resources and the absence of any significant prejudice to the accused, the delay in this case was not unreasonable

Is it fair to require accused individuals to prove prejudice? Would this be placing an unreasonable burden on individuals who are most sensitive Since Morin, very few s. 11(b) challenges have succeeded Although the case seems to re-state Askov, there are at least four important differences o The more serious the charge the less likely it will be stayed o The Crown no longer has the burden of proving that delay was caused by the accused, that institutional delay was justified, that there was no prejudice, or that the accused had waived her s. 11 (b) right o The comparative jurisdiction test is less important o Whether the accused has been prejudices is crucial

Formal attacks on the information or indictment


In provincial courts, charges are always contained in informations (regardless whether they are tried as indictable offences or summary convictions) If the trial is by indictment, the information is treated for all purposes as an indictment (see ss. 2 and 554(4)) In the case of superior courts, the information which began the process is replaced by an indictment (s. 566) s. 554 Criminal Code Trial by provincial court judge with consent (1) Subject to subsection (2), if an accused is charged in an information with an indictable offence other than an offence that is mentioned in section 469, and the offence is not one over which a provincial court judge has absolute jurisdiction under section 553, a provincial court judge may try the accused if the accused elects to be tried by a provincial court judge. s. 566 Criminal Code Indictment (1) The trial of an accused for an indictable offence, other than a trial before a provincial court judge, shall be on an indictment in writing setting forth the offence with which he is charged. Preferring indictment 191

(2) Where an accused elects under section 536 or re-elects under section 561 to be tried by a judge without a jury, an indictment in Form 4 may be preferred. What counts may be included and who may prefer indictment (3) Section 574 and subsection 576(1) apply, with such modifications as the circumstances require, to the preferring of an indictment pursuant to subsection (2). The allegations provided in the information or indictment must be proved beyond a reasonable doubt The document must give the accused adequate notice of what the burden of the case she has to meet In any trial there is only one information or indictment for each accused, though the document may contain more than one accused and more than one count A formal attach may be against one or all the counts A formal objection is sometimes introduced by general language such as a motion to quash for a formal defect, that the charge is void for uncertainty or that the charge does not name an offence known to law However, it is better to be more precise and distinguish between these types of objections The three reasons to challenge the indictment are Insufficiency Duplicity Improper joinder o Two offences which arose of different circumstances should not be joined together as one could taint the trier of fact on the other o Normally, prior offences will not be admissible at trial unless for similar fact evidence In the case of summary convictions, the proceedings of a successful formal objection may act as a new defence, since there is a time limitation of six months

A) Grounds
1. Insufficiency A charge in an indictment will be insufficient if it does not comply with the requirements se in ss. 581 and 583 of the Code s. 581 Criminal Code Substance of offence (1) Each count in an indictment shall in general apply to a single transaction and shall contain in 192

substance a statement that the accused or defendant committed an offence therein specified. Form of statement (2) The statement referred to in subsection (1) may be (a) in popular language without technical averments or allegations of matters that are not essential to be proved; (b) in the words of the enactment that describes the offence or declares the matters charged to be an indictable offence; or (c) in words that are sufficient to give to the accused notice of the offence with which he is charged. Details of circumstances (3) A count shall contain sufficient detail of the circumstances of the alleged offence to give to the accused reasonable information with respect to the act or omission to be proved against him and to identify the transaction referred to, but otherwise the absence or insufficiency of details does not vitiate the count. Indictment for treason (4) Where an accused is charged with an offence under section 47 or sections 49 to 53, every overt act that is to be relied on shall be stated in the indictment. Reference to section (5) A count may refer to any section, subsection, paragraph or subparagraph of the enactment that creates the offence charged, and for the purpose of determining whether a count is sufficient, consideration shall be given to any such reference. General provisions not restricted (6) Nothing in this Part relating to matters that do not render a count insufficient shall be deemed to restrict or limit the application of this section. s. 583 Criminal Code Certain omissions not grounds for objection No count in an indictment is insufficient by reason of the absence of details where, in the opinion of the court, the count otherwise fulfils the requirements of section 581 and, without restricting the generality of the foregoing, no count in an indictment is insufficient by reason only that (a) it does not name the person injured or intended or attempted to be injured; (b) it does not name the person who owns or has a special property or interest in property mentioned in the count; (c) it charges an intent to defraud without naming or describing the person whom it was intended to defraud; 193

(d) it does not set out any writing that is the subject of the charge; (e) it does not set out the words used where words that are alleged to have been used are the subject of the charge; (f) it does not specify the means by which the alleged offence was committed; (g) it does not name or describe with precision any person, place or thing; or (h) it does not, where the consent of a person, official or authority is required before proceedings may be instituted for an offence, state that the consent has been obtained. The same provisions apply to summary conviction proceedings (s. 795) s.795 Criminal Code Application of Parts XVI, XVIII, XX and XX.1 The provisions of Parts XVI and XVIII with respect to compelling the appearance of an accused before a justice, and the provisions of Parts XX and XX.1, in so far as they are not inconsistent with this Part, apply, with such modifications as the circumstances require, to proceedings under this Part. These provision strike a compromise between allowing enough notice to accused individuals and avoiding undue technicalities A motion to quash based on a defect apparent on the face of an indictment or information must be brought before the plea or thereafter only be leave of the court (s. 601(1)) s. 601(1) Criminal Code Amending defective indictment or count (1) An objection to an indictment or to a count in an indictment for a defect apparent on the face thereof shall be taken by motion to quash the indictment or count before the accused has pleaded, and thereafter only by leave of the court before which the proceedings take place, and the court before which an objection is taken under this section may, if it considers it necessary, order the indictment or count to be amended to cure the defect. Brodie v. R., [1936] SCR 188 The charge must specify the time, the place and the matter People should have sufficient notice so that they can prepare their case R. v. McKenzie, [1972] 2 SCR 409 Facts The respondent was employed to drive one of his employers taxis upon terms which entitled him to retain 45 per cent of the daily receipts and required him to account for the balance, less the cost of gasoline and oil, to his employer In accounting for a trip in which he picked up five passengers bound for different addresses, he only recorded one trip for one passenger He was charged that he did commit theft of the approximate sum of $16.50 the 194

property of Dominic Louis Christian contrary to the form of the statute in such case made and provided. He was convicted of theft by the trial judge, but acquitted by a majority judgment of the Court of Appeal o That Court expressed the view that this form of charge lacked any averment of the essential ingredients of an offence Issue: Was the charge void for insufficiency because it lacked the essential ingredients of the offence? Holding: No; appeal allowed Ritchie J The provisions of s. 492(2) (b) are to be read disjointly and envisage two alternative methods in which an indictment may be phrased so that a charge may be laid either in the words of the enactment that describe the offence or those of the enactment which declares the matters charged to be an indictable offence The charge in the present case complied with the Criminal Code in that it was in the words of s. 280 (now 334) which declares theft to be an indictable offence The respondents failure to account to his employer was theft within the meaning of s. 276(1) (now s. 330(1)) If there had been any doubt as to the conduct to which the charge related, further particulars describing the means by which the offence was alleged to have been committed could have been sought under s. 497(1) (f) Lecture Notes The Brodie requirements already appeared too onerous to the courts Ritchie J o In the Code of the time there were three definitions of offences o The case could be distinguished from Brodie o The indictment must not necessarily state which particular definition of theft he was facing (in terms of the section in the Code) as it was sufficiently described in the indictment R. v. Ct, [1978] 1 SCR 8 Facts The accused was charged with failing to provide a breath sample, the words "without reasonable excuse" being omitted from the information The CA quashed his conviction on the bases that information was incomplete Issue: Was the information for insufficiency because it lacked the terms without reasonable excuse? Holding: No; appeal allowed De Grandpr J The Crown submits that the words "without reasonable excuse" were brought to the attention of the accused by the specific reference to the section of the Criminal Code creating 195

the offence The golden rule is for the accused to be reasonably informed of the transaction alleged against her, thus giving her the possibility of a full defence and a fair trial When, as in the present case, the information recites all the facts and relates them to a definite offence identified by the relevant section of the Code, it is impossible for the accused to be misled To hold otherwise would be to revert to the extreme technicality of the old procedure R. v. Wis Development Corporation Ltd. et al, [1984] 1 SCR 485 Facts The accused were charged with 32 summary conviction offences relating to the breaches of the Aeronautics Act and the regulations issued thereunder Following the Crowns refusal to provide particulars, respondents, prior to plea, made an application to quash the information containing those counts A Provincial Court judge allowed the motion ruling that it did not satisfy the requirements of s. 510(3) of the Code He also ruled that this was not a case where particulars should be ordered The Court of Queens Bench and the Court of Appeal upheld the quashing Issue: Was the information void because it lacked sufficient details? Holding: Yes; appeal dismissed Lamer J The court agreed with the accused arguments expressed in their factum: The operation of a commercial air service within the meaning of s. 9(1) of the Aeronautics Act could therefore relate to a multitude of activities or usages of aircraft in Canada, for example the use of an aircraft to haul passengers or freight, the use of an aircraft as a demonstrator by a dealer or manufacturer, or indeed even the use of the aircraft in a photo session for the purposes of advertisement of another product such as liquor or cigarettes The statute under which the information at bar has been laid casts a broad net and the prohibition is directed at many diverse and unrelated uses of aircraft Lecture Notes It is important to distinguish a voidable information or indictment (which can still be remedied) and one which is void, which cannot Here the court found that the information was ab initio vitiated for want of sufficient details R. v. B. (G.), [1990] 2 SCR 30 Facts Each of the appellants, who are young offenders, was charged with sexually assaulting the complainant, a fellow elementary school student who was seven years old at the time of the alleged offence The complainant was sworn in by the trial judge and testified that the assault took place 196

sometime during the wintertime when she was in grade one The complainant's mother testified that her daughter experienced bed-wetting and nightmares during the late months of 1985 and the early months of 1986 The trial judge concluded that if the event did take place, its date had not been established o He noted that if the offence had taken place while the complainant was in grade one, it would have had to occur a year earlier than alleged o He found that the date is an essential element of the offence and refused to amend the information as requested by the Crown o Since one of the main ingredients of the offence had not been established beyond a reasonable doubt, the appellants must be acquitted The Court of Appeal found that the trial judge erred in failing to find that the date had been established with sufficient particularity and in failing to amend the information in light of the evidence presented at trial o It set aside the acquittals and ordered a new trial Issue: Was the information void because of the date had not been established with sufficient particularity? Holding: No; appeal dismissed Wilson J An information or indictment must provide an accused with enough information to enable her to defend the charge While time must be specified, the exact time need not be identified or proved In this case the information provided was adequate, having regard to the nature of the offence charged and the age of the victim Under s. 529(4.1) of the Criminal Code, a variance between the indictment and the evidence is not material with respect to the time of commission of the offence The common law had developed a similar rule: if the time specified in the information conflicts with the evidence and time is not an essential element of the offence or crucial to the defence, the variance is not material and the information need not be quashed Further, the Crown need not prove the alleged date unless time is an essential element of the offence, as when an accused defends the charge by providing evidence of an alibi for the date or time period alleged If time was not an essential element in the case, then a conviction could result even though the time of the offence is not proven Had the trial judge addressed this question, he would have been forced to conclude that time was not an essential element of the offence or crucial to the defence The Court of Appeal was therefore correct in holding that the time of the offence did not need to be proven beyond a reasonable doubt in the circumstances of this case. 2. Duplicity Duplicity is really another aspect of sufficiency 197

The common law against duplicity prohibits alternative charges in a single count There is nothing duplicitous about alternative counts A duplicitous charge is confusing to the accused and prejudices a fair defence In the case of indictments the relevant provisions are ss. 581(1) of the Code and the apparently contradictory s. 590(1) s. 581(1) Criminal Code Substance of offence Each count in an indictment shall in general apply to a single transaction and shall contain in substance a statement that the accused or defendant committed an offence therein specified. s. 590(1) Criminal Code Offences may be charged in the alternative A count is not objectionable by reason only that (a) it charges in the alternative several different matters, acts or omissions that are stated in the alternative in an enactment that describes as an indictable offence the matters, acts or omissions charged in the count; or (b) it is double or multifarious. In the case of summary convictions the relevant provision in s. 789(1) (although s. 581(1) and (6) still apply) s. 789(1) Criminal Code Formalities of information (1) In proceedings to which this Part applies, an information (a) shall be in writing and under oath; and (b) may charge more than one offence or relate to more than one matter of complaint, but where more than one offence is charged or the information relates to more than one matter of complaint, each offence or matter of complaint, as the case may be, shall be set out in a separate count. s. 581 Criminal Code Substance of offence (1) Each count in an indictment shall in general apply to a single transaction and shall contain in substance a statement that the accused or defendant committed an offence therein specified. ...

198

General provisions not restricted (6) Nothing in this Part relating to matters that do not render a count insufficient shall be deemed to restrict or limit the application of this section. A motion to quash with respect to a defect that is apparent on the face of the charge must be brought before the plea, and thereafter with leave of teh Court R. v. Sault Ste. Marie, [1978] 2 SCR 1299 Facts The accused City was charged for polluting under s. 32(1) of The Ontario Water Resources Commission Act In dismissing the charge against the City the trial judge found that the City had nothing to do with the actual operations, that the company was an independent contractor and that its employees were not employees of the City On appeal by trial de novo the judge found that the offence was one of strict liability and he convicted The Divisional Court set aside the charge as duplicitous, but the CA rejected this claim Issue: Was the information void for duplicity? Holding: No; appeal dismissed Dickson J The primary test for duplicity should be the practical one based on the only valid justification for the rule against duplicity, the requirement that the accused know the case he has to meet and be not prejudiced in the preparation of his defence by ambiguity in the charge In this case there was nothing ambiguous or uncertain in the charge, the City knew the case it had to meet Section 32(1) of The Ontario Water Resources Commission Act is concerned with only one matter, pollution and that is the gist of the charge and the evil against which the offence is aimed 3. Improper joinders of Counts One needs to be careful that two unrelated offences are not tried together, as the merits of one charge may influence the trier of fact on the other Nonetheless, it make sense to join charges to save judicial resources under some circumstances S. 590(3) gives the court a wide jurisdiction to order that the count in an indictment be divided into two or more counts s. 590(3) Criminal Code Order

199

The court may, where it is satisfied that the ends of justice require it, order that a count be amended or divided into two or more counts, and thereupon a formal commencement may be inserted before each of the counts into which it is divided. The same wide discretion is given to the trial judge to grant a motion before or during trial to separate the trial of an accused upon one of more counts (s. (591(3)) s. 591(3) Criminal Code Severance of accused and counts The court may, where it is satisfied that the interests of justice so require, order (a) that the accused or defendant be tried separately on one or more of the counts; and (b) where there is more than one accused or defendant, that one or more of them be tried separately on one or more of the counts. R. v. Racco (No. 1) (1975), 29 CRNS 303 (Ont. Co. Ct) Facts The accused was charged in Count 1 of the indictment for possession of counterfeit money without lawful justification or excuse o He had in his possession seven counterfeit United States $20 federal reserve notes He was also charged in Count 2 that at the same time and the same place, without lawful excuse, he had in his possession an explosive substance, one ground burst projectile simulator, for other than a lawful purpose The accused asked for a severance Issue: Should the counts of possession of counterfeit money and possession of an explosive substance be severed? Holding: No Graburn J The accused argued that notwithstanding any direction that this Court would give in relation to considering the evidence pertinent to each count separately, a jury would be unconsciously influenced by one count on considering the other Particularly, since the issue in relation to both counts is one of credibility, as the accused argues that both the money and the explosive were planted by the police Where the issue is substantially credibility and where there is a close nexus in time and place in relation to the counts as exists in this case, it is in the interests of justice that both of these counts be tried together The interests of justice require that this issue of a plant by the police be resolved by one tribunal and not by two tribunals To permit a severance here could lead to the following undesirable resul:.a jury on a trial of Count 1 could have a reasonable doubt whether the counterfeit money has been planted and acquit the accused., while a different jury might come to a different conclusion in relation to the second count 200

In the light of the identical nature of the defence to both offenses, such a result is not in the interests of the administration of justice and is not a criterion on which a court should grant a severance

As to the difficulty of the jury being influenced on one count by the evidence relating to the other, the judge felt it was possible to adequately instruct the jury so as to negate this danger

B) Saving devices
1. Particulars In response to the attack for insufficiency, the court may demand that the Crown furnish particular as per s. 587 of the Code Alternatively, particulars might come from informal interactions between the parties If the informal process is considered satisfactory by the defence, it is imperative that these particulars are place on the court record for the protection of the accused o This is because the Crown is bound by particulars in proving its charge s. 587 Criminal Code What may be ordered (1) A court may, where it is satisfied that it is necessary for a fair trial, order the prosecutor to furnish particulars and, without restricting the generality of the foregoing, may order the prosecutor to furnish particulars (a) of what is relied on in support of a charge of perjury, the making of a false oath or a false statement, fabricating evidence or counselling the commission of any of those offences; (b) of any false pretence or fraud that is alleged; (c) of any alleged attempt or conspiracy by fraudulent means; (d) setting out the passages in a book, pamphlet, newspaper or other printing or writing that are relied on in support of a charge of selling or exhibiting an obscene book, pamphlet, newspaper, printing or writing; (e) further describing any writing or words that are the subject of a charge; (f) further describing the means by which an offence is alleged to have been committed; or (g) further describing a person, place or thing referred to in an indictment. Regard to evidence (2) For the purpose of determining whether or not a particular is required, the court may give consideration to any evidence that has been taken. Particular (3) Where a particular is delivered pursuant to this section, (a) a copy shall be given without charge to the accused or his counsel; 201

(b) the particular shall be entered in the record; and (c) the trial shall proceed in all respects as if the indictment had been amended to conform with the particular. Thatcher v. R. (1984), 42 CR (3d) 259 Facts The accused was charged with first degree murder At the preliminary inquiry, evidence was adduced showing that the accused was either guilty of murder or of aiding and abetting a murder The accused applied pursuant to s. 516(1)(f) of the Code for an order for particulars describing the means by which he was alleged to have committed the offence as charged Issue: Should an order for particulars be granted to describe the means by which the accused committed a murder? Holding: No; appeal allowed for other reasons Maher J Section 516 empowers the court to order the Crown to furnish particulars where necessary for a fair trial and where necessary to enable the accused to make a proper defence to the charge against him In this case particulars were not needed for those purposes but were sought to fetter the prosecution's conduct of the case If there was evidence upon which a properly instructed jury could find the accused guilty of murder or that he abetted a murder, it had to be left to the jury to make either of such findings The right to do so could not be restricted by an order for particulars 2. Amendment A trial judge has a wide discretion to amend an indictment or information (ss. 601 and 795) See also ss. 590(2) and (3) s. 601 Criminal Code Amending defective indictment or count (1) An objection to an indictment or to a count in an indictment for a defect apparent on the face thereof shall be taken by motion to quash the indictment or count before the accused has pleaded, and thereafter only by leave of the court before which the proceedings take place, and the court before which an objection is taken under this section may, if it considers it necessary, order the indictment or count to be amended to cure the defect. Amendment where variance (2) Subject to this section, a court may, on the trial of an indictment, amend the indictment or a count therein or a particular that is furnished under section 587, to make the indictment, count or 202

particular conform to the evidence, where there is a variance between the evidence and (a) a count in the indictment as preferred; or (b) a count in the indictment (i) as amended, or (ii) as it would have been if it had been amended in conformity with any particular that has been furnished pursuant to section 587. Amending indictment (3) Subject to this section, a court shall, at any stage of the proceedings, amend the indictment or a count therein as may be necessary where it appears (a) that the indictment has been preferred under a particular Act of Parliament instead of another Act of Parliament; (b) that the indictment or a count thereof (i) fails to state or states defectively anything that is requisite to constitute the offence, (ii) does not negative an exception that should be negatived, (iii) is in any way defective in substance, and the matters to be alleged in the proposed amendment are disclosed by the evidence taken on the preliminary inquiry or on the trial; or (c) that the indictment or a count thereof is in any way defective in form. Matters to be considered by the court (4) The court shall, in considering whether or not an amendment should be made to the indictment or a count in it, consider (a) the matters disclosed by the evidence taken on the preliminary inquiry; (b) the evidence taken on the trial, if any; (c) the circumstances of the case; (d) whether the accused has been misled or prejudiced in his defence by any variance, error or omission mentioned in subsection (2) or (3); and (e) whether, having regard to the merits of the case, the proposed amendment can be made without injustice being done. Variance not material (4.1) A variance between the indictment or a count therein and the evidence taken is not material with respect to (a) the time when the offence is alleged to have been committed, if it is proved that the indictment was preferred within the prescribed period of limitation, if any; or (b) the place where the subject-matter of the proceedings is alleged to have arisen, if it is proved that it arose within the territorial jurisdiction of the court. Adjournment if accused prejudiced (5) Where, in the opinion of the court, the accused has been misled or prejudiced in his defence by 203

a variance, error or omission in an indictment or a count therein, the court may, if it is of the opinion that the misleading or prejudice may be removed by an adjournment, adjourn the proceedings to a specified day or sittings of the court and may make such an order with respect to the payment of costs resulting from the necessity for amendment as it considers desirable. Question of law (6) The question whether an order to amend an indictment or a count thereof should be granted or refused is a question of law. Endorsing indictment (7) An order to amend an indictment or a count therein shall be endorsed on the indictment as part of the record and the proceedings shall continue as if the indictment or count had been originally preferred as amended. Mistakes not material (8) A mistake in the heading of an indictment shall be corrected as soon as it is discovered but, whether corrected or not, is not material. Limitation (9) The authority of a court to amend indictments does not authorize the court to add to the overt acts stated in an indictment for high treason or treason or for an offence against any provision in sections 49, 50, 51 and 53. Definition of court (10) In this section, "court" means a court, judge, justice or provincial court judge acting in summary conviction proceedings or in proceedings on indictment. Application (11) This section applies to all proceedings, including preliminary inquiries, with such modifications as the circumstances require. R. v. Moore, [1988] 1 SCR 1097 Facts Accused was charged with eight counts of theft and possession of stolen property The information on two counts lacked the averment of an essential element of the offence The accused entered pleas of not guilty to the charges against him and elected to be tried by a provincial court judge without a jury The trial judge and counsel agreed that it was not possible to strike the pleas and that, since two of the counts simply failed to allege an offence rather than alleging an offence improperly, it was not possible to amend them at all A new information containing the necessary words omitted from the earlier information 204

was subsequently sworn Respondent's pleas of autrefois acquit were refused by the second trial judge who convicted on one of the counts The Court of Appeal allowed the respondent's appeal from conviction Issue: Does quashing an information, after plea, for failure to allege a material averment constitutes a verdict of acquittal for the purpose of pleading autrefois acquit to a new information? Holding: Yes; appeal dismissed Lamer J Section 529 (now 601) grants very wide powers to the trial judge, subject to certain limits, to cure any defect in a charge by amending it The judge can only quash if the required amendment cannot be made without injustice being done and it is a reversible error of law if a judge does so without coming to that conclusion If an amendment, as a matter of law, cannot be made without causing irreparable prejudice, the quashing of the charge at the trial is tantamount to an acquittal Relaying an amended charge before another judge would be no less prejudicial to the accused than the previous judge's amending the first one The judge should not have quashed here for an amendment would not have prejudiced the accused The Crown, however, cannot lay an amended charge once the accused has been acquitted, albeit by error The trial judge's decision is open to appeal The Court of Appeal, assuming error is found, will direct the trial judge to amend and hear the case, or will amend the charge itself and return the matter for trial on the amended charge R. v. Tremblay, [1993] 2 SCR 932 Facts The appellants were charged with keeping a bawdy-house for the purpose of the practice of indecent acts Nude dancers would perform in individual cubicles for their clients and would assume a variety of suggestive positions At trial, the Crown's motion to amend the charge by deleting the words "the practice of indecency", and its subsequent motion to include the words "practice of prostitution" were denied because they would cause serious prejudice to the accused The motions were made late in the trial when virtually all the evidence had been called The Court of Appeal materially amended the charge and entered a conviction on the basis of the amended charge Issue: Should the Crown ne allowed to amend the charge substantially? Holding: No; appeal allowed Cory J By virtue of s. 601 of the Code the courts now possess reasonably wide powers of amendment 205

Yet, it remains an important principle of criminal law that persons accused of a crime must know the charge brought against them in order to present a full answer and defence A court cannot amend an information or indictment where to do so would cause irreparable prejudice Moreover, a court cannot amend an information unless the evidence tendered is capable of supporting such a charge

In the present case it is not necessary to consider whether evidence of prostitution was disclosed during the trial The prejudice that granting the amendment would cause the accused is determinative of the issue When the motion for the amendment was brought it was obvious that the appellants had prepared their defence on the basis that the acts performed were not indecent o In light of the wording of the original charge the appellants quite properly prepared their entire defence on this issue 3. On appeal Motions to quash indictments or information for defects on they face must be brought before an accused has pleaded and thereafter by leave of the court (s. 601(1) Appeal courts are very reluctant to hear such objections for the first time on appeal This is felt to be a matter of the trial judge and there is a fear that such technical objections were deliberately left out for appeal in case there was an undesirable outcome

The Trial Process: Jury Trials


R. v. Bryant (1984), 48 OR (2d) 732 Facts The accused was charged, inter alia, with forcible confinement, indecent assault and rape and elected trial by judge and jury He was committed for trial, was released on bail, and failed to appear on the day fixed for trial He was subsequently arrested and tried by a judge without a jury, pursuant to s. 526.1 of the Criminal Code, which provides that a person who has elected trial by jury and who fails to appear at his trial shall be tried by judge alone unless he can establish a legitimate excuse for his failure to appear Here, the accused offered no excuse but applied for an order directing that he be tried by a jury, relying on s. 11(f) of the Charter That application was dismissed Issue: Should the accused be entitled to a trial by jury after having failed to appear at his trial? Holding: Yes; appeal allowed Blair JA 206

Section 526.1(1) on its face infringes the accused's right to trial by jury under s. 11(f) of the Charter The right to trial by jury is a right given to the accused which prevails unless he or she voluntarily chooses not to utilize it by electing another mode of trial Any waiver of a jury trial must be voluntary and made with full understanding Section 526.1, which takes away an accused's right to trial by a jury where he does not provide a legitimate excuse for failing to appear, by deeming him to have elected trial before a judge alone, cannot be regarded as a waiver In carrying out a s. 1 analysis, the court found that the Crown had not established that the denial of a jury trial to absconding accused was necessary for the attainment of the purpose of s. 526.1, which was to overcome the abuse of the judicial system and other administrative problems arising from the Bail Reform Act Other sanctions were available to the Crown to punish absconding accused, and there had been no evidence that the proper operation of the Bail Reform Act and the attainment of its objectives was dependant upon s. 526.1 The section could not therefore be justified as a reasonable limit on the right to trial by jury under s. 1 of the Charter, and was of no force or effect s. 11(f) Canadian Charter of Rights and Freedoms Arrest or detention Any person charged with an offence has the right f) except in the case of an offence under military law tried before a military tribunal, to the benefit of trial by jury where the maximum punishment for the offence is imprisonment for five years or a more severe punishment;

Number and qualifications of jurors


The number is set to twelve (s. 631.(5)) for all provinces The Code delegate to the provinces through s. 626(1) the power to determine the appropriate qualifications for somebody to setve as a jury, except that s. 626(2) expresely precluded sex ddiscrimination s. 631(5) Criminal Code Drawing additional cards if necessary If the number of persons who answer under subsection (3) or (3.1) is not sufficient to provide a full jury and the number of alternate jurors ordered by the judge, the clerk of the court shall proceed in accordance with subsections (3), (3.1) and (4) until twelve jurors and any alternate jurors are sworn. 207

s. 626 Criminal Code Qualification of jurors (1) A person who is qualified as a juror according to, and summoned as a juror in accordance with, the laws of a province is qualified to serve as a juror in criminal proceedings in that province. No disqualification based on sex Notwithstanding any law of a province referred to in subsection (1), no person may be disqualified, exempted or excused from serving as a juror in criminal proceedings on the grounds of his or her sex.
(2)

In R. v. Church of Sientology (1997) it was held that the exclusion of non-citizens from jury trials did not violate ss. 7, 11(d), 11(f) or 15 of the Charter

Challenge to array
s. 629 Criminal Code Challenging the jury panel (1) The accused or the prosecutor may challenge the jury panel only on the ground of partiality, fraud or wilful misconduct on the part of the sheriff or other officer by whom the panel was returned. In writing (2) A challenge under subsection (1) shall be in writing and shall state that the person who returned the panel was partial or fraudulent or that he wilfully misconducted himself, as the case may be. Form (3) A challenge under this section may be in Form 40. s. 630 Criminal Code Trying ground of challenge Where a challenge is made under section 629, the judge shall determine whether the alleged ground of challenge is true or not, and where he is satisfied that the alleged ground of challenge is true, he shall direct a new panel to be returned.

208

Two essential themes in jury selection is ensuring impartiality and ensuring representativeness (see R. v. Born With a Tooth) A jury can be a barrier to the application of oppressive laws, as long as it is representativeness In Canada there has been a strong rejection of the US system of jury selection and racial representation, challenging the idea that jurors cannot be impartial based on their belonging to individual groups Instead of dealing with whether a jury is representative or not, in Canada we tend to be based on the assumption that if a jury is properly instructed, it will be impartial R. v. Born With a Tooth (1993), 22 CR (4th) 232 (Alta. Q.B) Facts The accused native Indian was charged with several criminal offences and elected trial by jury The sheriff issued jury summonses to 252 people, 200 of whom were selected "at random" from the municipal population The remainder were native Indians residing on reservations within the judicial district o These were selected deliberately by the sheriff from utility customer lists and other sources in order to ensure a number of individuals of native origin were on the jury panel The Crown filed a written challenge to the array on the grounds of partiality and wilful misconduct. Issue: Should jury empanelling take into consideration racial characteristics of the jurors? Holding: No; challenge upheld OLeary The two fundamental elements of the criminal jury system are impartiality and representativeness Representativeness is guaranteed by ensuring that as far as practicable the population from which jury panels are selected is representative of the whole community and by selecting jury panels from that population on a random basis Artificially skewing the composition of jury panels to accommodate the demands of any of the distinct segments of Canadian society would compromise the integrity of the jury system The effectiveness of the jury system is based on its widespread acceptance by the community as a fair and just method of deciding issues of criminal responsibility, and manipulation of the composition of juries would erode that trust There was no justification for the assumption that some otherwise qualified members of Canadian society were incapable of judging the conduct of other members of the same community in a fair and impartial manner Lecture Notes 209

The court held that this kind of affirmative action assumes that Canadians are unable to handle the trials of other community members in a affair and impartial manner and that race issues cannot be appreciated by ordinary jurors o This assumption is unjustified o This is a small step away from the calculated inclusion of one group as jurors and the exclusion of others

R. v. F.A. (1984) The person wanted the trial moved to another jurisdiction where he felt he would find a jury panel that would be more understanding to his racial circumstances The complainant did not want this because she could face retribution or criticism from community members from having brought the case forth in the first place The court found that counsel could address cultural idiosyncrasies by alluding to expert evidence R. v. Born With a Tooth (1993), 22 CR (4th) 232 (Alta. Q.B) Facts The accused native Indian was charged with several criminal offences and elected trial by jury The sheriff issued jury summonses to 252 people, 200 of whom were selected "at random" from the municipal population The remainder were native Indians residing on reservations within the judicial district o These were selected deliberately by the sheriff from utility customer lists and other sources in order to ensure a number of individuals of native origin were on the jury panel The Crown filed a written challenge to the array on the grounds of partiality and wilful misconduct. Issue: Should jury empanelling take into consideration racial characteristics of the jurors? Holding: No; challenge upheld OLeary The two fundamental elements of the criminal jury system are impartiality and representativeness Representativeness is guaranteed by ensuring that as far as practicable the population from which jury panels are selected is representative of the whole community and by selecting jury panels from that population on a random basis Artificially skewing the composition of jury panels to accommodate the demands of any of the distinct segments of Canadian society would compromise the integrity of the jury system The effectiveness of the jury system is based on its widespread acceptance by the community as a fair and just method of deciding issues of criminal responsibility, and manipulation of the composition of juries would erode that trust 210

There was no justification for the assumption that some otherwise qualified members of Canadian society were incapable of judging the conduct of other members of the same community in a fair and impartial manner Lecture Notes The court held that this kind of affirmative action assumes that Canadians are unable to handle the trials of other community members in a affair and impartial manner and that race issues cannot be appreciated by ordinary jurors o This assumption is unjustified o This is a small step away from the calculated inclusion of one group as jurors and the exclusion of others

Challenge to poll
Peremptory challenges are challenges were the parties do not need to provide reasons for the challenge of a juror The number of peremptory challenges depend on the offence The risk is that they serve to exclude people with certain demographics from the case and thus deal with the representativeness of the jury s. 632 Criminal Code Excusing jurors The judge may, at any time before the commencement of a trial, order that any juror be excused from jury service, whether or not the juror has been called pursuant to subsection 631(3) or (3.1) or any challenge has been made in relation to the juror, for reasons of (a) personal interest in the matter to be tried; (b) relationship with the judge presiding over the jury selection process, the judge before whom the accused is to be tried, the prosecutor, the accused, the counsel for the accused or a prospective witness; or (c) personal hardship or any other reasonable cause that, in the opinion of the judge, warrants that the juror be excused. s. 633 Criminal Code Stand by The judge may direct a juror who has been called pursuant to subsection 631(3) or (3.1) to stand by for reasons of personal hardship or any other reasonable cause. s. 634 Criminal Code Peremptory challenges A juror may be challenged peremptorily whether or not the juror has been challenged for cause pursuant to section 638. 211

Maximum number (2) Subject to subsections (2.1) to (4), the prosecutor and the accused are each entitled to (a) twenty peremptory challenges, where the accused is charged with high treason or first degree murder; (b) twelve peremptory challenges, where the accused is charged with an offence, other than an offence mentioned in paragraph (a), for which the accused may be sentenced to imprisonment for a term exceeding five years; or (c) four peremptory challenges, where the accused is charged with an offence that is not referred to in paragraph (a) or (b). If alternate jurors (2.1) If the judge makes an order for alternate jurors, the total number of peremptory challenges that the prosecutor and the accused are each entitled to is increased by one for each alternate juror. Supplemental peremptory challenges (2.2) For the purposes of replacing jurors under subsection 644(1.1), the prosecutor and the accused are each entitled to one peremptory challenge for each juror to be replaced. Where there are multiple counts (3) Where two or more counts in an indictment are to be tried together, the prosecutor and the accused are each entitled only to the number of peremptory challenges provided in respect of the count for which the greatest number of peremptory challenges is available. Where there are joint trials (4) Where two or more accused are to be tried together, (a) each accused is entitled to the number of peremptory challenges to which the accused would be entitled if tried alone; and (b) the prosecutor is entitled to the total number of peremptory challenges available to all the accused. s. 635 Criminal Code Order of challenges (1) The accused shall be called on before the prosecutor is called on to declare whether the accused challenges the first juror, for cause or peremptorily, and thereafter the prosecutor and the accused shall be called on alternately, in respect of each of the remaining jurors, to first make such a declaration. Where there are joint trials

212

(2) Subsection (1) applies where two or more accused are to be tried together, but all of the accused shall exercise the challenges of the defence in turn, in the order in which their names appear in the indictment or in any other order agreed on by them, (a) in respect of the first juror, before the prosecutor; and (b) in respect of each of the remaining jurors, either before or after the prosecutor, in accordance with subsection (1). Challenges for cause Prosecutors and defence do not have information about individuals that make part of the jury panel other than the name or occupation and they may have been asked by the judge if there are any obvious sources of conflict This makes it difficult to raise a claim of impartiality The most common challenge for cause is the lack of indifference between the Queen and the accused s. 638 Criminal Code Challenge for cause (1) A prosecutor or an accused is entitled to any number of challenges on the ground that (a) the name of a juror does not appear on the panel, but no misnomer or misdescription is a ground of challenge where it appears to the court that the description given on the panel sufficiently designates the person referred to; (b) a juror is not indifferent between the Queen and the accused; (c) a juror has been convicted of an offence for which he was sentenced to death or to a term of imprisonment exceeding twelve months; (d) a juror is an alien; (e) a juror, even with the aid of technical, personal, interpretative or other support services provided to the juror under section 627, is physically unable to perform properly the duties of a juror; or (f) a juror does not speak the official language of Canada that is the language of the accused or the official language of Canada in which the accused can best give testimony or both official languages of Canada, where the accused is required by reason of an order under section 530 to be tried before a judge and jury who speak the official language of Canada that is the language of the accused or the official language of Canada in which the accused can best give testimony or who speak both official languages of Canada, as the case may be. The process of challenges First, potential jurors are called based on a lottery system Then they are asked questions by both counsel, though judges place limits on allowing fishing expedition to find the perfect jurors When a challenge is raised, the trial judge first looks at whether the challenge has basis, and if it does, then two trier of facts who have been already selected will decide whether the juror can be impartial In practice, challenges are difficult to bring forth; they are most common when: 213

o There has been so much pre-trial publicity that there is reason to believe that jurors to be biased o When there is widespread racism in the community R. v. Crosby (1979), 49 CCC (2d) 255 Osler J In the absence of any notorious episode in a community indicating that prejudice is rampant, an application to challenge for cause simply on the ground that man is prejudiced and that black and white may frequently be prejudiced against each other should not be allowed Otherwise, such challenges might become everyday occurrences R. v. Sherratt, [1991] 1 SCR 509 LHeureux-Dub J The jury must perform its duties impartially and represent the larger community as far as is possible and appropriate in the circumstances The fundamental right to a fair and proper trial is denied where the accused is not allowed to challenge any number of jurors for cause when the grounds of challenge are properly specified Counsel is entitled to determine whether any potential juror is, by reason of the pretrial publicity and the notoriety of the appellant, sufficiently impartial The trial judge cannot, in the exercise of his discretion in the area of admitting grounds of challenge for cause and settling the questions, effectively curtail the statutory right to challenge for cause That trial judges have a wide discretion in these matters and that jurors will usually behave in accordance with their oaths cannot supercede the right of every accused person to a fair trial, which necessarily includes the empanelling of an impartial jury. An accused does not have the right to a favourable jury and the selection procedure cannot be used to thwart the representativeness that is essential to the proper functioning of a jury Peremptory challenges, however, are justified on a number of grounds even though they, along with the Crown's right to stand aside, can be used to alter somewhat the degree to which the jury represents the community Challenges for cause are properly used to rid the jury of prospective members who are not indifferent or who otherwise fall within s. 567 of the Criminal Code, but they stray into illegitimacy if used merely, without more, to over-or under-represent a certain class in society or as a "fishing expedition" in order to obtain personal information about the juror Information obtained on an ultimately unsuccessful challenge for cause may, however, lead the challenger to exercise the right to challenge peremptorily or to stand aside the particular juror. 214

The issue raised in a challenge for cause is tried by a "mini-jury" of two jurors or two prospective jurors if no jurors have been sworn Section 567 of the Criminal Code places little, if any, burden on the challenger On the other hand, a reasonable degree of control must be retained by the trial judge and, thus, some burden placed upon the challenger to ensure that the selection of the jury occurs in a manner that is in accordance with the principles here articulated and also to ensure that sufficient information is imparted to the trial Thus, while there must be an "air of reality" to the application, it need not be an "extreme" case A valuable distinction might be drawn in pre-trial publicity cases between mere publication of the facts of a case and situations where the media misrepresents the evidence, dredges up and widely publicizes discreditable incidents from an accused's past or engages in speculation as to the accused's guilt or innocence It may well be that the pre-trial publicity or other ground of alleged partiality will, in itself, provide sufficient reasons for a challenge for cause The threshold question is not whether the ground of alleged partiality will create such partiality in a juror, but rather whether it could create that partiality which would prevent a juror from being indifferent as to the result In the end, there must exist a realistic potential for the existence of partiality, on a ground sufficiently articulated in the application, before the challenger should be allowed to proceed Lecture Notes The jury system is a way in which the public increases it knowledge of criminal justice o Means by which the public legitimacy of the justice system is increased R. v. Williams, [1998] 1 SCR 1128 McLachlin J The prosecution and the defence are entitled to challenge potential jurors for cause on the ground of partiality Candidates for jury duty are presumed to be indifferent or impartial and this presumption must be displaced before they can be challenged and questioned Usually the party seeking the challenge calls evidence substantiating the basis of the concern Alternatively, where the basis of the concern is widely known and accepted, the law of evidence may permit a judge to take judicial notice of it The judge has a wide discretion in controlling the challenge process and should permit challenges if there is a realistic possibility that the jury pool may contain people whose racial prejudice might incline them to favour the Crown rather than the accused in deciding the matters that fall to them in the course of the trial. Judicial directions to act impartially cannot always be assumed to be effective in countering racial prejudice 215

Where doubts are raised, the better policy is to err on the side of caution and permit prejudice to be examined The expectation that jurors usually behave in accordance with their oaths does not obviate the need to permit challenges for cause where it is established that the community suffers from widespread prejudice against people of the accuseds race sufficient to create a realistic potential for partiality

The potential for partiality is irrefutable where the prejudice can be linked to specific aspects of the trial, like a widespread belief that people of the accuseds race are more likely to commit the crime charged. The trial judge has the discretion to determine whether widespread racial prejudice in the community, absent specific links to the trial, is sufficient to give an air of reality to the challenge in the particular circumstances of each case Section 638(2) of the Criminal Code requires two inquiries and entails two different decisions 1. The first stage is the inquiry before the judge to determine whether challenges for cause should be permitted o The test at this stage is whether there is a realistic potential or possibility for partiality 2. If the judge permits challenges for cause, a second inquiry occurs on the challenge itself o At this stage, the question to be determined by the triers is whether the candidate in question will be able to act impartially Section s. 638(1)(b) is intended to prevent persons who may not be able to act impartially from sitting as jurors o This object cannot be achieved if the evidentiary threshold for challenges for cause is set too high The appropriate evidentiary standard on applications to challenge for cause based on racial prejudice is a realistic potential for partiality (the rule in R. v. Sherratt) o Prejudice less than widespread might in some circumstances meet this test. A judges discretion to allow challenge for cause must be exercised in accordance with the Canadian Charter o Section s. 638(1)(b) should be read in light of the fundamental rights to a fair trial by an impartial jury and to equality before and under the law o The rule in Sherratt suffices to maintain these rights without adopting the United States model or a variant on it o It protects the accuseds right to a fair trial by an impartial jury and the privacy interests of prospective jurors while avoiding lengthening trials or increasing their cost Lecture Notes The question of how challenges for cause can be made, was clarified in this case First, there is an inquiry before a judge to see if challenges are allowed at all Two statements were made by the Court with regards to when challenges of cause will be 216

allowed o First, when there is a realistic potential of partiality in the jury pool due to racial bias This test can be passed through judicial notice o Later on the SCC (in Fine) decided that these threshold test is not met just because the case deals with sexual assault

The Trial Process Further Considerations


The adversarial system The best way to discover the historic truth of a case and get its full picture, is allowing both parties to battle out This is not so much about factual and scientific truth Jerome Frank, Courts on Trial: Myth and Reality in American Justice We shouldnt blame the lawyers for misconstruing the truth People should have more appropriate representation and they should have better access to that representations R. v. Murray (2000), 144 CCC (3d) 289 Facts The accused lawyer was charged with attempting to obstruct justice by concealing videotaped evidence of crimes committed by client charged with murder Accused was instructed by client to retain tapes but not reveal their contents, and claimed his intention was to use them at trial to discredit witness for prosecution Issue: Was the accused guilty of obstructions of justice? Holding: Yes; appeal dismissed Gravely J Applying the "tendency test," the actus reus of the offence of attempting to obstruct justice is the doing of an act which has a tendency to pervert or obstruct the course of justice. Wilfully" doing the act for the purpose of obstructing the course of justice constitutes the mens rea. On the face of the evidence, the accused's act in concealing the critical tapes had the tendency to obstruct the course of justice at several stages of the proceedings While the accused's act in concealing the tapes was prima facie caught by the tendency test, he could not be said to have attempted to obstruct justice if he had legal justification for his conduct. While the accused's discussions with his client about the tapes were covered by solicitorclient privilege, the physical objects themselves were not 217

Solicitor-client privilege protects communications between solicitor and client o These tapes were not "communications," but dramatic evidence of crime which preexisted the solicitor-client relationship

Nor was concealing the tapes permissible on the basis that they had some exculpatory value, since they were overwhelmingly inculpatory Not only does solicitor-client privilege not protect physical evidence, but there is also a suggested obligation on defence counsel to turn over incriminating physical evidence to the prosecution By putting the tapes beyond the reach of the police and the Crown, the accused clearly intended to impede the prosecution Despite the inferences that could be drawn against the accused's credibility based on his actions, a defence strategy to use the tapes at trial was reasonably feasible That tended to support the accused's evidence that he did not intend to permanently suppress them The conduct in which he engaged was not specifically criminalized by s. 139(2) The only official guide on this issue is contained in the Law Society's Rules of Professional Conduct, which states that a lawyer must not knowingly attempt to influence the course of justice by suppressing evidence that "ought to be disclosed," but provides no guidance as to the nature of such evidence Assuming that the accused did intend to use the tapes in the defence, he may well have believed that he had no legal duty to disclose the tapes until resolution discussions or trial. Where credibility is a significant issue, the accused must be acquitted if his evidence is believed; or if his evidence is not believed, but a reasonable doubt remains as to his guilt after considering his evidence in the context of the evidence as a whole

Res Judicata
Two principles A person should not be tried for the same events There should not be multiple punishments Res judicata also serves to Save judicial resources Maintain the legitimacy of the judicial system Spare the accused from multiple trials and from being found guilty for one charge based on the outcome of others s. 607 Criminal Code 218

Special pleas (1) An accused may plead the special pleas of (a) autrefois acquit; (b) autrefois convict; and (c) pardon. In case of libel ( 2) An accused who is charged with defamatory libel may plead in accordance with sections 611 and 612. Disposal (3) The pleas of autrefois acquit, autrefois convict and pardon shall be disposed of by the judge without a jury before the accused is called on to plead further. Pleading over (4) When the pleas referred to in subsection (3) are disposed of against the accused, he may plead guilty or not guilty. Statement sufficient (5) Where an accused pleads autrefois acquit or autrefois convict, it is sufficient if he (a) states that he has been lawfully acquitted, convicted or discharged under subsection 730(1), as the case may be, of the offence charged in the count to which the plea relates; and (b) indicates the time and place of the acquittal, conviction or discharge under subsection 730(1).

1. Double jeopardy
The rule against double-jeopardy is set to protect the person for being convicted twice for the same event s. 11(h) Canadian Charter of Rights and Freedoms Arrest or detention Any person charged with an offence has the right h) if finally acquitted of the offence, not to be tried for it again and, if finally found guilty and punished for the offence, not to be tried or punished for it again R. v. Riddle, [1980] 1 SCR 380 Facts 219

Issues: 1. Can the accused raise the plea of autrefois acquit in a summary conviction court? 2. If so, is the plea available when in an earlier proceeding the charge is dismissed following nonappearance of the informant and refusal of an adjournment? Holding: 1. Yes, 2. Yes; appeal dismissed Dickson J The proper procedure in summary conviction matters is not to raise the special plea of autrefois acquit, but simply to enter a general plea of not guilty embracing the concept of res judicata

Accused was charged with common assault He pleaded not guilty and the matter was adjourned for trial At the trial the Crown applied for an adjournment as the complainant was not present. The application was refused. As the Crown called no evidence, the charge was dismissed and the accused was discharged. A week later, the complainant swore a new information in terms identical to the first information When the matter came on before another provincial court judge, the accused entered a plea of autrefois acquit and the charge was dismissed The Alberta CA dismissed the Crowns appeal

The submission that the special pleas authorized in Part XVII (ss. 534-537) of the Criminal Code, headed Procedure by Indictment, are not available in respect of summary conviction offences, provision for which is contained in Part XXIV, failed Something akin to the plea of autrefois acquit was available at common law to a defendant accused of an offence punishable by summary conviction Section 743 (now 808) is intended to supplement, and not to supplant, common law rights In principle, there is no reason why a different situation ought to prevail where the defendant has not obtained the certified copy as per s. 743 Nor, in principle, is it easy to distinguish between the situation where the Crown leads evidence which fails to make out a case for the defendant to answer and the situation where, as here, no evidence is led So long as the case has proceeded to a verdict and a dismissal, that should be sufficient Lecture Notes The principle that the person must not be put in jeopardy twice for the same offence is a principle of fundamental justice This rule can be pleaded in summary convictions The case also set out when an accused was to be considered previously in jeopardy o When there was an acquittal or conviction, this is obviously the case o The more difficult question is when short of an acquittal or a conviction a person is in jeopardy? Jeopardy begins when there has been a plea and the judge has the capacity 220

of entering a conviction A trial that is stayed and not recommenced, is considered not to have happened at all So the person is not in jeopardy If the information is flushed before there being a plea, then there is no jeopardy either

When are matters considered identical? Some of these matters are answered in ss. 609 and of the Code It is not whether they just make part of the same transaction, but whether the accused could have been convicted of the first charge based on the evidence that has been tendered for the second s. 609 Criminal Code What determines identity (1) Where an issue on a plea of autrefois acquit or autrefois convict to a count is tried and it appears (a) that the matter on which the accused was given in charge on the former trial is the same in whole or in part as that on which it is proposed to give him in charge, and (b) that on the former trial, if all proper amendments had been made that might then have been made, he might have been convicted of all the offences of which he may be convicted on the count to which the plea of autrefois acquit or autrefois convict is pleaded, the judge SHALL give judgment discharging the accused in respect of that count. Allowance of special plea in part (2) The following provisions apply where an issue on a plea of autrefois acquit or autrefois convict is tried: (a) where it appears that the accused might on the former trial have been convicted of an offence of which he may be convicted on the count in issue, the judge shall direct that the accused shall not be found guilty of any offence of which he might have been convicted on the former trial; and (b) where it appears that the accused may be convicted on the count in issue of an offence of which he could not have been convicted on the former trial, the accused shall plead guilty or not guilty with respect to that offence.

R. v. Wigglesworth, [1988] 1 SCR 541 Facts Appellant police officer committed a common assault, as defined in the Criminal Code, which was also a "major service offence" under the Royal Canadian Mounted Police Act The major service offence was dealt with first The trial judge quashed the information for the charge of common assault under s. 24(1) of the Charter on the ground that the accused was being tried twice for the same 221

misconduct contrary to s. 11 of the Charter The Saskatchewan Court of Queen's Bench allowed an appeal from that judgment holding that the common assault charge and the major service offence constituted separate offences An appeal to the Court of Appeal was dismissed Issue: Does a "major service offence" under the Act precluded subsequent proceedings under the Criminal Code for the same misconduct on the ground that such proceedings would violate the accused's right, under s. 11(h) of the Charter? Holding: No; appeal dismissed Wilson J A matter could fall within s. 11 either because by its very nature it is a criminal proceeding or because a conviction in respect of the offence may lead to a true penal consequence In cases where the two tests conflict the "by nature" test must give way to the "true penal consequence" test. If a particular matter is of a public nature, intended to promote public order and welfare within a public sphere of activity, then that matter falls within s. 11 This is to be distinguished from private, domestic or disciplinary matters which are regulatory, protective or corrective and which are primarily intended to maintain discipline, professional integrity and professional standards or to regulate conduct within a limited private sphere of activity The proceedings before the Royal Canadian Mounted Police Service Court are accordingly neither criminal nor quasi-criminal proceedings However, an officer charged and convicted under the Code of Discipline faces a true penal consequence since conviction can result in imprisonment for one year Nevertheless, appellant does not have the benefit of s. 11(h) because he was not being tried and punished for the same offence The "offences" were quite different; one was an internal disciplinary matter where the accused was found guilty of a major service offence, and the other was the criminal offence of assault where the accused must account to society at large for his conduct Lecture Notes RCMP officer convicted of assault under RCMP proceedings He later appears in provincial court for the same assault One question was whether the RCMP committee could be have been considered to have given a penal sanction Was the RCMP proceeding a criminal proceeding? o The first question is whether the proceeding was penal by nature The court finds that the proceeding was not by nature penal, but instead regulatory o The second question was whether the proceeding have penal consequences, however? If the consequences are such that they amount to being penal, then it could 222

offend s.11(h) S. 11 was not considered infringed in this case, however, because the accused had committed two separate offences o Assault AND an assault that is contrary to his duties o It is thus acceptable to be punished for both

2. Multiple punishment
The policy forbidding multiple punishment has at its aim that a wrongdoer receives an adequate penalty, but not more R. v. Kinnear (2005), 30 CR (6th) 1 (Ont. CA) Facts Accused was charged with theft, threatening death, escaping lawful custody, using imitation firearm while committing indictable offence of threatening death, using imitation firearm while committing indictable offence of escape custody Trial judge entered convictions on all counts Trial judge, on his own initiative, raised question of constitutionality of s. 85(4) of the Criminal Code which provides that any sentence imposed upon conviction on charge of using imitation firearm while committing indictable offence shall be served consecutively to any other sentence imposed at same time The judge ruled that s. 85(4) contravened prohibition against cruel and unusual punishment in s. 12 of the Charter There was also the issue that some of the convictions should be set aside pursuant to the Kienapple rule against multiple convictions Issue: Were the multiple convictions? Holding: Yes; appeal allowed Doherty JA The court applied the Kienapple rule to stay two of three charges relating to use of firearm Section 85(4) does not abrogate Kienapple rule The rule in Kienapple applies unless Parliament has clearly indicated otherwise In R. v. Kienapple the court recognized and arguably expanded the common law rule prohibiting more than one conviction for the same criminal wrong The principle foreclosed more than one conviction for offences arising out of the same delict The relevant inquiry so far as res judicata is concerned is whether the SAME CAUSE OR MATTER (rather than the same offence) is comprehended by two or more offences. The principle that emerged from Kienapple provides that where the same transaction gives rise to two or more offences with substantially the same elements and an accused is found guilty of more than one of those offences, that accused should be convicted of only the 223

MOST SERIOUS of the offences The other charges should be stayed Laskin J linked the Kienapple rule to the court's power to protect against abuses of its process He described the rule as designed to "protect an individual from an undue exercise by the Crown of its power to prosecute and punish". Lecture Notes The accused was caught shoplifting socks, when stopped by the police he pulled a gun to force the guard to let them go The gun was found to not be real He was charged with two counts of using an imitation firearm while carrying out an indictable offence He was also charged with using an imitation gun while running from a police officer o These law two charges could have been considered redundant The court uses a factual and legal nexus so as to assess the violation of the the Kienapple principle (i.e. two offences with substantively the same elements) o In this case both offences did in fact arise from one single act o Did these constitute a single criminal wrong or delict? o The crucial distinction from Kienapple is one between different wrongs, and the same wrong being committed in different ways The court finds that some charge here did deal with the same wrong committed in different ways o Being charged with possessing a firearm while committing an indictable offence and possessing a firearm while committing a specific offence (which is indictable) amounts to the same wrong

224

Você também pode gostar